You are on page 1of 155

MULTIPLE-

CHOICE
1, She gets up at 6 and meditates ____________ an hour before going to work.
A. during B. while C. for D. through
2, He ____________ robbed as he was walking out of the bank.
A. had B. did C. got D. were
3, Three years ago, my father ____________ with cancer.
A. was diagnosed B. had diagnosed C. diagnosed D. was being diagnosed
4, She was working on her computer with her baby next to ____________.
A. herself B. her C. her own D. hers
5, Some actions that are habits for ______________, may look weird to people from other
nationalities.
A. the Japanese B. the Japanese people C. the Japanese D. Japanese
6, My mother would like to consider herself _______________ but I think even she would be
shocked if we took her to see that film!
A. broad-minded B. independent C. stubborn D. well-behaved
7, Police arrested a man late last night in connection with the murder. Police expect to charge the
____________ later this morning.
A. guilty B. suspect C. condemned D. arrested
8, I'm sorry ____________ you all this time. It wasn't my intention.
A. to be ignored B. to have been ignored C. to have been ignoring D. to ignore
9, Marie Curie, one of the best-known ___________ in working with radiation, died in 1934.
A. debutants B. revolutionaries C. pioneers D. rebels
10, Much of the neighbourhood was demolished in the 1940s when living ____________ had
deteriorated.
A. situations B. conditions C. circumstances D. states
11, John: "Do you know when Professor Smith's expected to give us our grades?"
Jessa: "I'm certain she'll do it ______________.”
A. as soon as she's able
B. she's able to do it soon
C. soon she's able to
D. however she's soon able to
12, I'm recent years, software development has begun shifting to agile development practices
_____________ within organizations.
A. because of speed changing
B. due to the speed of change
C. as long as the changing speed
D. as a result of changed speeding

13, Despite its prevalence and serious effects, diabetes research has historically been
underfunded ______________ other diseases.

A. for research compared to


B. comparing the research of
C. compared to research for
D. in comparison with researching

14, The walls of the local military command were ____________ by anti-government graffiti
and that was the first sign of general rebellion in the city.

A. defaced B. destroyed C. mutilated D. deformed

15, They ______________ off from London in a minivan, armed with little more than a strong
_____________ of adventure.

A. headed - thirst B. appetite - hugely C. dropped - buzz D. put - spirit

16, No pharmaceutical compounds have proven effective _______________ neurons or helping


the body establish new neural pathways.

A. in permanently restoring
B. permanent restoration of
C. at restoring permanence of
D. at permanent restoration in

17, Peter: "Do you think it would be okay to install an app on this computer?”
Alex: "We were clearly told ___________ anything onto the school computers.”
A. not to download
B. not downloading
C. don't download
D. to not downloading

18, The comparison to others as a ______________ for social class continues and is being
driven by a variety of factors, including prolific advertising and easy credit to _______________
consumers.

A. guideline - lobby B. benchmark - endorse C. pattern - hype D. point - entice

19, What at first appeared a random hotchpotch of activity became a pattern of elegant waves
_______________ determined.

A. rhythmically B. rhythmicity C. rhythm D. nonrhythmic


20, _______________, the same students who participate in the program in winter should
continue to work on the project in spring, to ensure continuity.

A. Ideally B. Promptly C. Afterwards D. Beforehand

VERB - FORM (LEVEL: B2 – C2)

In all mysteries, there is usually someone who knows what really happened. Our English
teacher told us about a particular class she ___________ (have) when she first
___________ (start) teaching. One hot summer day, when everybody wished they were
at the beach, she ___________ (teach) phrasal verbs. She noticed one student
___________ (chew) what must have been a whole packet of gum. She ___________
(tell) him to get rid of the gum, which he did immediately. He ___________ (stand) up,
throwing the gum out the window, and it went flying straight through the open window of
a flat opposite the school. The whole class ___________ (go) silent. Honestly, she told us
that he couldn’t have timed it better if he had tried. After that, everyone ___________
(suggest) where the gum ___________ (land). One explanation was that the gum had
stuck to the TV screen. Then everyone wondered about the people who ___________
(live) there. What in the world would they think when they ___________ (see) it? Those
people would have been quite puzzled when they ___________ (find) the gum, no matter
where it had ended up in their house. Anyway, the case of the mysterious gum was
certainly no mystery to the class or our teacher.

The holiday of a lifetime


Clare and her husband, Adam, had been planning their trip to Africa for weeks. They
___________ (decide) to spend a few days relaxing on the beach before embarking on
what they hoped would be the adventure of a lifetime. The first day had been wonderful.
They'd seen elephants, giraffes, buffaloes, and hyenas. But it was when night
____________ (fall) that the real adventure began. They ____________ (set up) camp,
made a fire, and had some dinner. They decided they'd have an early night as it
____________ (be) a long day. They ____________ (not sleep) for long when Clare
suddenly woke up. She was sure she'd heard thunder, but then she realized the ground
____________ (shake) and their guide was shouting and clapping outside the tent. By
now Adam and Clare ____________ (be) wide awake. Terrified, they unzipped their tent
and peered out just as a herd of zebras ____________ (stampede) past them in a cloud of
dust, narrowly avoiding them and their guide. ‘I expect it's lions out hunting’,
____________ (explain) their guide, but don't worry. They won'tcome into our camp.'
The next morning, when they got up after a long and restless night, Adam and Clare
____________ (find) lion prints all around their tent.
Rhino farming
The South African White Rhino could become extinct in the wild in the next two decades.
They ____________ (be) almost extinct once before, at the end of the nineteenth century,
but ____________ (buy) back from the brink of extinction by successful conservation
efforts. The rhino ____________ (hunt) for its horn, and poaching is by far the greatest
threat to its existence. In many Asian countries, powdered rhino horn ____________
(believe) to treat anything from fevers to cancer, although there is no scientific evidence
whatsoever to support these claims. Now there are calls to manage rhinos under farmed
conditions and legalize the trade in one of the most valuable animal products on earth.
There are plenty of supporters who ____________ (be) only too eager to adopt this
method of sustainable use, which ____________ (involve) sedating rhinos and then
removing their horns, a practice not nearly as inhumane as killing such truly magnificent
creatures for their horns. The horn-like a nail, grows back, so it ____________ (will /
theoretically) possible to 'harvest' rhino horn several times from a single animal. But is it
ethically correct to use this creature ____________ (support) a market in quack
medicine?

WORDFORM

I. SENTENCE
1, My uncle usually takes a lot of ___________ photos. (BEAUTY)
2, Sugar is not an _____________ food because we need it to live. (HEALTH)
3, Have you finished that _____________ for History class yet? (COMPOSE)
4, I think were all in _____________ that something must be done about the problem.
(AGREE)
5, Megan was told by her dietician that she was becoming ______________ thin and
should eat more. (DANGER)
6, Elliot recently read a survey about people’s feelings on ____________ intelligence.
(ART)
7, All the coastal towns in this area are ______________ by the strong tradition of dance
and there is a festival of dance that takes place every August. (CHARACTER)
8, Australia is renowned for the ______________ of its fauna, with native animals unlike
any others in the world. (DIVERSE)
9, The party was absolutely fab, but ______________, I spilled a drink on Kate's new
white dress and she was furious with me. (FORTUNE)
10, Researchers have been studying the ______________ phenomenon is known as false
memory since the mid-1970s. (PSYCHOLOGY)
11, The sun generates heat by forcing ______________ atoms to bond with one another
and release energy through a process of physics called nuclear fusion. (HYDRO)
12, What at first appeared a random hotchpotch of activity became a pattern of elegant
waves _______________ determined? (RHYTHM)
13, The latest suggestion is that the rhythms could be ________________ in detecting
processes going on in different regions of the brain. (DECIDE)
14, Although functional, durable, and perhaps fashionable in a different era, the grey,
utterly utilitarian clothing made its wearers look robotic and _______________.
(EMOTE)
15, Technology's continual surge forward is well evidenced in television, advertising, and
_______________ so it's not difficult to deduce that the general public's knowledge of
computers. (MEDIA)

KEY:
1, beautiful
2, unhealthy
3, composition
4, agreement
5, dangerously
6, artificial
7, characterized
8, diversity
9, unfortunately
10, psychological
11, hydrogen
12, rhythmically
13, decisive
14, emotionless
15, multimedia

II. PARAGRAPH
KEY:
1, psychological
2, participants
3, composition
Some of the most colorful and entertaining ways to use the English language are colloquialisms. A
colloquialism is a word, phrase or language convention used in informal English.

THE PANDEMIC OF CORONAVIRUS STRAIN


The COVID-19 pandemic, otherwise known as the coronavirus pandemic, is an ongoing global
pandemic of coronavirus disease 2019 (COVID-19) inflicted by severe acute respiratory
syndrome coronavirus 2. Thereafter the virus was initially identified in December 2019 in
Wuhan, China. Immediately after the epidemic leads off, Chinese scientists sequenced the
genome of SARS-CoV-2 and made the data available to researchers worldwide. The resulting
genomic sequence data has shown that Chinese authorities rapidly detected the epidemic and
that the amount of COVID-19 cases have done increasing because of human-to-human
transmission afterward singular introduction with the human population.
Manifestations concerning COVID-19 are profoundly variable, ranging from none to life-
threateningly severe. Transmitting COVID-19 frequently occurs while people are uncovered to
respiratory droplets or small airborne particles exhaled by a contaminated anyone.
Recommended preventive measures comprise social distancing, wearing face masks in public,
ventilation and air-filtering, hand washing, covering one's mouth when sneezing or coughing,
disinfecting surfaces, and monitoring and self-isolation for people uncovered or symptomatic.
Numerous vaccines have been full-blown and widely distributed since December 2020.
Contemporary treatments concentrate on addressing symptoms, and work ought to underway to
develop medications that prevent the virus. Nevertheless, the pandemic brought up controversy
about racial and geographic discrimination, health equity, and the balance amid public health
imperatives and individual entitlements.
~ Laura: "................" ~ Bob: "That's great. Congratulations!"
VĂN BẢN A VĂN BẢN B
Mặc dù mọi sự quảng cáo về các công Khi trang web lần đầu tiên giúp Internet
việc hiện đại mà sẽ không thể tưởng tượng có thể truy cập trên toàn thế giới, không ai
được với các thế hệ trước, thực tế là, tôi dự đoán sẽ có những vị trí như người tối
tin rằng phần lớn lực lượng lao động vẫn ưu hóa công cụ tìm kiếm, quản lý phương
đang được sử dụng trong nghề truyền tiện truyền thông xã hội và vô số công
thống như bán hàng. Hầu hết các hành việc khác liên quan đến công nghệ ngày
động và quyết định của công nhân đều có nay. Hơn nữa, ngay cả những công việc
thể được dự đoán, dựa trên những gì họ đã trông giống như cách đây một thế kỷ cũng
làm trong những tình huống tương tự, và rất khác bây giờ. Ví dụ: các nhân viên
phần lớn những công việc dự đoán này sẽ ngân hàng vẫn quan tâm đến các nhiệm vụ
dễ bị ảnh hưởng bởi tự động hóa trong như xử lý tiền mặt cơ bản. Tuy nhiên, họ
những thập kỷ tới. Hơn nữa, vấn đề là liệu cũng đã đảm nhận những vai trò đòi hỏi
các công việc có được tạo ra bởi công chuyên môn cao hơn như “mối quan hệ
nghệ sẽ đủ nhiều để bù đắp cho những ngân hàng”. Khía cạnh mới này của vai
công việc đã biến mất hay không. Và trò liên quan đến điều mà không máy móc
trong khi chắc chắn sẽ có nhiều cuộc gọi nào có thể làm được: xây dựng mối quan
để cải thiện cơ hội đào tạo, điều đó không hệ và củng cố lòng trung thành của khách
thực tế khi mong đợi rằng phần lớn lực hàng, tư vấn về một loạt các dịch vụ tài
lượng lao động có thể được dạy để thực chính khác. Thật vậy, khi công nghệ tiếp
hiện một số vai trò vượt quá tầm với của quản nhiều công việc thường ngày hơn,
công nghệ. Tuy nhiên, điều này không có các năng lực như đối xử thiện cảm với
nghĩa là chúng ta nên bỏ lỡ cơ hội để bắt khách hàng sẽ ngày càng quan trọng khi
đầu các cuộc thảo luận có ý nghĩa về việc đề cập đến khả năng tuyển dụng. Chúng ta
làm, hay đúng hơn là thất nghiệp, mà có thể tự tin rằng xu hướng này sẽ tiếp tục
chúng ta đối mặt với tư cách một xã hội và và chắc chắn nhất là lúc chúng tôi bắt đầu
các loại chiến lược mà chúng ta có thể sử nói về các chính sách của chính phủ để
dụng để thích nghi với một thực tại mới. đối phó với những thay đổi sắp tới, cả về
việc làm và cách chúng tôi thực hiện
chúng.
VĂN BẢN A VĂN BẢN B
Cách đây một thời gian, một trang web đã nêu bật Trẻ nhỏ thường được cho là những chuyên gia thực
những rủi ro của việc đăng ký công khai - thông báo sự khi học ngôn ngữ. Cho đến khi khoảng bảy tuổi,
trực tuyến về nơi ở của bạn. Quan điểm của trang web chúng có thể tiếp thu ngôn ngữ mà chúng tiếp xúc
khá phiến diện: bạn có thể nghĩ rằng bạn chỉ đang nói mà không cần giáo viên giải thích sự khác biệt giữa
với cả thế giới rằng, 'Này, tôi đang ở đây' - nhưng bạn các thì, hoặc giữa chủ ngữ và đại từ tân ngữ. Bằng
cũng đang quảng cáo những điều gần gũi của mình cho cách nào đó, họ chỉ xoay sở để có được nó, và họ
tất cả mọi người ở khắp mọi nơi - không phải tất cả họ làm như vậy một cách vô thức, hay nói cách khác là
là những người bạn có thể muốn gặp. Điều này dường không nỗ lực thực sự. Vì vậy, có lẽ không có gì ngạc
như chứng tỏ nhận thức ngày càng tăng rằng có thể có nhiên khi rất nhiều khóa học, ứng dụng và tài liệu
mặt trái đối với tất cả việc chia sẻ rầm rộ trên web đã giảng dạy ngôn ngữ tuyên bố giúp bạn học ngoại ngữ
được kích hoạt. Cơ hội mới rộng lớn để xuất bản bất kỳ khi trưởng thành giống như cách bạn học ngôn ngữ
và mọi khía cạnh trong cuộc sống của chúng ta cho đầu tiên khi còn nhỏ. Nhưng điều đó có khả thi
khán giả toàn cầu tiềm năng mang đến tất cả các khả không? Hay thậm chí đáng mơ ước? Không nên
năng trêu ngươi: Sự giàu có! Danh tiếng! Vì vậy, chúng đánh giá thấp những người học trưởng thành. Một
ta lao vào vòng xoáy của internet, quăng những lời thú đứa trẻ có thể mất bảy năm để trở nên thành thạo
tội, ảnh cá nhân và câu chuyện vào vòng xoáy kỹ thuật một cách hợp lý (mặc dù vốn từ vựng bị hạn chế),
số. Quá muộn, chúng tôi nhận ra rằng nước đông đúc trong khi một người lớn có thể đạt đến khả năng giao
và nguy hiểm - và chúng tôi đã bị lạc. Chán nản? Có lẽ, tiếp nâng cao trong một năm. Điều đó nghe có vẻ
nhưng đừng bỏ cuộc. Tương lai này có một bản đồ, giống như một tuyên bố táo bạo, nhưng tôi là bằng
được vẽ cho chúng ta nhiều năm trước bởi một nhóm chứng sống, đã đạt được trình độ trung cấp đến cao
người tiên phong trực tuyến liều lĩnh. Trong những cấp trong 16 người trong số họ - và hầu hết trong số
ngày đầu của web, họ đi thuyền trên vùng biển này và đó là khi trưởng thành. Đối với tôi, chìa khóa để học
tìm thấy tất cả các bãi cạn nguy hiểm. Trong những rất nhiều ngôn ngữ là khả năng kết hợp các phương
ngày đầu của trang web, họ đi thuyền trên vùng biển pháp vô thức mà chúng ta đã sử dụng khi còn nhỏ
này và tìm thấy tất cả các bãi cạn nguy hiểm. Họ kiếm với các phương pháp có ý thức được sử dụng bởi
được nhiều tiền từ công việc của mình, tìm thấy và người lớn. Chúng ta có thể đạt được điều tốt nhất của
đánh mất bạn bè, điều hướng những cám dỗ và nguy cả hai thế giới bằng cách tuân theo năm nguyên tắc
hiểm của người nổi tiếng - tất cả đều rất lâu trước khi của tôi, để trở thành những người học từ vựng
phát minh ra mạng xã hội. chuyên nghiệp, bất kể ngôn ngữ nào.

VĂN BẢN C

Rafflesia là một loài thực vật ký sinh độc đáo được tìm thấy ở Đông Nam Á. Rafflesia được ví
như một loại nấm vì nó thiếu chất diệp lục và không thể thực hiện quá trình quang hợp. Như bạn
có thể tưởng tượng, phần duy nhất của Rafflesia có thể được phân biệt rõ ràng như một cây là
hoa, đây là bông hoa lớn nhất trên thế giới. Nhiều nhà thực vật học rất muốn khám phá lý do tại
sao những bông hoa lại lớn như vậy, vì vậy gần đây họ đã thực hiện các phân tích phân tử trên
thực vật. Kể từ khi nó được thành lập như là một chồi nhỏ của nụ hoa khổng lồ nặng bảy ký ngày
nay, nó đã phát triển gần 80 lần, điều này đã khơi dậy sự giác ngộ của mọi người. Mặc dù sự biến
đổi phi thường này đã diễn ra hàng chục triệu năm, nhưng sự thay đổi nhanh chóng này vẫn là
một trong những thay đổi mạnh mẽ nhất trong lịch sử. Tốc độ phát triển của con người cao như
vậy có thể so sánh với chiều cao hiện tại của chúng ta là 146 mét. Mặt khác, loại cây này cũng
không bình thường. Mùi của nó rất khó chịu, nhưng đặc tính khủng khiếp này lại thu hút các loài
thụ phấn như ruồi. Người ta tin rằng hoa Rafflesia lớn có thể khơi dậy mùi hôi trong thời gian dài.
How strong language changes memories?
Researchers have been studying the psychological phenomenon known
as false memory since the mid-1970s. The first experiments involved
having participants watch videos of car accidents and after they did,
researchers would ask some of them the question, 'How fast were the
cars going when they smashed into each other?' Other participants were
asked the same question, but the verb smashed was changed to either
hit, collided, bumped, or contacted, and depending on the verb,
participants would answer differently.
In another experiment, researchers asked participants to recall certain details about the
accidents. Even though broken glass had not appeared in any video, some would purport
that they had in fact seen glass break if the researcher used a more dynamic verb in the
question. Nowadays, researchers are studying the phenomenon to help people recall
events in their lives so painful that they had been burying their memories of them for
decades.
This phenomenon also raises questions about the accuracy of witness testimonials in the
legal arena. In the past, courts used to often accept eyewitness testimonies as unassailable
facts, yet research has since shown that our memories can be manipulated by a multitude
of factors, such as a barrister's choice of words or aggressive witness cross-examination
during the trial.

School: then and now


Education has changed enormously over the years. For a start, curriculums have changed
to better reflect the needs of students. Fifty years ago, home economics and woodwork
exemplified subjects that could prepare students for adult life. Nowadays, life skills are
offered alongside a range of academic subjects, and students now have to sit standardized
tests in reading, writing, and maths.
Another difference is that students were subject to tougher disciplinary action, which
often involved corporal punishment, for being disruptive or acting disrespectfully —
towards teachers. Learning methods are constantly changing.
Memorizing information based on repetition, known as rote learning, has fallen out of
favor. We are heading towards virtual assimilation classrooms where teachers will be able
to recreate a 3D ancient Roman villa in a history lesson or turn a classroom into a galaxy
where students can study natural phenomena like meteor showers.
SWEATSHOPS
Poor working conditions, long hours, and low wages are what millions of children endure
in unregulated garment factories. Known as sweatshops, these facilities are common in
developing countries where labor laws are often not enforced. A factory can be nothing
more than a room with ten or so sewing machines. The children, most of whom do not
attend school, undertake several sewing tasks such as stitching labels into clothes.
They scrape a living in order to support themselves and their families. Working six or
sometimes seven days a week from sunrise to sunset, they sleep, eat and shower at the
factory which, in essence, is their home. The practice amounts to child exploitation and
denies them any hope of a brighter future since it is done at the expense of their education.
But until there is an end to the grinding poverty that exists in some countries, child labor
in sweatshops is likely to continue.

Learning more than how to surf


Technology's continual surge forward is well evidenced
in television, advertising, and multimedia, so it's not
difficult to deduce that the general public's knowledge of
computers must develop at the same pace.

The level of computer literacy required to succeed in life


is hardly limited to emailing, surfing the web, and
having a basic knowledge of Microsoft Word. A
seasoned employee is likely to be well versed in the ins
and outs of computer software and hardware, at least to
some extent. In fact, former UK culture minister Ed Vaizey recently postulated that knowing
how a computer works would be 'on a par with knowledge of the arts and humanities. He also
suggested that knowing a bit about computer coding would acquaint people with the digital
world in a way that would allow them to become active participants in its evolutionary process.

Acquiring above-average skills in computer usage isn't even all that's required, because as
computer systems and programs become obsolete, users have to learn new sets of keystrokes,
menu options, and interfacing. It goes far beyond basic office management and data processing;
medical equipment is often tethered to a computer system that aids doctors and nurses in
interpreting vital data. While there may still be some fields of employment in which computers
take a back seat to the work (think fishing), there isn't a job, field, or profession that wouldn't
benefit from a digital revolution. Come to think of it, even the smallest of fishing boats are
likely to be fitted with GPS devices!

III. FILL IN THE BLANKS


(exciting/ green cities/ modern-day/ construction/ proposed/ ecological/ aim)

Ranked amongst the top five (1) _____________ of the future, Sherford in England has
got to be one of the most (2) _____________ urban projects of the 21st century. With
(3) _____________ due to commence in 2014, the thinking behind the new town is to
provide a solution to urban sprawl slum areas and the (4) _____________ problems
that come with them. In other words, it will be a shining example of sustainable
urbanism.
The (5) _____________ location for the new town is in one of the greenest parts of the
English countryside: Devon, Although it will be a modern town with all the necessary
facilities for (6) _____________ living, the town will have a traditional feel to it. Local
architectural designs will be used to make sure the town has a distinctive Devon
character. Developers, Red Tree, claim that their (7) _____________ is for Sherford to
take 'the best from the past and move it forward. They have a vision of a town where
social, economic, and environmental sustainability will prosper.

KEY:
1, green cities
2, exciting
3, construction
4, ecological
5, proposed
6, modern-day
7, aim

2. CONTENT:
(exciting/ green cities/ modern-day/ construction/ proposed/ ecological/ aim)

MAYAN TECHNOLOGY
The ancient Maya, who lived in parts of present-day Central America, had a very
sophisticated, complex civilization and achieved remarkable things. By observing
the skies they created a highly accurate calendar; they introduced new farming
methods, and they managed to build spectacular temples and great cities
without the use of metal or the wheel.

One of their greatest accomplishments is vulcanization, which is the process of


combining rubber with other materials to make it stronger and longer-lasting.
Until recently, the discovery was credited to Charles Goodyear of the United
States, who patented the technology in 1843. However, scientists and historians
now believe that the Maya were producing rubber products from around 1600 BC
- about 3,400 years before Goodyear. In fact, in the 16th century when Spanish
explorers arrived in the area, they were astonished to see rubber balls, which
were objects they had never encountered before. Even describing the stretchy,
bouncy material proved difficult as it did not exist in Europe and was therefore
beyond their experience.

3. CONTENT:
(exciting/ green cities/ modern-day/ construction/ proposed/ ecological/ aim)

The future place in space


Many people wonder if what they saw in Total Recall, a film in which humans inhabited
Mars, would happen one day in their lifetime. While space exploration mostly centers
around travel to Mars, scientists have set their sights on destinations further out in our
solar system once the technology to do so gets up to speed. In fact, when mankind has
reached the fourth planet from the sun, we will have likely found ways to reach arguably
far more intriguing celestial bodies, at least in terms of their potential to sustain life.
One such destination is Europa, one of Jupiter's four Galilean moons. Current technology
has revealed that its ice-covered surface contains vast amounts of liquid water
underneath, and space explorers have anticipated finding extra-terrestrial life there. Both
NASA and ESA recently announced that they are to send probes to Europa.
Unfortunately, radiation levels around Europa would wreak havoc with the probes'
instruments, preventing an actual landing. Instead, the probes will conduct fly-by
missions while orbiting Jupiter, and they are bound to make some tantalizing discoveries.

KEY (2) & (3):


1, green cities
2, exciting
3, construction
4, ecological
5, proposed
6, modern-day
7, aim
Colloquialisms
MARK TWAIN
Environmental

COMMITMENT

Dear Ken
Today, I would like to continue to talk about last week's topic. I know you were
questioning about the incident. How to solve environmental problems effectively and
concisely? I intend to have an intention to pthat situation with the helpful suggestions of a
few of my experienced friends. Today's topic I just wanted to let you know. Thank you
for taking the time to read this letter.
See you again,
Quang.
Grammarist
There was a time when, as an educated person, you would have been expected to discuss any
(0) INTELLECTUAL or cultural topic. You would have read the latest novel, been familiar
with the work of poets, and been wholly (16) ......... with the current state of art and music. You
would have felt equally relaxed discussing philosophical ideas in some (17) ........., should the
subject of the discussion have changed.
This would have meant the (18) ......... of issues related to the results of scientific research.
However, as significant discoveries accumulated, it became (19) ......... difficult for any one
person to keep abreast of developments across the entire field. A point was reached where the
pace of progress was so great that a single brain became completely (20) ......... to absorb the
wealth of information. Scientists could no longer (21) ......... switch back and forth between
disciplines as before. They became specialists.
A broadly-educated person today can still have a general (22) ......... of most of the specialisms,
but not in the (23) ......... detail in which research workers are themselves immersed. Trapped
inside their own special areas, the pitfall for most research scientists is an (24) ......... to
communicate with those working on areas bordering their own, let alone totally (25) .........
areas.

Cibele – A Serious Game About Internet Love and Sex


Cibele is a short story game that investigates the mystery of First Love. In Cibele we play
as a girl named Nina who meets a person online and develops a relationship with them.
The story of this game is the progression of that relationship.
• Read the following passage and mark the letter:

In the Democratic Republic of Vietnam, today is the first day of school. I envisioned the ecstatic scene
of the first day of school all around me. The children were all overjoyed because after a few months
away from school and many unexpected alterations, they were able to reunite with their teachers and
friends. But, much better, the children will begin to receive a completely Vietnamese education from
this point forward. They have such fortunate because of the sacrifices of so many of their compatriots.
So, what do you think of it? During the next school year, please be diligent in your efforts to learn, be
obedient, listen to your teacher, and love your classmates.
In an attempt to continue up with our country, we need to restore the legacy that our ancestors left us
after 80 years of slavery. The country has high expectations for their young generation in that
construction. For example, whether or not Vietnam's rivers will be picturesque and if its people can
compete with the world's foremost powers is attributable in significant part to the children's learning
and development. In honor of your first day back to school, I can only wish you a year filled with
happiness and a successful school year.

Where will you be able to purchase a ticket to your adolescence – a slow-motion film of the
most glorious years of your life? We were carefree at the time, full of enthusiasm and passion.
When we realized it, we began to collect pieces of ourselves together. Those are the moments
that we savor and squander, fall, and rise from. What is the value of youth? Youth is
irreplaceable! “Youth is like a shower,” he says. There's nothing wrong with showering in the
rain, even if it's gotten you upset.
Health, time, and money are the most valuable time in life; money is the only thing lacking,
whereas time and health are always abundant. At that age, there was excitement and a touch of
impatience, extravagance, we ventured to live and dream, we were not frightened, and we
were not exhausted of our aspirations. Youth would also be a finite phenomenon since it is both
profound and transitory the same as a "shower". Nonetheless, the rain-interrupted abruptly after
a torrential downpour with large droplets and a thick blanket of rain.
• You are going to read four reviews of a book about how architecture can affect the emotions.
For questions (1 – 4), choose from the reviews (A – D). The reviews may be chosen more than
once.

The Architecture of Happiness


(Four reviewers comment on philosopher Alain De Botton’s book)

A.
Alain de Botton is a brave and highly intelligent writer who writes about complex subjects, clarifying the arcane for
the layman. Now, with typical self-assurance, he has turned to the subject of architecture. The essential theme of his
book is how architecture influences mood and behavior. It is not about the specific architectural characteristics of
space and design, but much more about the emotions that architecture inspires in the users of buildings.
Nonetheless, architects do not normally talk nowadays very much about emotion and beauty. They talk about
design and function. De Botton's message, then, is fairly simple but worthwhile precisely because it is simple,
readable, and timely. His commendable aim is to encourage architects, and society more generally, to pay more
attention to the psychological consequences of design in architecture: architecture should be treated as something
that affects all our lives, our happiness, and well-being.

B.
Alain de Botton raises important, previously unasked, questions concerning the quest for beauty in architecture, or
its rejection or denial. Nevertheless, one is left with the feeling that he needed the help and support of earlier
authors on the subject to walk him across the daunting threshold of architecture itself. And he is given to making
extraordinary claims: ‘Architecture is perplexing ... in how inconsistent is its capacity to generate the happiness on
which its claim to our attention is founded.’ If architecture's capacity to generate happiness is inconsistent, this
might be because happiness has rarely been something architects think about. De Botton never once discusses the
importance of such dull, yet determining, matters as finance or planning laws, much fewer inventions such as the lift
or reinforced concrete. He appears to believe that architects are still masters of their art when increasingly they are
cogs in a global machine for building in which beauty, and how de Botton feels about it, are increasingly beside the
point.

C.
In The Architecture of Happiness, Alain de Botton has a great time making bold and amusing judgments about
architecture, with lavish and imaginative references, but anyone in search of privileged insights into the substance
of building design should be warned that he is not looking at drain schedules or pipe runs. He worries away, as many
architects do, at how inert material things can convey meaning and alter consciousness. Even although he is a
rigorous thinker, most of de Botton’s revelations, such as the contradictions in Le Corbusier's theory and practice,
are not particularly new. However, this is an engaging and intelligent book on architecture and something everyone,
professionals within the field, in particular, should read.

D.
Do we want our buildings merely to shelter us, or do we also want them to speak to us? Can the right sort of
architecture even improve our character? Music mirrors the dynamics of our emotional lives. Mightn’t architecture
work the same way? De Botton thinks so, and in The Architecture of Happiness, he makes the most of this theme on
his jolly trip through the world of architecture. De Botton certainly writes with conviction and, while focusing on
happiness can be a lovely way to make sense of architectural beauty, it probably won’t be of much help in resolving
conflicts of taste.

Page 1
• Read the following passage and mark the letter (A, B, C, or D) on your answer sheet
to indicate the correct word that best fits each of the numbered blanks.

SOAP OPERAS
It is surely beyond (0) _______ that soap opera is the most consistently popular type of
television program in the world. It has succeeded in (1) _______ the imagination of
millions since it first (2) _______ as a genre back in the 1930s. The word ‛soap' alludes
to the role originally played by detergent manufacturers, who promoted their
merchandise during commercial breaks. Soap operas have been (3) ________ as
mindless entertainment, with viewers only (4) _______ to these programs with the
intention of escape from reality.
Soaps are often set in friendly, tightly-knit neighborhoods, evoking nostalgic feelings in
some viewers, since such communities may no longer exist in many areas. The subject
matter of soaps also (5) _________ great appeal for viewers since the stories (6)
________ focus on domestic problems they may have experienced themselves.
There have been a significant shift in attitudes with many soaps now (7) _______ moral
and social issues. The characters and situations (8) _______ are complex and ambiguous,
providing much food for thought and no easy answers.
1, A. dispute B. argument C. dissent D. challenge

2, A. increasing B. progressing C. progressive D. progression

3, A. remarkable B. extraordinary C. noteworthy D. impressive

4, A. operating B. operational C. operative D. postoperative

5, A. mutilating B. disfigured C. deformed D. disfiguring

6, A. replacements B. alternatives C. surrogates D. substitutions

7, A. politicians B. legislators C. lawmakers D. policymakers

8, A. satisfactorily B. abundantly C. adequately D. sufficiently


WORDFORM
One woman in five is a shopaholic
Retail therapy has become one of Britain's most (0) _________ leisure PLEASURE
(1) ............... . But the percentage of the population suffering from the serious PURSUE
(2) .................. condition of shopping addiction is reaching crisis point, MEDICINE
(3) ................ the number of drug and drink addicts in the UK combined. TAKE
Experts believe ten percent of the population, and possibly twenty percent of
women, are manic, compulsive shoppers. Most shopaholics are (4) ............... HEAVY
in debt, and the condition has led to family break-ups, depression,
(5) ................ and even suicide. HOME
Known as oniomania, the condition has been known to psychiatrists since the
early 1900s but only now is it reaching epidemic proportions. Jim Goudie, a
(6) ............. psychologist at Northumbria University, said, 'One of the reasons
CONSUME
behind this sudden rise is that shopping has never been so (7) .............. .
ATTRACT
Shopping centers are beautiful these days, absolute wonderlands. Store cards
are proposed at the till and people can obtain (8) ............. of credit cards with
FIST
relative ease.'
Goudie believes shopping addiction masks deeper problems. 'Mostly there is
underlying depression and (9) ….............. . Often it can be a disturbing
relationship with one's parents. Cold and (10) …............. parents often lavish ANXIOUS
EMOTION
presents on children and they then come to associate that with pleasure.'

KEY:
1, pursuits 2, medical 3, overtaking 4, heavily 5, homelessness
6, consumer 7, attractive 8, fistfuls 9, anxiefy 10, unemotional
• Use the word given in capitals at the end of some of the lines to form a word
that fits in the space in the same line.
Wind turbines take the energy of the wind and convert it to (1. ELECTRIC) …............. power.
The idea of (2. HARNESS) …............. the wind's power has existed for thousands of years in the
form of windmills and water pumps, but modern engineers have extended this concept to a wide
range of (3. APPLY) …............., from small devices that power traffic lights, to multi-acre wind
farms that power entire cities.

The largest wind turbines in use today rise to about 400 feet in height; they require huge towers
with deep (4. FOUND) ….............., are expensive to build, and are vulnerable to weather-based
(5. INTERRUPT) .…............. . For at least a century, engineers have dreamed of pulling electricity
from high in the atmosphere, where the winds provide as much as four times the power of ground-
level flows. Nonetheless, only recently have (6. LIGHT) .…................. materials and computer
guidance systems emerged that make the idea feasible.

While no single design has emerged as a (7. FRONT) .…............., the most efficient airborne
turbines swoop through the air like a kite at 800-2,000-foot altitudes. Known as crosswind models,
these models are equipped with rotors that act as both turbines and propellers and are tethered to the
ground using a long cable. Direct-drive generators send electricity down the tether to a ground
station. Given the higher wind speeds at altitude, crosswind turbines can deliver twice the energy
per unit of capacity than (8. CONVENE) .…............. turbines, and with no tower, they use 90%
less material, reducing capital costs.

For all their advantages, however, the questions surrounding airborne wind power generation are
(9. SIGN) …................. . How do you safely suspend airborne turbines hundreds or thousands of
feet off the ground? How do you avoid interference with aviation? How do you keep them aloft for
long periods without having to perform maintenance? Enthusiasts for the technology believe all
these issues are not only surmountable, but they are also worth overcoming because when it comes
to wind's potential for providing consistent, (10. EXPEND) ……..............., and renewable energy,
the sky is completely the limit.

+ KEY:
1, electrical 2, harnessing 3, applications 4, foundations 5, interruptions
6, lightweight 7, frontrunner 8, conventional 9, significant 10, inexpensive
• Use the word given in capitals at the end of some of the lines to form a word
that fits in the space in the same line.
Astronomer Sir Edmund Halley (1656-1742) is best known for the comet that bears his name. Yet
one of his greatest (1. ACCOMPLISH) …............., in the eyes of his contemporaries, was to
chart, using (2. CALCULATE) …............. made on his sea voyages on the warship Paramore, the
'alterations of the compass'. These (3. VARY) ….............. are now known as 'declination', that is,
the angle between magnetic north and true (4. GEOGRAPHY) …............... north. Without it,
sailors were unable to correct their compasses. It was therefore impossible to deduce longitude
precisely and navigate the oceans.

So it was that Halley, one of only two men in the land at that time paid to conduct scientific
research, set sail for the Cape Verde Islands with the grand plan of charting declination in the North
and South Atlantic. The trip was (5. QUICKEN) …….............. aborted because of crew (6.
SUBORDINATE) …................., but Halley returned to the seas a second time.

This voyage took him and his crew to Rio de Janeiro, down past South Georgia, up again to
Newfoundland, and back to England. From these travels Halley published, in 1701, a 'New and
Correct Chart Shewing the Variations of the Compass in the Western and Southern Oceans'. More
(7. SOPHIST) …................. successors to this primitive cartographic effort proved (8.
DISPENSE) …................. to seamen for more than a century before a slow change in the
terrestrial magnetic field rendered them (9. ACCURACY) …................ .

£70,000 will have to be raised before he (10. BARK) ….............., and Sir Vivian Fuchs, who led
the first cross-Antarctica expedition, is providing support for his efforts to do this. Dr. Clark
became fascinated by Halley during a two-year posting to Halley Station in Antarctica, where he
read biographies of the great scientist. 'Halley led a (11. MARK) ….............. life: Dr. Clark says.
'He was not only a respected scientist but also led expeditions. He was not just an astronomer but
also researched geophysics. While he was Astronomer Royal, he mapped the (12. JUDGE)
…............... of the stars, and also found time for other interests.'

KEY:
1, accomplishments 2, calculations 3, variations 4, geographical
5, quickly 6, insubordination 7, sophisticated 8, indispensable
9, inaccurate 10, embarks 11, remarkable 12, judgments
• Use the word given in capitals at the end of some of the lines to form a word that fits
in the space in the same line.
In 2008, renegade Greenpeace (1. ACT) ….… Junichi Soto and Toru Suzuki seized a container
of whale meat from a delivery depot in Aomori Prefecture, Japan. They believed that the meat was
procured from government (2. WHALE) …... research and was destined for sale in Japanese
restaurants, a practice publicly condemned by the Japanese government. The confiscation sparked
controversy for the government, which hastened a cover-up, (3. CHARACTER) …… the meat as
'a a souvenir' of the whalers. However, it was the fate of Soto and Suzuki which seemed
particularly unjust.
Although the police launched an (4. INVESTIGATE) …… into the whalers' actions, no charges
were brought against them. Instead, they charged the activists and even (5. TREAT) …... them
during their detention. The pair were denied legal (6. REPRESENT) …… and interrogated for a
relentless 12 hours. The activists were even stigmatized as terrorists and egregiously compared to a
doomsday cult that released poisonous gas in a Tokyo underground station. The pair didn't expect
to be celebrated for uncovering the government scandal, but they were nonetheless surprised by the
outcome of their actions. Ultimately the men were sentenced to one year in prison, which the judge
(7. MERCY) ….... suspended for three years. Unfazed, Sato expressed no regrets. 'Because of our
case, many inside Japan are now aware of the whaling industry's kickbacks and (8. EMBEZZLE)
….....' he remarked in an article he wrote for a British newspaper.
Despite (9. SIGN) …...... reducing their presence in the Antarctic Ocean, Greenpeace's efforts to
protect these (10. MAJESTY) …..... marine mammals date back decades. The organization, which
began in 1971, (11. BARK) …..... in the mid-1980s on a mission to establish Antarctica as a world
park. Although (12. EVENT) ......... successful, when Japan's whaling fleet appeared in the late
1980s under the guise of 'scientific research, it was clear a loophole had been exploited.
In an attempt to bypass regulations, the government stipulated that the (13. EXPLORE) ….....
could only be subsidized by sales of whale meat. But few had the wool pulled over their eyes and
fleets of Greenpeace vessels showed up to confront the Japanese whaling ships. (14.
DRAMA) …....... scenes of ships a tenth the size of the government's fleet were broadcast on the
news. It was not until several years later that the Japanese government finally put an end to its
"research", partly because of the Tokyo Two (15. REVEAL) ……... and partly because of the (16.
INVOLVE) …...... of Greenpeace.

KEY:
1, activists 2, whaling 3, characterizing 4, investigation
5, mistreated 6, representation 7, mercifully 8, embezzlement
9, significantly 10, majestic 11, embarked 12, eventually
13, exploration 14, dramatic 15, revelations 16, involvement
I. SENTENCE: (WORDFORM)
1, The temptation to regard the 30-year-old as an indispensable thoroughbred can establish
irresistible to Rangers supporters and neutral connoisseurs. (DISPENSE)
2, He felt her firmness and obeyed the impulsion at once, just as the other two men had, a
moment before, obeyed unquestioningly the wave of her hand. (QUESTION)
3, The danger of such rapid growth could breed hubris or a sense of overconfidence, that could
expose the economy to risks, said panelists at a forum in Dubai. (CONFIDENT)
4, Our main problem has been our inconsistency which can let us down, but that means
sometimes we end up doing ourselves justice. (CONSIST)
5, But this is one issue on which I think incoherence and vacillation, combined with a liberal
dose of pious platitudes, are accurately what most people want. (COHERE)
6, The sympathetic nervous system, not white blood cells, is critically essential in the regulation
of energy expenditure and thermogenesis. (EXPEND)
7, By taking a hard look at individual project components, project managers can zero in on more
cost-effective alternatives. (EFFECT)
8, Participation in the Soweto Kinch Live Box Anniversary Tour, of which his Artsfest
appearance is a part, continues a creative association with like-minded experimentalists.
(EXPERIMENT)
9, The content will encompass all aspects of chemistry, from biochemistry to organic and
inorganic chemistry to physical chemistry, including interdisciplinary topics. (DISCIPLINE)
10, Repairing wrecked companies is a tricky task, whether they are in modern businesses like
telecoms, or genuinely unfashionable metal-bashing. (FASHION)
11, In this intervention, the computer or therapist asks the client to brainstorm a list of problems,
and then rank them in terms of intractability and irksomeness. (TRACTABLE)
12, It was a lesson he preached indefatigably, and with unshakable confidence, to the press,
Cabinet Ministers, anthropology practitioners, and community groups. (SHAKE)
13, Consequently, there is no indisputable diagnostic evidence in the photograph to support
Meert's claim that the unconsolidated material is a well-developed paleosol. (CONSOLIDATE)
14, Following disestablishment in 1920, the Church in Wales initially fared better than the non-
conformist churches, which suffered a decline during the late 20th century. (ESTABLISH)
15, Traditionally, screening for syphilis infection is a two-step process that involves an initial
nontreponemal test followed by a confirmatory treponemal test. (CONFIRM)
16, They all appeared to him very young, indistinguishably alike, and he knew that they were all
brothers, and also immortal. (DISTINGUISH)
17, Furthermore, the government undertook to cover smoking-related illnesses under Medicare
at a time when Washington was well aware of the healthfulness of tobacco.
(HEALTH)
18, Terrorists and insurgents must lay down their weapons, and enable the vitally important
reconstruction and humanitarian work to press ahead. (HUMAN)
19, Nothing else in creating possessions of what mankind has, namely religion and economic
complexity, the devil lies exasperatingly in the details of our causative factors that combine
them. (EXASPERATE)
20, The only conclusion you can draw from this is that foreknowledge only extended to the
timing and targets of the attacks, but not to the specific planes that would be used. (KNOW)
21, Even sections on well-documented attractions, such as the booming wine industry, include
lesser-known or uncommercialized options in addition to well-known sites. (COMMERCE)
22, The damage all chemical stupefied have done to innocent bystanders such as parents and
friends is immeasurable and so extremely costly to society as a whole. (MEASURE)
23, Once self-sabotaging thoughts and behaviors have been resolved, the individual can begin to
use hypnotic suggestibility to improve the body's functioning. (SUGGEST)
24, The application of estimates of test performance from one population to another depends on
the relation between the range of abnormality in the two groups. (NORMAL)
25, The aforementioned, whether inadvertent or intentional, augments the book and contributes
to Sotos's successful rendering of destructive compulsiveness as the subject in Comfort and
Critique. (COMPEL)
26, Mary has a track record of speaking out of turn or (not) undiplomatically, though she often
complains that opponents misquote her or deliberately take her comments out of context.
(DIPLOMAT)
27, Studies have demonstrated that, contrary to arguments of soil infertility, this primitive
cultivation practice ensures that callousness in the soil is not compromised. (FERTILE)
28, In the respiratory field in our hospital, patients with chronic obstructive pulmonary disease
or hyperventilation syndrome are misdiagnosed as having asthma almost daily.
(VENTILATE)
29, Similarly, the discarnate being may find certain memories in the subconsciousness of the
medium that will recall certain facts connected with his past earth life. (CONSCIOUS)
30, In the absence of such a comprehension, spirituality will always be construed as
extracurricular or a complement to education at best, and a delusory distraction at worst.
(CURRICULA)
31, A non-metric multidimensional scaling ordination identified two gradients of species
replacement distinguished by differences in forest canopy species and groundcover.
(DIMENSION)
32, The suppression and mistreatment of Black people are maintained despite its malevolence
and its moral dubiousness because it serves the needs of this system. (TREAT)
• Read the following passage and mark the letter (A, B, C, or D) on your answer sheet
to indicate the correct word that best fits each of the numbered blanks.
The difficulty of defining or measuring intelligence in non-human animals makes the subject
difficult to study scientifically birds. In general, birds have comparatively extensive brains
compared to their head size. Bird intelligence has been studied through several attributes and
abilities. Many of these studies have been on birds such as quail, domestic fowl, and pigeons
kept under captive conditions. It has, nevertheless, been noted that field studies have been
limited, unlike those of the apes.
In the 1970s, on the Li River, Pamela Egremont observed fishermen who allowed the birds to
eat every eighth fish they caught. Writing in the Biological Journal of the Linnean Society, she
reported that, once their quota of seven fish was filled, the birds "stubbornly refuse to move
again until their neck ring is loosened. They ignore an order to dive and even resist a rough push
or a knock, sitting glum and motionless on their perches." Visual or auditory signals and their
association with food and other rewards have been well studied, and birds have been trained to
recognize and distinguish complex shapes. This may be an essential ability that aids their
survival.
1, What makes scientists assume that birds are quite intelligent?
• Use the word given in capitals at the end of some of the lines to form a word that fits
in the space in the same line.
These problems are (1. KNOW) ___________, but the tragedy for science is that too few people are
willing to break away from the present system. However, as the eminent (2. BIOLOGY)
___________ Ron Vale argued recently — (3. FIT) __________, in a preprint — preprints may be a
way out of the impasse because they don’t entail a major shift away from the norm.
That may seem an odd claim in view of the fact that preprint archives have been in existence for twenty
years, yet preprints have not been adopted (4. UNIVERSE) __________ . This slove uptake is not
only a (5. REFLECT) ___________ of the inherent (6. CONSERVE) ___________ of scientists
but also a result of the widespread (7. CONCEPT) ____________ that journals won’t accept
manuscripts that have been posted online as preprints. There is also a fear that (8. PUBLISH)
__________ of papers without peer review risks opening the floodgates to ‘junk science’ — something
which, so far at least, has yet to occur. Preprints may not be peer-reviewed, but authors know full well
that preprints are immediately opened up for critique and discussion by a worldwide community of
reviewers.

KEY:
1, well-known 2, biologist 3, fittingly 4, universally
5, reflection 6, conservatism 7, misconception 8, publication
These gray-skinned sea mammals are often misidentified as elephants. Sea cows, dissimilar to terrestrial
animals like cows and rabbits, seem to have little hair. In addition to its smooth skin, the sea cow has
several hairs scattered over its surface. Each of them has a spherical skull, a small eye, and a
largemouth. Their subtleties are modeled after paddles, and they don't have hooves on their feet either.
A mature sea cow measures between 2.7 m and 4 m in length and weighs between 250 kg and 300 kg.
The recently conceived sea cow is around a meter long and weighs between 20 and 35 kilograms,
depending on each species.
Sea cows swim by moving their wide tails up and down. Two appendages on the front of its body
enable it to swim differently. Gradually, they swim at a speed of approximately 10km/h, which is
equivalent to cycling. Sea cows, while swimming unhurriedly, may cover huge distances - up to 600
kilometers in only several other days. Sea cows communicate with each other by subtle cries, loud
noises, or canine-like sounds. Sea cows are herbivorous species, according to scientists. Seagrass is a
favorite meal of sea cows, and they eat it in enormous quantities. To maintain their energy levels, they
should indeed eat a staggering amount of seagrass (roughly 28-40 kg of seagrass) each day. The sea
cow's substantial skeleton helps it descend to the bottom when it's searching for and eating seagrass. To
consume seagrass, they typically use two teeth of equivalent length. Additionally, sea cows have
reached an advanced age. As long as the circumstances are adequate, they can last up to 70 years or so.
Sea cows are usually solitary, notwithstanding they can frequently be seen in groups.
Alongside the Western Pacific to the Indian Ocean, manatees may be discovered in 37 nations and
territories. There is a considerable decline in sea cow populations even in areas where there was a
significant concentration of them. Sea cows have vanished in areas including Taiwan as well as the
Maldives... Categorized in the International Red Book as a species at extreme danger of extinction, sea
cows are listed as endangered. As a result of overhunting and fishing, sea cow populations have
plummeted across the world. As a consequence of marine pollution, the essential food maintains by sea
cow species, seagrass is depleted. This is one of the factors contributing to the sea cow's brink of
extinction.
Toothed whales or including dolphins have acquired a remarkable sensory skill known as echolocation,
which is utilized for detecting food and navigating underwater. Toothed whales make noises by moving
air between air passages or sinuses in their heads. Sounds are reflected or echoed back from objects, and
it is believed that these are received through an oil-filled channel in the lower jaw and transported to the
animal's middle ear.
When swimming ordinarily, low-frequency sounds are generated; the echoes from these sounds provide
information about the seafloor, shorelines, underwater obstructions, water depth, and the presence of
other underwater creatures. According to a recent theory, hunters may utilize very high-intensity
focused noises to stun or disorient animals. Whale songs are composed of distinct sequences of groans,
moans, roars, sighs, and high-pitched squeals which may last up to ten minutes or more. It is
hypothesized that these sounds might be utilized for communication reasons such as identifying other
people, long-distance interaction, warning others of hazards, and navigation. For this reason, baleen
whales lack vocal cords, experts are uncertain how whale songs are produced.
It is thought that toothed whales and dolphins have a multi-dimensional perspective of the environment
because of echolocation. Several toothed whales are known to communicate with each other using
whistles, clicks, groans, and other sounds. Whale vocalizations are the sounds that whales use to
communicate with each other and with one another. Singing is the term used to describe certain species
of whales that produce a sequence of predictable and regular sounds that are similar to human singing.
Consequently, the sound is created by humans exhaling air through their larynx. The larynx's vocal
cords open and release to divide the air stream into distinct compartments. The throat, tongue, and lips
shape these pockets into the desired sound.
Whales, dolphins and sound
An incredibly tough and intricate procedure, reaching a judgment often requires jurors to reconcile a
strong sense of justice with a comprehensive understanding of the law.
Consider the instance of Stephen Owen, whose twelve-year-old son Darren was killed when a vehicle
knocked him off his bike and crushed him. The lorry driver, Mr. Taylor, left the scene without reporting
the collision. Whenever the police caught him, they discovered he had never held a driver's license. Mr.
Taylor exhibited no remorse for his actions, which saddened the family immensely. For dangerous
driving, he was forbidden from driving and sentenced to eighteen months in prison, that being said he
only spent twelve months.
Stephen Owen couldn't get over his son's death. When Taylor was freed from imprisonment, he was
astonished at the rapidity with which he was released. It became apparent that Taylor had continued to
drive following his release from prison, he wrote to the Queen to demand that the prohibition be
enforced. Owen allowed the incident to completely consume his life, rendering him incapable of leading
a regular life. Taylor was tracked down to his house in Kent, where he was accosted on the street. He
shot Taylor twice at point-blank range with a sawn-off shotgun, injuring him in the back and his
common-law wife, Alison Barratt, in the arm. They made it out alive, but Owen was arrested and
charged with attempted murder.
A prosecutor maintained that Owen's shooting of Taylor was premeditated, while Owen claimed to have
discharged the gun in a moment of near insanity during his trial. "Any parent who receives a phone call
only to learn of a child's death must feel sympathy, compassion, and indulgence for that parent," the
judge told the jurors. As for Owen, he cautioned that the jury needed to concentrate on whether he had
intended to kill at the moment of the shooting, regardless of what transpired previously.
A crime of passion
Americans have long been bombarded with the sleek, tantalizing images that come courtesy of the
advertising industry. The Internet has joined the ranks of glossy magazines, newspapers, billboards, and
other purveyors of print to present women and men with physical appearances ranging from improbable
to impossible. Certain software programs enable marketers to digitally alter the images of models, often
making people - and women in particular - appear skinny beyond belief. When these advertisements are
churned out on a massive scale, the effects are pervasive and toxic. After all, it's easy to come away
with the message that the models featured on the covers of fashion magazines epitomize physical
perfection. Nonetheless, if the ideal look is more or less unattainable in reality, it might have grave
implications.
Eating disorders are estimated to affect at least thirty million people in the United States, according to
the National Institute of Health. Bulimia and anorexia nervosa is the most prevalent eating disorders. A
strong dread of becoming overweight and a distorted body image characterize the two illnesses. Experts
generally agree that serious depression and anxiety appear to be, at least in part, a matter of biology:
patients are more likely to be genetically predisposed to these disorders than the general population. In
addition, persons with anorexia or bulimia tend to think negatively and respond to stress something
beyond usual.
How might the existence of digitally changed models be responsible for the emergence of these two
eating disorders if they are biologically based? While it would be unreasonable to blame the advertising
industry's methods entirely, research does reveal a relationship between teenage exposure to ultra-thin
models in the media and the development of body image disorders. According to one research, forty-
seven percent of girls in fifth to twelfth grades expressed a desire to reduce weight as a result of
magazine pictures, and sixty-nine percent of those girls said magazine images influenced their
perception of what a desirable body looks like. Undoubtedly, some of these girls may be predisposed to
significant depression and anxiety due to genetics. Some people may also have negative thought habits
and a strong reactivity to stress. Nevertheless, evidence suggests that doctored pictures of unachievable
body types will be a trigger for people who are already susceptible to eating problems.

An Honest Picture of Health


• Use the word given in capitals at the end of some of the lines to form a word that fits
in the space in the same line.

Art History
People who enjoy paintings are sometimes reluctant to analyze them for fear of spoiling the
richness and (1. SPONTANEOUS) …............ of their experience. It has been suggested that some
of the work done by art (2. HISTORY) …............., whose concern is with theory rather than
practice, ignores and indeed denies the aesthetic experience, the (3. FUNDAMENT) ….….........
pleasure of looking, as well as the very special act of artistic (4. CREATE) ……......... . This view
is a bit like the notion that knowing the ingredients of the recipe, recognizing the method of
cooking, and seeing the utensils employed detract from the taste of the dish.
(5. KNOWLEDGE) …................ the importance of enjoying something does not, of course,
preclude a thorough knowledge of the object that is arousing pleasure. It might be more (6.
PLEASE) …............... if we know more about the object we are viewing. Moreover, pleasure is
not a simple matter. The arousal of our senses - and how we recognize and register it - is itself open
to (7. INTERROGATE) …............... . It is also (8. HISTORY) …............... located. Why we like
particular (9. CHARACTER) …............... of certain sorts of objects at any one time is not simply
the result of our genes or our particular personalities but is determined by values promoted within
the society of which we are part. So, while no one seeks to (10. ESTIMATE) ……............. the
importance of sensuous and (11. INSTINCT) ……............ responses to art objects, the notion that
the sensuous is undermined by the (12. INTELLECT) ……............ is a legacy from a period in
the past which promoted art as an alternative to thought.

KEY:
1, spontaneity 2, historians 3, fundamental 4, creativity
5, Acknowledging 6, pleasurable 7, interrogation 8, historically
9, characteristics 10, underestimate 11, instinctive 12, intellectual

But in addition to this force, there are also numerous weaknesses. These are gaps in basic
knowledge due to the tendency to follow "fashionable" topics, especially the limited capacity of
practice and creativity due to the cumbersome and heartfelt vegetarian learning.

The young man, instead of talking to his co-worker about his professional responsibilities
or trying to discuss them with his superiors, did it himself.
• Use the word given in capitals at the end of some of the lines to form a word that
fits in the space in the same line.

CHERNOBYL DISASTER
The Chernobyl catastrophe was a nuclear accident in the Soviet Union on Saturday, 26 April
1986. It is considered the worst nuclear disaster in history both in terms of cost and
(1. CASUAL) …….…, and is one of only two nuclear energy accidents rated at seven — the
maximum (2. SEVERE) …….… — on the International Nuclear Event Scale, the other being
the 2011 Fukushima Daiichi nuclear disaster in Japan. The initial (3. EMERGE) …….…
response, together with later (4. CONTAMINATE) ….….… of the environment, ultimately
involved more than 500,000 (5. PERSON) ….….… and cost an estimated 18 billion Soviet
rubles — roughly US$68 billion in 2019, adjusted for (6. INFLATE) ….….…. .

The accident was initiated during a safety test on an RBMK nuclear (7. REACT) …….… . The
test was a (8. SIMULATE) ……….… of an electrical power outage to help create a safety
procedure for maintaining reactor cooling water (9. CIRCLE) …...…..… until the backup
electrical (10. GENERATE) ……..…… could provide power. Three such tests had been
conducted since 1982, but they had failed to provide a solution. On this fourth attempt, an (11.
EXPECT) ….…..… 10-hour delay meant that an (12. PREPARE) ………… operating shift
was on duty. During the planned decrease of reactor power in (13. PREPARE) …..….… for the
electrical test, the power (14. EXPECT) …….….… dropped to a near-zero level. The operators
were able to only (15. PART) ….……...… restore the specified test power, which put the
reactor in an unstable condition. This prospect was not made evident in the operating
(16. INSTRUCT) .…….…..…, so the operators proceeded with the electrical test. At the
conclusion of the experiments, the operators initiated a shutdown of the reactor, but a
(17. COMBINE) .….....……. of unstable conditions and reactor design flaws resulted in an
(18. CONTROL) .…....……. nuclear chain reaction.

KEY:
1, casualties 2, severity 3, emergency 4, decontamination
5, personnel 6, inflation 7, reactor 8, simulation
9, circulation 10, generators 11, unexpected 12, unprepared
13, preparation 14, unexpectedly 15, partially 16,
instructions
17, combination 18, uncontrolled
• Use the word given in capitals at the end of some of the lines to form a word that fits
in the space in the same line.
INITIATION RITES
In the depths of the Brazilian Amazon jungle, many native tribes still Follow their traditional
ways of life, carrying on ancient practices and (1. RITE) …….… . The indigenous Sateré —
Mawe tribe are among these. They value (2. STRONG) ……...… and courage — qualifies
needed to protect the tribe from hostile neighbors and natural (3. PREDATE) …….… . For this
reason, their (4. WAR) ….….… are highly revered. As a result, young boys in the tribe are
raised with the dream of joining the warrior ranks.
However, this is not an effortless (5. TAKE) …..….… . In one of the tribe's most sacred rituals,
boys hoping to gain this status must show themselves (6. CAN) …..….… of enduring the worst
that the jungle has to offer. They prove their worth by remaining calm, often entirely
(7. EXPRESS) ……….…, while experiencing (8. EXCRUCIATE) …….… pain.
The young men's strength is tested in an initiation against a particularly (9. VENOM) ……….…
species of ant, a single sting from which is capable of causing hours of pain. Nicknamed the
'bullet ant' because its sting is said to be as painful as being shot, the powerful toxin released by
the ant attacks the (10. NERVE) …..….… system, causing (11. CONTROL) .….......…. shaking
even after the waves of pain have ceased, as well as disorientation and (12. HALLUCINATE)
…..… The effects of a single ant sting have been known to last for days; the warrior hopefuls of
the tribe, however, do not sustain just one sting.
In preparation for the (13. INITIATE) ……...… ceremony, bullet ants are harvested from the
jungle and sedated in a solution of herbs. While (14. CONSCIOUS) …..….…, the ants are
woven into a pair of gloves made from leaves, with their stingers facing inward. As the ants
come to, they become (15. INCREASE) …….… agitated and are raring to sting. To be
considered a man of the tribe, boys as young as twelve must then thrust their hands into the
gloves for a full five minutes or longer, being stung the entire time. The initiates sing and dance
with the tribe during the ordeal, but this (16. TRACT) ……..… is their only relief.
When the gloves are removed, the ants' venom continues acting for hours, during which time the
boys are in constant pain. Enduring the pain demonstrates the boys' (17. READY) ………..…
for (18. MAN) …….…., and few cry out as doing so would demonstrate (19. WEAK) ……...…
. Each boy will (20. EVENT) ….…….… wear the gloves twenty times over several months
before the initiation is complete.

KEY:
1, rituals 2, strength 3, predators 4, warriors
5, undertaking 6, capable 7, expressionless 8, excruciating
9, venomous 10, nervous 11, uncontrollable 12,
hallucinations
13, initiation 14, unconscious 15, increasingly 16, distraction
17, readiness 18, manhood 19, weakness 20, eventually
• Use the word given in capitals at the end of some of the lines to form a word that fits
in the space in the same line.
Spring in Japan is distinctive for a particular season: it is the moment of ohanami (the (1.
OBSERVE) ……..… of flowers) where almost all of Japan celebrates the flowering of cherry
trees. The cherry blossom (sakura) has become the symbol and national flower of Japan, depicted in
(2. PAINT) ……...… and other art forms for over ten centuries.
As a consequence of the large North-South extension of Japan, trees will blossom on one of the
(3. SOUTH) ……….… islands as early as January, reaching Tokyo in April and Hokkaido in the
North in May. Kyoto, with its ancient shrines and temples, is the main tourist attraction at this time
of year. Whoever has the (4. INCLINE) ….….… and time could follow the blossom route,
northwards, enjoying each different blossoming one after the other. The cherry-blossom
connoisseur, as opposed to the mass, of the public, however, (5. APPRECIATE) …..….…
though they may-. be of the beauty of. the flowering season keeps a wary eye on the weather during
this period to be sure to catch the buds at their best when they are just over half-open and before,
they reach their full-blown (6. MATURE) …..….… .
It's a matter of timing: you want to catch the blossom at its best since it only lasts for a few days
before the spring breeze scatters the petals like white or pink snow. The Japanese flowering cherry,
as the tree is known outside Japan, is native to the country and, somewhat (7. SURPRISE)
……….., none of its 400 different (8. VARY) ……...… are fruit-bearing. The Japanese,
nevertheless, do make use of the tree in other ways, as well as primarily appreciating it for its
delicate beauty. The double petalled blossoms that bloom after the single-petalled ones are
preserved in salt and later used to make a (9. FRESH) ………... drink, with the (10. ADD) …..
……… of hot water. The blossom opens again in the water and this is why Sakura-tea is looked
upon as a special drink to be served on (11. FELICITY) .…........…. occasions such as weddings.
The wood of the tree is also highly respected: it is very troublesome and glows with a rich, dark red
color. You will find cherry wood used in smaller furniture items, and other ornaments such as
platters, checkerboards, inlaid jewelry boxes, and so forth.
In many ways, the cherry blossom has come to (12. SYMBOL) ……….…..…. the
Japanese’character: traditionally, the life of the (13. WAR) .….…..….…, the samurai was
compared to the fleeting life of the cherry blossom, since the samurai was always ready to sacrifice
his life for that of his master. Furthermore, the cherry blossom appeals (14.
ESSENCE) …………..… to the delicate aesthetic taste of the Japanese people and has come to
represent emotions such as joy, (15. OPTIMIST) …..….….…, (16. YOUTH) ……..…..…, and
cheerfulness in the face of difficulty.

KEY:
1, observation 2, paintings 3, southernmost 4, inclination
5, appreciative 6, maturity 7, surprisingly 8, varieties
9, refreshing 10, addition 11, felicitous 12, symbolize
13, warrior 14, essentially 15, optimism 16, youthfulness
• Use the word given in capitals at the end of some of the lines to form a word that fits
in the space in the same line.
Only approximately 65 percent of the world's fish stocks were within (1. BIOLOGY) ……...…
viable levels in 2017, according to a 2020 study by the United Nations' Food and Agriculture
Organization (FAO). Ninety percent of the population conducted so in 1990. Despite the rapid (2.
REDUCE) ……....… in harvest (3. SUSTAIN) ……...… in less than 30 years, scientists predict
that worldwide fish output will increase in the future years, (4. RISE) ……...… from 179 million
metric tons in 2018 to 204 million metric tons in 2030.
One reason why demands were made for the (5. PROTECT) ……...… by 2030 of at least 30% of
the world's seas in marine protected areas (MPAs) is that fish populations must be preserved.
Currently, just 2.7 percent of the world's oceans are (6. COMPLETE) ……...… protected, while
certain countries have already committed their territories to conserve huge areas of the sea. For
instance, 78% of its water is protected in the small island of Palau, whereas 23% in the United
States. Nevertheless, most other nations are far from achieving this (7. OBJECT) ……....… .
Some research has demonstrated that MPAs — which restrict all fishing activity in its harshest (8.
IMPLEMENT) ………...… — assist marine creatures to rebound within their borders. These
populations, by contrast, can spread through the "(9. SPILL) ……...… effect" into neighboring
areas which are (10. ACCESS) …...…...… for fishing. But other studies show that fisheries
(11. MANAGE) …..…...… can be as (12. SUCCEED) …........… in helping to conserve critical
fish reserves instead of prohibiting (or severely restricting) all fishing.
But most researches have the (13. ADVANCE) …….....… of being carried out over a short period
and depending (14. PRIMARY) ……….… on modeling rather than actual data, McClanahan
makes the (15. ARGUE) ……...…. .

KEY:
1, biologically 2, reduction 3, sustainability 4, rising
5, protection 6, completely 7, objective 8,
implementation
9, spillover 10, accessible 11, management 12, successful
13, disadvantage 14, primarily 15, argument
• Use the word given in capitals at the end of some of the lines to form a word that fits
in the space in the same line.

A. Georgina
These days, there’s a self-help book for every problem that could probably plague our lives. By
reading a self-help book, we can acquire the skills needed to cure depression, become that
confident team leader at work or snare that special someone. I would say that some of these
books do merit reading and there are some that I would even recommend. But a self-help book
should only ever serve as a supplementary aid and never as a stand-alone cure. This is especially
true if you have a recognized medical disorder, like depression, or another serious personal
problem. In cases like that, you can utilize the book as a valuable resource, but make sure you’re
also seeing a psychologist who has got the capability of helping you.

B. John
Go to the self-help section of any major bookshop and you’ll be inundated with choice. It’s
virtually impossible to choose which one is the best, but they all claim to be on their covers! In
my personal experience, I have found the information I’ve read in these books fascinating and
they helped me - um - help myself! But let’s face it - not all books are created equally. If you’re
in the market for one of these books, I suggest perusing online reviews before buying. And
while it might take a bit of determination to put the ideas into practice successfully, it’s arguably
much more affordable than consulting a professional.

C. Meredith
The majority of self-help books out there work on the assumption that people can do what a
professional can do, especially when it comes to personal problems. We’ve all had our fair share
of personal drama and, in my experience, the last thing I wanted to do was read about how to
solve it. I’m sure the writers of these books are not being dishonest - they have the intention of
actually helping people. However, my advice to anyone who’s undergoing a personal crisis is to
consult a therapist or counselor and leave the book on the shelf.
D. Sam
When it comes to searching for a way to teach yourself how to do something, whether it be how
to be more productive in life, reduce feelings of intimidation or stop being so hard on yourself,
there is no one-size-fits-all book. Everyone has their way of approaching obstacles and what
works for one person - or even the majority of people - doesn’t necessarily work for another. I
believe some people are rather responsive to self-help books and gain a great deal of valuable
knowledge that they’re able to apply to their lives. For others, though, they have to seek help
elsewhere, either from another book or another source entirely.
-
KEY:
1, activists 2, whaling 3, characterizing 4, investigation
5, mistreated 6, representation 7, mercifully 8, embezzlement
9, significantly 10, majestic 11, embarked 12, eventually
13, exploration 14, dramatic 15, revelations 16, involvement

BLOGGING YOUR WAY BACK TO HEALTH


Anyone who suffers compound fractures of both legs (1) ……..… to indulge in a moment of
self-pity. However, Heather McLean is not one to feel sorry for herself. Instead, the 16-year-old
schoolgirl, who in April last year survived a horrific car accident, started a blog (called Breaking
News), sharing all the successes and (2) ……..… on her slow and often painful road to recovery.
Initially, she had only a few followers, mainly family members, and friends, but after appearing
on a local radio phone-in where she mentioned her blog, her fame grew rapidly and she soon had
thousands of subscribers.
But given that there are well over one hundred million blogs worldwide on the two main
blogging (3) ……..…, it must take more than a plug on local radio to make a blog successful. So
what was Heather's secret? 'Keep it interesting,' says Heather. 'People don't want to read about
what you ate for lunch or what you watched on TV. But there's so much going on around you in
a hospital and so many interesting people with often very dramatic and (4) ……..… stories. You
just have to keep your eyes and ears open.'
Not all of the attention she received was positive or supportive, however. Her blog was a
frequent target for spammers (people leaving comments promoting their commercial websites),
and (5) ……..…. visitors left derogatory and sometimes hurtful comments. 'It's very easy to hide
behind a username,' observes Heather. 'But you have to feel sorry for people like that. They must
have very sad lives if they feel that they have to (6) ……....… in such cowardly behavior.'
By far the biggest draw to Heather's blog - as evidenced by the comments of visitors - was
her humorous and (7) ….......… positive attitude even in her darkest moments. (8) ……..…. this
comments best sums it up: 'Thank you, Heather. You are such a brave girl and an inspiration to
us all.'

1, A. can be expected B. have expected C. might be expected D. expected


2, A. reversals B. setbacks C. hindrances D. hold-ups
3, A. policies B. principles C. podiums D. platforms
4, A. heart-breaking B. soul-stirring C. touching D. converging
5, A. occasionally B. sporadically C. irregularly D. periodically
6, A. luxuriate B. indulge C. coddle D. fulfill
7, A. overwhelmingly B. incredibly C. devastatingly D. exceedingly
8, A. Perchance B. Apparently C. Conceivably D. Perhaps
• Use the word given in capitals at the end of some of the lines to form a word that fits
in the space in the same line.
TIME'S UP ON CURFEW LAWS
Teenagers in the United States love their (1. DEPEND) ……….…, and a looming curfew is a
(2. QUINT) ….…..… experience for American teens. Traditionally, curfews are issued by a
parent or guardian in the interest of safety and peace of mind. This type of curfew, or a family's
decision not to enforce any curfew at all, is personal, and (3. RIGHT) ……….… so.
Nevertheless, another kind of curfew has become more common in cities and towns across the
country. Intending to deter teen crime, mischief, and car accidents, some (4. MUNICIPAL) ….
…….… have turned juvenile curfews from family decisions into public ordinances.
The idea of a legal curfew for minors may have been conceived with good intentions. In
practice, however, these policies are (5. EFFECT) ……..…, unfair, and (6. PRODUCE) ……..
… — not to mention a violation of parental rights, according to the American Civil Liberties
Union. Indeed, curfew laws may be downright (7. CONSTITUTE) ……….… . In the town of
Sumner, Washington, a father allowed his fourteen-year-old son to go to a convenience store
after 11:00 P.M. Sumner had adopted a curfew law that prohibited people under the age of
eighteen from being in public places or businesses past that hour, with limited exceptions. The
father was fined by law (8. FORCE) ……....… . He (9. SEQUENT) ……….… pursued a legal
challenge against the town. The case went all the way to the Washington Supreme Court, where
Sumner's curfew ordinance was struck down. The law was judged to be " (10. CONSTITUTE)
……….… vague." The ACLU, which filed the case on behalf of the father, claims curfew laws
encroach on the rights of parents to decide how their children should be raised.
But isn't it (11. RESPONSE) ….……..… — or even dangerous-not to impose a curfew on
your child? Proponents of curfew laws argue that if parents cannot be trusted to enforce a
curfew, then officials should provide one to ensure teens are home by a (12. REASON)
………...… hour. Adolescents and young adults take more risks than any other age group, and
one (13. INDICATE) ………..… of this is the troubling statistics (14.
ROUND) ….…….… teens and driving. In 2015, only sixty-one percent of high school drivers
reported always wearing a seat belt when riding with a passenger. The risk of a fatal accident is
three times as high for drivers ages sixteen to nineteen as for drivers over the age of twenty.
And dangers only increase at night. No matter the age, the (15. FATAL) ………...… rate jumps
threefold for nighttime drivers in car accidents. For teens (16. SPECIFIC) …….…..…, the
hours between 9:00 P.M. and 6:00 A.M. account for more than forty percent of deaths from car
accidents. This indicates to some that a law keeping teens off the road late at night is a positive
safety measure.
KEY:
1, independence 2, quintessential 3, rightfully 4, municipalities
5, ineffective 6, counterproductive 7, unconstitutional 8, enforcement
9, subsequently 10, unconstitutionally 11, irresponsible 12, reasonable
13, indication 14, surrounding 15, fatality 16, specifically

PATIENT SAFETY
In a well-documented case in November 2004, a Dixon-Woods and her team spend time in hospitals to
female patient called Mary was admitted to a hospital try to understand which systems are in place and how
in Seattle, USA, to receive treatment for a brain they are used. Not only does she find differences in
aneurysm. What followed was a tragedy, made worse approaches between hospitals, but also between units
by the fact that it needn’t have occurred at all. The and even between shifts. ‘Standardisation and
patient was mistakenly injected with the antiseptic harmonization are two of the most urgent issues we
chlorhexidine. It happened, the hospital says, because have to tackle. Imagine if you have to learn each new
of ‘confusion over the three identical stainless steel system wherever you go or even whenever a new
bowls in the procedure room containing clear liquids senior doctor is on the ward. This introduces massive
— chlorhexidine, contrast dye, and saline solution’. risk.’
Doctors tried amputating one of Mary’s legs to save
Dixon-Woods compares the issue of patient safety to
her life, but the damage to her organs was too great:
that of climate change, in the sense that it is a
she died 19 days later.
‘problem of many hands, with many actors, each
This and similar incidents are what inspired contributing to the outcome, and there is difficulty in
Professor Dixon-Woods of the University of identifying where the responsibility for solving the
Cambridge, UK, to set out on a mission: to improve problem lies. ‘Many patient safety issues arise at the
patient safety. It is, she admits, going to be a level of the system as a whole, but policies treat
challenge. Many different policies and approaches patient safety as an issue for each organization.’
have been tried to date, but few with widespread
Nowhere is this more apparent than the issue of
success, and often with unintended consequences.
‘alarm fatigue’, according to Dixon-Woods. Each bed
Financial incentives are widely used, but recent
in an intensive care unit typically generates 160
evidence suggests that they have little effect. ‘There’s
alarms per day, caused by machinery that is not
a danger that they tend to encourage effort
integrated. ‘You have to assemble all the kit around
substitution,’ explains Dixon-Woods. In other words,
an intensive care bed manually,’ she explains. ‘It
people concentrate on the areas that are being
doesn’t come built as one like an aircraft cockpit.
incentivized but neglect other areas. ‘It’s not even
This is not something a hospital can solve alone. It
necessarily conscious neglect. People have only a
needs to be solved at the sector level.’
limited amount of time, so it’s inevitable they focus
on areas that are measured and rewarded.’ Dixon-Woods has turned to Professor Clarkson in
In 2013, Dixon-Woods and colleagues published a Cambridge’s Engineering Design Centre to help.
study evaluating the use of surgical checklists ‘Fundamentally, my work is about asking how we can
introduced in hospitals to reduce complications and make it better and what could go wrong,’ explains
deaths during surgery. Her research found that those Clarkson. ‘We need to look through the eyes of the
checklists may have little impact, and in some healthcare providers to see the challenges and to
situations might even make things worse. ‘The understand where tools and techniques we use in
checklists sometimes introduced new risks. Nurses engineering may be of value.’ There is a difficulty, he
would use the lists as box-ticking exercises – they concedes: ‘There’s no formal language of design in
would tick the box to say the patient had had their healthcare. Do we understand what the need is? Do
antibiotics when there were no antibiotics in the we understand what the requirements are? Can we
hospital, for example.’ They also reinforced the think of a range of concepts we might use and then
hierarchies – nurses had to try to get surgeons to do design a solution and test it before we put it in place?
certain tasks, but the surgeons used the situation as an
opportunity to display their power and refuse.

• Read the following passage and mark the letter (A, B, C, or D) on your answer sheet
to indicate the correct word or phrase that best fits each of the numbered blanks.
Smartphones have spread beyond the boardroom and keeping a close eye on what's new is Mike
Lazaridis, chief executive of RIM, the company that makes the BlackBerry smartphone. His device
may no longer be quite as trendy as it once was but it is still (21) .................. − particularly for
Argentinian milk farmers. Mr. Lazaridis says: "I like the way it's being used by the farmers so they
are always up to date. Milk is a (22) .................. item, so delivery and production standards are
incredibly important." He is reminded of the company's early days: "This is where we started in
1987, with a wireless vertical data business for the Swedish lumber (23) .................., and then on to
fire brigades, ambulances, and garbage trucks."
While the BlackBerry was quick to (24) .................. itself as the accessory of choice for business
professionals such as Wall Street traders, the cheaper price (25) .................. and new, more colorful
handsets now (26) .................. offer have opened it up to a much wider market. Mr. Lazaridis says:
"It was obvious that business people and self-employed professionals would buy the BlackBerry,
but we were not expecting the (27) .................. we had from ordinary consumers."
The greatest challenge for RIM, however, could lie in an economic (28) ..................., when
individuals and companies alike may see devices like the BlackBerry as luxuries they can (29)
.................. without. The company has to make its product indispensable to the lifestyles of
consumers or prove that they bring a real (30) .................. on investment for business users.

21, A. constructive B. functional C. active D. operative


22, A. perishable B. short-lived C. decaying D. destructible
23, A. manufacturers B. enterprise C. commerce D. industry
24, A. install B. establish C. create D. demonstrate
25, A. designs B. programmes C. plans D. systems
26, A. at B. on C. for D. under
27, A. return B. retort C. reply D. response
28, A. downturn B. tall C. decrease D. drop
29, A. make B. carry C. do D. stay
30, A. return B. gain C. interest D. profit

• Read the following passage and mark the letter (A, B, C, or D) on your answer sheet
to indicate the correct word that best fits each of the numbered blanks.

REMOTE NATURAL ENVIRONMENT


With (1) .................... transportation technologies, it is now possible to reach even the most distant
regions in the world. (2) ......................, I would contend that the merits of this development are
outweighed by its drawbacks.
(3) ..................., the ability to travel anywhere in the world offers (4) ................... benefits. At the
outset, the discovery of new areas allows everyone to gain a better perception of the earth and new
species. This newfound knowledge can act as a precursor to clinical (5) ................... or provide
scientists with valuable insights to more effectively combat environmental issues such as climate
change. Access to previously unreachable scenic beauty also suggests tourism opportunities, as travel
enthusiasts are always on the lookout for new exotic getaways.
Nonetheless, I believe the (6) ................... perspectives of this trend are eclipsed by more immediate
threats to life in this remote natural world. More often than not, as major corporations set foot on and
(7) .................... tourism in these areas, the construction of tourism facilities drives out the indigenous
people living there. Hotels and recreational facilities emerged at the expense of the locals'
accommodation. These people are (8) .................... assimilation, which, as history has indicated, results
in the loss of civilizations and cultures. The presence of foreign visitors in these places also poses
certain threats to the natural environment, specifically water contamination, land degradation, and
natural resource depletion.
In conclusion, (9) ..................... the thought of traveling to remote natural areas may sound, I conclude
that they are better off left untouched.

1, A. the aid of B. the help of C. support of D. assistance of


2, A. That's why B. Nevertheless C. Even though D. All the same
3, A. On the one hand B. Other way C. Moreover D. On one side
4, A. appreciable B. substantial C. considerable D. extensive
5, A. breakthroughs B. steps forward C. improvements D. tapers off
6, A. constructive B. favorable C. peremptory D. positive
7, A. tap into B. expand C. exploit D. utilize
8, A. forced to B. coerced into C. influenced by D. pushed out
9, A. look interesting B. as attractive as C. as fascinating as D. as exciting as
• Read the following passage and mark the letter (A, B, C, or D) on your answer sheet
to indicate the correct word that best fits each of the numbered blanks.
During a decade in which the British (11) ................... industry was finally obliged to make
watchful friends with business, biography has (12) .................... the cultural transition beautifully.
The best biographies still brim with scholarship but they also sell in their thousands. Readers -
ordinary ones with birthday presents to get, book vouchers to spend and rainy holidays to fill - love
buying books about the life and times of their favorite people. Every year before Christmas, a lorry
load of (13) ................... biographies appears on the suggestion table in bookshops.
That biography has done so well is thanks to fiction's vacation of middle-ground, that place
where authorial and readerly desire just about match. Novels in the last ten years,
(14) ..................... to claim the attention of the common reader, have dispersed across several
registers, with the high ground still occupied by those literary novels which continue to play with
(15) ................... concerns about the narrator's impotence, the narrator's fibs and the hero's failure to
veritably exist.
Biography, by contrast, has until recently shown no such unsettling (16) ................... . At its
heart lies the biological plot, the (17) ................... arc with triumphs and children, perhaps a middle-
aged slump or late-flowering dotted along the way. Pages of footnotes peg this central story, this
actual life, into a solid, teeming context. Here was a man or woman who wrote letters, had friends,
ate breakfast, and smelt a certain way. The (18) ................... of being written about rematerializes
the subject on the page. Writing a life becomes a way of reaffirming that life itself endures.
Until now, that is. Recently biography has started to display all the quivering (19) ..................
which changed the face of fiction twenty years ago. Exhaustion now characterizes the genre. All the
great lives have been done. But there are ways of proceeding. Ian Hamilton was the pioneer who
failed to find J.D. Salinger. Five years later, Janet Malcolm's study of Sylvia Plath, The Silent
Woman, (20) ................... exposed how academics and biographers stalk and hunt one another
around the globe in a bid to possess and devour their subject.

11, A. distributing B. publishing C. issuing D. circulating


12, A. line-managed B. line-operated C. line management D. well-managed
13, A. brick-wall B. brick-thick C. brick veneer D. solid brick
14, A. vulnerable B. capable C. impotent D. unable
15, A. postmodernist B. deconstructive C. post-modern D. contemporary
16, A. self-effacement B. patience C. self-abnegation D. humility
17, A. birth-to-death B. born-to-die C. mortal D. life-and-death
18, A. procedure B. progression C. process D. proceeding
19, A. self-reflection B. self-scrutiny C. self-monitoring D. introspection
20, A. dazzlingly B. marvelously C. splendidly D. brilliantly
• Read the novel quote and mark the letter (A, B, C, or D) on your answer sheet to
indicate the correct word that best fits each of the numbered blanks.
When we think about how (1) .................... and how common is this type of comic, we
understand why it (2) ..................... the imagination of certain philosophers. To cover a good deal
of ground only to come back (3) ................... to the starting point is to make a great effort for a nil
result. So we might be tempted to define the comic in this latter fashion. And such, indeed,
seems to be the idea of Herbert Spencer: according to him, laughter is the indication of an effort
that suddenly encounters avoid. Kant had already said something of the kind: "Laughter is the
result of an expectation, which, of a sudden, ends in nothing." (4) ...................., these definitions
would apply to the last few examples given, although, even then, the formula needs the addition
of sundry limitations, for we often make an ineffectual effort which is in no way provocative of
laughter.
While the last few examples are (5) .................... of a great cause resulting in a small effect, we
quoted others, immediately before, which might be defined inversely as a great effect springing
from a small cause. The truth is, this second (6) .................... has scarcely more validity than the
first. Lack of proportion between cause and effect, whether appearing in one or the other, is
never the direct source of laughter. What we do laugh at is something that this lack of proportion
may in certain cases disclose, namely, a particular (7) ..................... arrangement which it reveals
to us, as through a glass, at the back of the series of effects and causes. Disregard this
arrangement, and you let go of the only clue capable of guiding you through the labyrinth of the
comic. Any hypothesis you (8) .................... would select, while possibly applicable to a few
carefully chosen cases, is liable at any moment to be met and overthrown by the first unsuitable
instance that comes along.
1, A. passionate B. intense C. extraordinary D. vehement
2, A. was fascinated B. has fascinated C. will be fascinated D. fascinated
3, A. unwittingly B. innocently C. coincidentally D.
deliberately
4, A. Undoubtedly B. Indisputably C. Definitely D. Indeed
5, A. drawings B. engravings C. illustrations D. adornments
6, A. delineation B. explanation C. ambiguity D. definition
7, A. automatic B. technical C. mechanical D. productive
8, A. unless B. likewise C. otherwise D.
dissimilarly
Many parents try to discourage their offspring from
embarking on a career in the music business. They feel that
the music industry is not the glamorous one that young
people imagine it to be and that the chances of their son or
daughter becoming a highly paid star are extremely slim.
Although this is quite correct, good opportunities do exist
for young people in the music industry, particularly for
those who place greater value on job satisfaction than on
salary levels.
Naturally, there are opportunities for those who have
studied an instrument, but many jobs in the industry do not
require either the technical ability or performing skills.
There are many openings, for example, in publishing,
retail, and in recording studios where what is required is a
lively interest in music and a good level of background
knowledge. A degree in a relevant subject, whilst not certainly essential, can also be useful, and
courses specifically designed to meet the needs of the music industry are now available.

WRITING THE ESSAY (150 – 200 words)


………………………………………………….
…………………………………………………….………………………………..…….…
………………………………………………….
…………………………………………………….……………………….……………..…
………………………………………………….
…………………………………………………….…………………………….…….….…
………………………………………………….
…………………………………………………….…………………………….…….….…
………………………………………………….
…………………………………………………….…………………………….…...……..
………………………………………………….
…………………………………………………….……………………………….……..…
………………………………………………….
…………………………………………………….…………………………….…….….…
………………………………………………….
…………………………………………………….…………………………….…….….…
………………………………………………….
…………………………………………………….…………………………….…….….…
………………………………………………….
…………………………………………………….…………………………….…….….…
………………………………………………….
…………………………………………………….………………………….……….….…
………………………………………………….
…………………………………………………….…………………………….….…….…
………………………………………………….
…………………………………………………….………………………….……….….…
………………………………………………….
…………………………………………………….…………………………….….…….…
A flood occurs when water inundates land that's normally dry, which can happen in a multitude
of ways. Excessive rain, a ruptured dam or levee, rapid melting of snow or ice, or even an
unfortunately placed beaver dam can overwhelm a river, spreading over the adjacent land, called
a floodplain. Coastal flooding occurs when a large storm or tsunami causes the sea to surge
inland.Most floods take hours or even days to develop, giving residents time to prepare or
evacuate. Others generate quickly and with little warning. So-called flash floods can be
extremely dangerous, instantly turning a babbling brook or even a dry wash into rushing rapids
that sweep everything in their path downstream.Climate change is increasing the risk of floods
worldwide, particularly in coastal and low-lying areas, because of its role in extreme weather
events and rising seas. The increase in temperatures that accompanies global warming can
contribute to hurricanes that move more slowly and drop more rain, funneling moisture into
atmospheric rivers like the ones that led to heavy rains and flooding in California in early 2019.
….………………………….……….….…
Evidence for the existence of sharks dates from the Ordovician period, 450–420 million years
ago, before land vertebrates existed and before a variety of plants had colonized the continents.
Only scales have been recovered from the first sharks and not all paleontologists agree that these
are from true sharks, suspecting that these scales are actually those of thelodont agnathans. The
oldest generally accepted shark scales are from about 420 million years ago, in the Silurian
period. The first sharks looked very different from modern sharks. At this time the most common
shark tooth is the cladodont, a style of thin tooth with three tines like a trident, apparently to help
catch fish. The majority of modern sharks can be traced back to around 100 million years ago.
Most fossils are of teeth, often in large numbers. Partial skeletons and even complete fossilized
remains have been discovered. Estimates suggest that sharks grow tens of thousands of teeth
over a lifetime, which explains the abundant fossils. The teeth consist of easily fossilized
calcium phosphate, an apatite. When a shark dies, the decomposing skeleton breaks up,
scattering the apatite prisms. Preservation requires rapid burial in bottom sediments. (Shark, n.d.)
………………………………………………….
…………………………………………………….…………………………….….…….
…………………………………………………….
…………………………………………………….…………………………….….…….…
Nearsighted people experience blurred vision while looking at distant objects, such as street
signs viewed from inside a car. Conversely, farsighted people experience blurred vision while
looking at nearby objects, such as words on a page. The originators of these two conditions
differ as well. Nearsightedness, or myopia, occurs either when a person's eye is too long from
back to front or when the cornea (the clear layer at the front of the eye) is too curved. On the
other hand, farsightedness, or hyperopia, results when the eye is too short or when the cornea is
not curved enough. Fortuitously, corrective lenses can restore normal vision in both cases. As
you might expect, these lenses are of opposing types: nearsightedness is improved with a lens
that is thinner in the middle and thicker at the edges, while farsightedness is compensated for
with a lens that is thicker in the middle and thinner at the edges.

• Read the following passage and mark the letter (A, B, C, or D) on your answer sheet
to indicate the correct word or phrase that best fits each of the numbered blanks.
To celebrate Google's 10th birthday in 2008, Google launched a competition which is called 10100
. (0) ........... of the public were asked to take part and (1) ........... suggestions for projects which
they wanted to bring to life with the intent to make the world a better place. One of the winnings
(2) ........... was from a company in New Zealand called Shweeb. Their aim was to (3) .............
elements of the monorail together with those of the bicycle, and to produce a clean and fast
vehicle, which could reduce congestion. In the end, Google (4) ........... $1 million in Shweeb's
research and development.
The world's first Shweeb was built at the Velocity Valley Adventure Park in New Zealand.
Riders sit in a capsule suspended from a rail, and pedal as fast as they can for three laps around a
200-meter (5) ........... . In the past, prizes were awarded to anyone who broke a speed (6) ...........,
such as being the fastest rider from their country. However, the adventure park has recently
stopped these records after upgrading the capsules to have electric assistance.
Shweeb has become a very popular visitor (7) ........... and time will tell if it may also be the
(8) ........... of the future for commuters, business travelers, and tourists.

0, A. Members B. Aspects C. Representatives D. Elements

1, A. submit B. apply C. encourage D. approach

2, A. accession B. gaining C. acquisition D. attainment

3, A. Another B. An alternative C. Any more D. A further

4, A. unanticipated B. unexpected C. unforeseen D. unpredicted

5, A. Extensive B. All-encompassing C. Wide-ranging D. Large-scale

6, A. get cracking on B. embarking on C. preamble of D. beginning of

7, A. popularly B. commonly C. generally D. prevalently

8, A. flare-up B. ebullition C. outbreak D. occurrence


Of the more than 4,000 species of mammals that have (0) .....D...... our earth over the past
10,000 years, the horse is one of fewer than a dozen which have been successfully domesticated.
Domestication is not simply a (1) ................ of human intention. If it were, it is possible that we
would now be sitting in our fireside chairs with a hyena curled at our feet.
Much of what we take for (2) ................. as useful in the modern horse - speed, size, and
intelligence, for example - can be explained through the evolutionary changes it has
(3) ................ in response to a changing diet. As the Ice Age advanced and forests (4) ................
away, to be replaced by windswept savannah, many herbivores were (5) .................. to change
their diets from leaves to grass. The little leaf-browsing predecessor of our modern horse - the
ur-horse - began to change and adapt to a new ecological niche on the plains. The head
(6) ................ longer, with the eye, positioned at some (7) ................ from the mouth so that in
exposed spaces it could keep a careful (8) ................. for predators while it grazed. A larger
brain began to develop, probably because, as a grazer, it needed greater tactile sensitivity in its
lips to choose its food.

• Read the following passage and mark the letter (A, B, C, or D) on your answer sheet
to indicate the correct word or phrase that best fits each of the numbered blanks.

Due to the development and use of (1) ____________ weapons near the end of World War II
and their subsequent (2) ____________ and deployment by many countries, the potential risk of
nuclear devastation of Earth's civilization and life is a common theme in speculations about a
Third World War. (3) ____________ major concern is that biological warfare could cause a very
large number of casualties, either intentionally or inadvertently by an accidental release of a
biological agent, the (4) ____________ mutation of an agent, or its adaptation to other species
after use. (5) ____________ apocalyptic events like these, caused by advanced technology used
for destruction, could potentially make the Earth's surface uninhabitable.
Before the (6) ____________ the Second World War, the First World War (1914 –1918) was
believed to have been "the war to end all wars", as it was (7) ____________ believed that never
again could there possibly be a global conflict of such magnitude. During the interwar period,
WWI was typically referred to simply as "The Great War". The (8) ____________ of World
War II in 1939 (9) ____________ the hope that mankind might have already "outgrown" the
need for such widespread global wars.

1, A. atomic B. nuclear C. fissile D. microscopic

2, A. accession B. gaining C. acquisition D. attainment

3, A. Another B. An alternative C. Any more D. A further

4, A. unanticipated B. unexpected C. unforeseen D. unpredicted

5, A. Extensive B. All-encompassing C. Wide-ranging D. Large-scale


6, A. get cracking on B. embarking on C. preamble of D. beginning of

7, A. popularly B. commonly C. generally D. prevalently

8, A. flare-up B. ebullition C. outbreak D. occurrence

9, A. challenged B. disproved C. refuted D. contradicted

• For each space, choose ONE word which you think best completes the sentence.
Look carefully at the words both before and after each space.
Looking inside their heads, we can see that's the obvious reason. The elephant boasts the largest
brain of any land mammal, as well as an impressive encephalization quotient. This is the size of the
brain relative to what we'd expect for an animal's body size and the elephant's EQ is approximately
as high as a chimpanzee's. And despite the distant relation convergent evolution has made it
remarkably similar to the human brain, with as many neurons and synapses in a highly developed
hippocampus and cerebral cortex. It is the hippocampus strongly associated with emotion that aids
recollection by encoding important experiences into long-term memories. The cerebral cortex, on
the other hand, enables problem-solving which elephants display in many creative ways. They
likewise tackle problems cooperatively, sometimes even outwitting the researchers and
manipulating their partners. But perhaps the most amazing thing about elephants is a capacity even
more important than cleverness. Their sense of empathy, altruism, and justice.
• Read the following passage and mark the letter (A, B, C, or D) on your answer sheet
to indicate the correct word that best fits each of the numbered blanks.

REMOTE NATURAL ENVIRONMENT


With (1) ................... transportation technologies, it is now possible to reach even the most distant
regions in the world. (2) ........................, I would contend that the merits of this development are
outweighed by its drawbacks.
(3) ..................., the ability to travel anywhere in the world offers (4) ................... benefits. At the
outset, the discovery of new areas allows everyone to gain a better perception of the earth and new
species. This newfound knowledge can act as a precursor to clinical (5) ................... or provide
scientists with valuable insights to more effectively combat environmental issues such as climate
change. Access to previously unreachable scenic beauty also suggests tourism opportunities, as travel
enthusiasts are always on the lookout for new exotic getaways.
Nonetheless, I believe the (6) ................... perspectives of this trend are eclipsed by more
immediate threats to life in this remote natural world. More often than not, as major corporations set
foot on and (7) .................... tourism in these areas, the construction of tourism facilities drives out the
indigenous people living there. Hotels and recreational facilities emerged at the expense of the locals'
accommodation. These people are (8) .................... assimilation, which, as history has indicated, results
in the loss of civilizations and cultures. The presence of foreign visitors in these places also poses
certain threats to the natural environment, specifically water contamination, land degradation, and
natural resource depletion.
In conclusion, (9) ........................ the thought of traveling to remote natural areas may sound, I
conclude that they are better off left untouched.

1, A. the aid of B. the help of C. support of D. assistance of


2, A. That's why B. Nevertheless C. Even though D. All the same
3, A. On the one hand B. Other way C. Moreover D. On one side
4, A. appreciable B. substantial C. considerable D. extensive
5, A. breakthroughs B. steps forward C. improvements D. tapers off
6, A. constructive B. favorable C. peremptory D. positive
7, A. tap into B. expand C. exploit D. utilize
8, A. forced to B. coerced into C. influenced by D. pushed out
9, A. look interesting B. as attractive as C. as fascinating as D. as exciting as
• Use the word given in brackets to form a word that fits in the space. The first one has
been done as an example (0).
THE AMAZONS
The Amazons, a tribe of fierce female (0. WAR) WARRIORS feature in Ancient Greek
mythology from 2,000 years ago. The Ancient Greeks believed them to inhabit an area called
Scythia, covering present-day Ukraine, southern Russia, and western Kazakhstan. They were
fascinated by the Amazons, writing stories about their heroic and (1. COURAGE)
.......................... deeds, and displaying them on pots and items of (2. JEWEL) ........................... .
They were tall and beautiful but also completely (3. FEAR) ........................... and as strong as
the men they fought. The Amazons lived in an all-female society. According to which story one
reads, they either abandoned their male babies, sent them to live with their fathers from another
tribe, or else gave them away at (4. BORN) ......................... . There may be some (5. TRUE)
........................... in the Amazon myth because archaeologists have discovered burial sites in
Ukraine and (6. NEIGHBOUR) ...................... countries dating from 2,000 to 3,000 years
ago. The women were buried with weapons and some display (7. EVIDENT) ......................... of
wounds. Their bones suggest they would have been (8. USUAL) ......................... tall for women
of this time.

KEY:
1, courageous 2, jewellery / jewelry 3, fearless 4, birth
5, truth 6, neighbouring 7, evidence 8, unusually
• Read the following passage and mark the letter (A, B, C, or D) on your answer sheet
to indicate the correct word that best fits each of the numbered blanks.

BERTHA
Fifty feet below downtown Seattle, the world's largest (1) _________ machine, nicknamed
Bertha after the city's (2) _________ mayor, lies still. This (3) _________ vehicle weighs 7,000
tons, stretches nearly 330 feet long, and has a 60-foot diameter. When fully (4) _________,
Bertha has an intrepid crew of 25. In early August, Bertha began carving out a two-mile road
tunnel to replace the Alaskan Way viaduct, an ugly double-decker motorway that has been (5)
_________ Seattle's waterfront since the 1950s.
Dozens of (6) _________ were considered, including twin tunnels, another elevated roadway,
and even scrapping the highway in favor of improved surface roads and better public transport.
As it happens, (7) _________ dithered long enough for tunneling technology to develop (8)
_________ facilitating another option: a single tunnel large enough to accommodate two levels
of multi-lane traffic. Many local (9) _________ are against Bertha, but she has already chewed
up and spit out one politician. As the new mayor settles into his new role, he will surely be
hoping that Bertha (10) __________ finds her appetite for muck, not mayors.

1, A. tunnel-drilling B. drillhole C. tunnel-boring D. soil-drilling

2, A. increasing B. progressing C. progressive D. progression

3, A. remarkable B. extraordinary C. noteworthy D. impressive

4, A. operating B. operational C. operative D. postoperative

5, A. mutilating B. disfigured C. deformed D. disfiguring

6, A. replacements B. alternatives C. surrogates D. substitutions

7, A. politicians B. legislators C. lawmakers D. policymakers

8, A. satisfactorily B. abundantly C. adequately D. sufficiently


9, A. delegators B. commissioners C. legislatures D. representatives
10, A. speedily B. quickly C. rapidly D. precipitately
• Complete this text about use of the fork by adding the following words in the correct
place. Use each word once.

have been used at the end of inventory of


fork-wielding have done claimed pretended

Fork-like implements for spearing food (0) ___________ for over 2000 years. For instance, the
Ancient Romans used to use special spoons with one prong or two (1) ___________ the
handle for winkling out shellfish. The first ‘modern’ fork is thought to have been used in the
eleventh century by the wife of the Venetian Doge. Forks are mentioned repeatedly three
centuries later, in 1361, in an (2) __________ the plate owned by the Florentine Commune.
Nevertheless, the first real evidence that forks had spread as eating tools came some two
hundred years later, in a 1570 engraving of a knife, fork, and spoon. In 1605, King Henri III of
France and his companions were satirized for their (3) __________ effeminacy.

‘They would rather touch their mouths with their little forked instruments than with their
fingers,’ wrote Thomas Artus, who (4) __________ that they looked especially silly as they
strove to capture the peas and broad beans on their plates – as well they might (5) ___________
since early forks had long, widely separate prongs and scooping with them must have been
impracticable.

access board companson granted justification


question saying short surprise worse

It's funny how we all tend to take education for (1) __________ these days but only a couple of hundred
years ago, a good education was out of the (2) _________ for the majority of the population. Education
was the prerogative of the select few and it was only in the last century that schooling became widely
available for all children up to the age of 16 or 18. The trouble is, familiarity breeds contempt, as the
saying goes, so it comes as no (3) __________ that a large number of schools come under attack
nowadays, both from students and from parents. Some of the comments you might hear are: 'Education
is going from bad to (4) __________ !' or 'Our schools are so (5) _________ of funds they can't create
any more sporting facilities!' However, it's very easy to forget that, generally speaking, most children
nowadays have (6) _________ to a prettY good all-round education, It goes without (7) _________ that
there's no (8) __________ between the facilities generally available nowadays in schools and colleges
and what was around last century. Not only that, but so much knowledge is on hand at the click of a
mouse. So, one might say, there is thoroughly no (9) _________ for ignorance or narrow-mindedness in
today's world. Well, that's a thought to take on (10) _________, isn't it?
• Complete this text about sports commentary by adding the following words in the
correct place. Use each word once.

international throughout referee qualified


decision players pitch assessed tournament

Who would want to be a match official?


Dennis O’Mackton, 53, is an (0) __________ hockey referee and has also worked as a match official
in other sports, including tennis, football, and boxing.

I’ve been a match official of one kind or another (1) __________ much of my life. Hockey (2)
__________, tennis umpire, rugby linesman, I’ve done it all. And when I tell people, the reaction
varies from pity (‘Oh dear, you weren’t good enough to be a player?) to disbelief (‘Why on earth would
anyone want to do that?’) Accurately, I think both these attitudes are mistaken for several reasons, and
I'd like to set the record straight. For a start, many people truly prefer to be a (3) __________ or
umpire than a player, and I’m one of them.

Having said that, many match officials have probably at times wondered why they bother to do it.
Whatever (4) __________ you make, especially where there’s been foul play, there’s always going to
be someone who disagrees. And sometimes in the heat of the moment, emotions can get the better of
people. I’ve had five angry footballers all yelling at me on several occasions. You have to have thick
skin - I sometimes ask myself why I am giving up my Sunday to run around a (5) __________ in the
cold rain with (6) __________ who don’t seem to respect me or the work I do, and who think it’s their
right to give the ref a load of abuse.

But despite this, I’ve always loved it. I now (7) __________ top-level professional hockey games.
Believe me, for a couple of hours when I’m out on the (8) __________ with the players, it’s the best
seat in the house. The skill of some of these top (9) __________ is astounding, as are their fitness
levels.

Of course, in this line of work, you don’t start at the top. I oversaw something like 1,000 Sunday
morning hockey games before I ever did a pro game. These were junior and adult games. I think this is
key to a match official’s arbitration success at higher levels. There has to be enough experience for him
or her to draw on, whatever the sport is. There is a concern that in some sports, young match officials
are rising very quickly and officiating in a major (10) _________ before they’re ready. I know that
I’ve evolved in the years I’ve worked as a hockey referee. In the early years, I wasn’t really (11)
___________ formally, not like newly (12) __________ referees nowadays. But informally, I was
being (13) __________ by coaches, players, and spectators every game. I occasionally heard criticisms
in some of my matches that I was stopping play too frequently. So now I’m more reluctant to blow the
whistle unless it’s a more serious foul.
• Complete this text on the citation section of the article by adding the following words
in the correct place. Use each word once.

unexpected among peak car unnoticed transport


recognized average environmentalists congestion social

The end of the road for motormania


Something (0) __________ is happening to our __________ person traveled in a car that
car-crazy culture. What are the forces driving us continues today.
out of motoring? Is the west falling out of love
with the car? What could be driving us off the road? Fuel costs
and rising insurance premiums may be a factor.
For (1) _________, it seems an impossible And urban gridlock, combined with an absence of
dream, but it is happening. While baby boomers parking places and (7) __________
and those with young families may carry on using charging, makes an increasing number of us look
four wheels, a combination of our aging societies on the car as a dumb way to move around in cities
and a new attitude (2) __________ the where there are public (8) __________
young seems to be breaking our 20th-century car alternatives. Demographics are another possible
addiction. Somewhere along the road, we reached explanation. It is surely no accident that ‘(9)
the high point of the car and are now cruising __________’ happened first in Japan, which has
down the other side. the world’s oldest population. Pensioners do not
That zenith takes several forms. Sales of new cars drive to work, and many don’t drive at all. There
have almost halved in the US, down from nearly is also the rise of “virtual commuters” who work
11 million in 1985 to about 5.5 million now. We from home via the Internet.
shouldn’t take much notice of that, though. Cars
On the side of these new employment patterns,
last longer these days, and sales go up and down
leisure lifestyles are also changing. The biggest
with the economy. But we have hit (3)
fall in car use in the US is (10)
__________ ownership, too. And, more to the
point, peak per-capita travel.
__________ people under 35. The fraction of
American 17-year olds with a driver’s license has
The phenomenon was first (4) _________ in The fallen from about three-quarters to about half
Road... Less Traveled, a 2008 report by the since 1998. Twenty-somethings have recently
Brookings Institution in Washington DC, but had gone from driving more than the (11)
been passing largely (5) ________ for years. _________ to driving less. (12) _________
Japan reached it in the 1990s. They talk thereof scientists detect a new ‘culture of urbanism’. The
“de motorization”. The west had its tipping point stylish way to live these days is in inner-city
in 2004. That year the US, UK, Germany, France, apartments, not the suburbs. Richard Florida, an
Australia, and Sweden all saw the start of a urban studies theorist at the University of Toronto
decline in the number of kilometers the (6)
• You are going to read a short text. Three sentences have been removed from the text.
Choose from line sentences A-G the one which fits each gap (1 - 3).

Volunteer tourism
It is hard to argue that the actual contribution to development amounts to a great deal directly.
(1) __________, they have usually paid a significant fee for the opportunity to be involved in
this work: money that, if donated to a local community directly, could potentially pay for a
greater amount of labor than the individual volunteer could ever hope to provide. (2)
__________. Hence, it is unsurprising that many academic studies allude to the moral issue of
whether gap year volunteering is principally motivated by altruism – a desire to benefit the
society visited – or whether young people aim to generate ‘cultural capital’ which benefits them
in their careers. (3) __________.

SENTENCES:
A. Whilst volunteer tourists can get involved in building homes or schools
B. At its worst, international volunteering can be imperialist
C. Yet it also bypasses the democratic imperative of representative government and reduces
development to individual acts of charity
D. However, if volunteering is largely limited to individuals of means from wealthier areas of
the world
E. This is especially so in the case of gap years, in which the level of technical skill or
professional experience required of volunteers is negligible
F. However, where volunteer tourism is talked up as sustainable development and the marketing
of the gap-year companies merges into development thinking
G. However, the projects may play a role in developing people who will, in the course of their
careers and lives, act ethically in favor of those less well-off

1, Scientists have known for some time that gold deposits are derived from hot, mineral-rich
water that flows through the earth's crust.

2, In 2013, Australian scientists used a mathematical model to show a strong link between the
formation of gold deposits and seismic activity.

3, Many biomes appear on multiple continents but share a similar latitude or elevation; however,
hot desert regions exist not only at the equator but also as far south as Australia.

4, Research has shown that music training can help improve reading skills through the way it
teaches students to distinguish between sounds and recognize language patterns.

5, Many wikis were first launched in the twenty-first century, while encyclopedias have been
used as reference sources since the eighteenth century.

6, At moments of sorrow and exhaustion, it is only too easy to look back over the years and feel
that our lives have, in essence, been meaningless.

7, Mistakes do not have to be absurd; they can be signs of how little information we have on
which to base the most consequential decisions.
8, But by taking life a little less seriously, understanding, and empathizing with the idea that life
in the universe is vastly complex and random, you will not break under such pressures.

9, It is the hippocampus strongly associated with emotion that aids recollection by encoding
important experiences into long-term memories.

10, The elephant boasts the largest brain of any land mammal, as well as an impressive
encephalization quotient.

11, Shakespeare's poetry showed the world that English was a rich, vibrant language with
limitless expressive and emotional power.

12, These exercise sessions also help to break up the study sessions which is crucial to avoiding
cramming a scientifically disproven method.

13, Studying these samples will be the job of the Analytical Lab, a trio of instruments designed
to search for the molecular fingerprints of life.

14, The judge pronounced that the landfill company had dem; stretch a cavalier disregard for
regulations and that the damage inflicted on the environment was substantial.

15, There have been rumors that the leading lady is romantically involved with the lighting
technician but perhaps we shouldn't assume too much.

16, More nations will be approaching US consumption levels in years to come and previous data
suggest that at least four additional Earths will be needed to sustain such a level.

17, As the government embarks on its plans, environmentalists are asking if economic growth
really is compatible with environmental preservation.

18, Undoubtedly, people in the developed world are living longer, but are the dreams of living
forever that have been a mainstay of popular culture.

19, For the first time since the dinosaurs disappeared, humans are driving animals and plants to
extinction faster than new species can evolve.

20, Dickens remained until his death 34 years later innumerable the most popular novelist the
English-speaking world had ever known.

21, While the Human Genome Project may grab the headlines, bioinformatics has a much more
direct impact on our lives for the advances it has enabled in the field of agriculture.

22, Attempts have been made to describe how Ellington recordings have come into being, but
the process is so subtle that verbalization appears crude.

23, Cynics would assume it is an instance of dissonance reduction - of employers willing


passionately to entrust in tests for which they have separated with an extensive amount of
money.
VĂN BẢN:
Hành động có ảnh hưởng mạnh hơn lời nói. Khi chúng ta cười, nét mặt chúng ta tràn ngập
niềm hân hoan và tình thân mến. Một nụ cười niềm nở tự đáy lòng có thể thay cho lời nói : "Tôi
thật sư quý mến anh" hay là "Tôi thật sự rất vui khi gặp bạn vì bạn làm cho tôi hạnh phúc".

… Nụ cười có sức tác động vô cùng mạnh mẽ ngay cả khi người ta không nhìn thấy nó. Các
công ty điện thoại trên toàn nước Mỹ đều có chương trình huyấn luyện mang tên "Sức mạnh
của nụ cười". Trong chương trình này, các điện thoại viên được yêu cầu phải luôn mĩm cười
khi trả lời điện thoại vì người nghe có thể nghe được nụ cười của họ qua giọng nói. Bạn hãy thử
nhắc máy lên gọi cho bạn bè hay người thân của mình. Lần đầu, bạn không mỉm cười khi nói.
Lần thứ hai, bạn mĩm cười thật tươi. Bạn sẽ thấy điều kỳ diệu trong thái độ mà bạn sẽ đem đến
cho người nhận cuộc gọi. Trong nhiều trường hợp, điều này còn quyết định hiệu quả của cả một
ngày làm việc của họ và của chính chúng ta.

People get inspiration from a variety of sources. We're conditioned to believe that it should be
something moving, such as a beautiful landscape, a touching work of art, or a well-crafted story. For
many people, though, they derive their motivation from how much money they'll make from a
particular endeavor. Money does allow us to live comfortable lives; for those who like to live
extravagantly, they need a lot of it to do so, and then some miserly folks simply like accumulating it
but never spending it. Much criticism, though, is levied at people who sacrifice decency in the
pursuit of money. And so often, when looking at the corporate world and the actions that take place
in this environment, it's not hard to find stories of people taking bribes or of officials involved in the
misuse of funds. There's also an aggressiveness that persists in how people do business; like hawks,
they wait to snatch up an undervalued property in the hope of turning a profit, even at the expense
of low-income residents who are powerless to stop it. But as the saying goes, it is not the person
who has too little, but the one who craves more, that is poor.
TEXT 1:
Postgraduate students occasionally write dissertations on rather bizarre topics. Some might
seem utterly ridiculous to the casual onlooker and it's anyone's guess as to the researcher's
motive behind exploring such a topic. One studies jumping height differences between cat fleas
and dog fleas, another hypothesized that mosquitoes may be more attracted to Limburger cheese
than humans are and a third gives an analysis of the dangers of sword swallowing. In terms of
the study of life, mankind leaves no stone unturned.
A perhaps more useful but still unusual subject was published by a researcher at University
College London. An authority on neuroscience, Eleanor Maguire studied the brains of London's
black cab drivers. Having to memorize routes along more than 25,000 streets, a cabbie of the
British capital has anything for poor memory. Maguire learned that these assiduous drivers have
larger posterior hippocampi - the part of the brain responsible for spatial memory. This serves as
proof that a brain is like a muscle and it's imperative that we flex it.

TEXT 2:

June and Jennifer Gibbons were twin sisters known as 'the Silent Twins' due to their
slightly abnormal habit of speaking only to each other and not anyone else. Born in
Barbados, their family relocated to Haverfordwest in Wales when they were infants, and
their ethnic differences made integration with the local community quite difficult. The
two girls were inseparable during their youth, a result of their being ostracised from their
community yet needing companionship. Making matters worse was the unintelligible
language that the sisters used to speak to each other; it was a mixture of English and their
native language.
Although bullied at school, the two hardly had misspent youths; they enjoyed writing
fiction and dreamt of being writers. Their first novels, however, failed to attract much
literary attention and, in a sad twist of events, they turned to a life of crime and exhibited
even more aberrant behavior. The twin sisters eventually ended up in a mental health
facility, and after much fruitless analysis and observation, staff concluded that their
condition was seemingly incurable. It wasn't until one sister, Jennifer, passed away that
June was able to live a normal life. What was certainly true about the sisters is that
throughout their troubled lives together, they loved and trusted each other
unconditionally.
TEXT 3:

The two passages explore the concept of cities as environments in which people can achieve
success, but with contrasting views.
The first text suggests that cities offer fertile ground for development In any number of
professions for those who are driven to succeed. Not only do cities offer countless opportunities,
but because they attract the best of the best, competition plays a role In pushing others to keep
up and therefore also succeed. The text also asserts that competing against talented individuals
provides an opportunity to learn. I am inclined to agree with this. We cannot deny that most
careers are forged in cities and that being around successful people brings about a deal to do
better. Nevertheless, it should be noted that competition is not always healthy and it can result in
unscrupulous behavior.
The second text argues that expectations are not always met and that this gives rise to some
very undesirable dwellings. It proposes that failure is made worse when it is measured against
the success of others; some of whom like; to flaunt. their accomplishments. In addition, it puts
forward the idea that failed ambitions can trigger depression and rob people of their will to
succeed, as well as lead to crime. There is no doubt In my mind that cities can be tough places to
pursue one's dreams. Competition is intense due to so many people vying for the same career
opportunities, and expectations are often unrealistic.
All things considered, cities are where careers are built and will always appeal to people
hungry for success. Nonetheless, success is not guaranteed, and people should reassess their
goals regularly in an attempt to avoid the disappointment of failure.
TEXT 4:

There is a universal understanding that evil is bad. Why then has it flourished? Perhaps it is
because decent, law-abiding people have not spoken out against it or taken measures to prevent
it. The reasons for their reluctance are many, but how can these be overcome to turn inaction
into action?
There is a time when good men and women must stand up for what is right, even when it
involves risk, but that moment comes only after evil has already been well established. For
instance, how many people could dictators have killed with no one to take their orders? How
many people would have starved to death without obedient middlemen? With dutiful followers,
however, evil rulers killed millions of people in the 20th century alone. Were it not for basically
decent people ready to obey, evil would quickly fail. Such people would not initiate destruction
by themselves, but perhaps in the name of duty, loyalty, or the greater good, they cooperate with
evil. Conceivably, they are driven by fear of repercussions should they not obey. Each one plays
only a small part in the misery brought upon others, and therefore they do not contemplate the
results of their actions — as far as they are concerned, it is nothing to do with them.
If obedience towards evil is rooted in fear, how can we eliminate the fear? The answer may lie
in our collective strength. Our history is full of instances of revolution and rebellion – some
peaceful, many more bloody. Regimes, dictators, and tyrants have been toppled by the will of
the people. Surely this shows us that a combined, concerted effort can face down evil and
prevent it from causing further destruction. This holds real for battles great and small. A good
illustration of this is the Neighbourhood Watch schemes that are in force in neighborhoods
around the world. Working together, residents patrol and protect their areas from criminals.
In summary, it appears that people do not act to prevent harm if doing so puts their own lives
at risk. This fear can be overcome by working with others in the struggle against the ills of our
time.
1, Researchers have been studying the psychological phenomenon known as false memory since
the mid-1970s. (PASSIVE-VOICE)
→ The psychological phenomenon ………..…….…………………………………..…..…….
……………

2, Although it will be a modern town with all the necessary facilities for modern-day living, the
town will have a traditional feel to it.
→ In spite of ………..…….………………..………..…………..…….………..….……………

3, I'm writing to tell you one of my most embarrassing stories - it happened at the beginning of
the school year.
→ If ………..…….………………..…….……………………..……………………..…….
………..….……………

4, Though it doesn't feel great to embarrass herself, Daisy thinks that it is important to learn to
laugh at herself and not take life too seriously.
→ Despite ………..…….………………..…….……………………..
……………………………..…….…….…

5, The only improvement that I can suggest is creating better workspaces for the students.
→ I suggest ………..…….………………..…….……………..…….…….…………..

……………………..…….

6, The judges were terribly dissatisfied that the contestants didn't complete their performance.
(IF)
→ ………..…….………………..…….……..…….……………..…….…….…………..
……………………..…….

7, He used to be an airline pilot. He was able to travel the world without paying for plane
tickets. (WISH)
→ ………..…….………………..…….……..…….……………..…….…….…………..
……………………..…….
8, A decline in honeybee populations has been recorded in recent years. The decline of
honeybee populations was significantly lower over the 2013–2014 winter. (RELATIVE
CLAUSE)
→ The decline of honeybee populations ………….………………….……..
……………………..……. ………..…….………………..…….……..…….……………..…….
…….…………..……………..…………..…….
9, Dean Kamen invented a two-wheeled electric vehicle called a Segway. He founded the
FIRST Robotics Competition for high school students in 1989. (RELATIVE CLAUSE)
→ Dean Kamen, …….……….…………………………….………………….……..
……………………..……. ………..…….………………..…….……..…….……………..…….
…….…………..……………..…………..…….

10, The artist was pleased with the finished portrait, although nervous about the sitter’s reaction
when it was unveiled. (UNLESS)
→ ………..…….………………..…….……..…….……………..…….…….…………..
……………………..…….

• Read the following passage and choose the best answers:


Literature, in its broadest sense, is any written work. Etymologically, the term derives from
Latin litaritura “writing formed with letters,” although some definitions include spoken or sung
texts. More restrictively, it is writing that possesses literary merit. Literature can be classified
according to whether it is fiction or non-fiction and whether it is poetry or prose. It can be
further distinguished according to major forms such as the novel, short story, or drama, and
works are often categorized according to historical periods or their adherence to certain
aesthetic features or expectations.

Taken to mean only written works, literature was first produced by some of the world’s earliest
civilizations—those of Ancient Egypt and Sumeria—as early as the 4th millennium BC; taken
to include spoken or sung texts, it originated even earlier, and some of the first written works
may have been based on a pre-existing oral tradition. As urban cultures and societies developed,
there was a proliferation in the forms of literature. Developments in print technology allowed
for literature to be distributed and experienced on an unprecedented scale, which has culminated
in the twenty-first century in electronic literature.

1,
Jellyfish and sea jellies are the (1) ______ common names given to the medusa-phase of certain
gelatinous members of the subphylum Medusozoa, a major part of the phylum Cnidaria.
Jellyfish are mainly (2) ______ marine animals with umbrella-shaped bells and trailing
tentacles, although a few are anchored to the seabed by stalks rather than being mobile. The bell
can pulsate to provide (3) ______ for highly efficient locomotion. The tentacles are armed with
stinging cells and may be used to capture prey and disperse widely and enter a sedentary polyp
phase before reaching sexual maturity.
Jellyfish are found all over the world, from surface waters to the deep sea. Large, often colorful,
jellyfish are common in (4) ______ worldwide. Adjacent to jellyfish are eaten by humans in
certain cultures. They are considered a delicacy in some Asian countries, where species in the
Rhizostomae order are pressed and salted to remove excess water. Australian researchers have
described them as a "perfect food", sustainable, and (5) ______ but relatively low-calorie. They
are also used in research, where the green (6) ______ protein used by some species to cause
bioluminescence has been adapted as a fluorescent marker for genes inserted into other cells or
organisms.

1, A. natural B. colloquial C. informal D. easy-going

2, A. swimming-freely B. freedom-swim C. free-swimming D. unattached swimming

3, A. propulsion B. driving-force C. momentum D. impotence

4, A. seaside B. coastal zones C. the embankment D. marginal areas

5, A. high in protein B. abundant-protein C. high-protein D. protein-rich

6, A. candescent B. fluorescent C. luminous D. phosphorescent


TRANSFORMATION:

1, The European Space Agency’s Rosetta mission aims to land a probe on a comet.
(FOR)
→ The aim of the European Space Agency’s Rosetta mission is ………..…….
…………… on a comet.

2, The authorities believe there will not be any disturbances at the protest (EXPECTED)
→ The protest ………..…….………………..…….……………………..…….
…………… peaceful.

3, The manager told the press that his team had played very badly. (RECORD)
→ The manager ………..…….………………..……… that his team had played very
badly.

4, My grandmother would always make us eat everything on our plates when we visited
her (WASTE)
→ Nothing we were given to eat …………..…….…………… at my grandmother’s
house.

5, Paul couldn’t stop thinking about the argument he had had with his brother.
(OVER)
→ Paul ………..……….….……..……… mind the argument he had had with his brother.

Making a faux pas


I was staying recently in rather a smart hotel in Melbourne, Australia. No (0) SOONER
had I got into bed than I became aware of music coming from the room next to mine. I
knocked quite gently on the partition wall in (1) ………..……….…. to indicate that it was
unacceptably loud. I walked out into the corridor to get the room number (2) ………..
……….…. telephoning the culprit. A sleepy Australian voice replied and I told him in no
uncertain (3) ………..……….…. that his music was disgracefully loud. After ten minutes
the noise was (4) ………..……….…. better. As I appeared to be getting (5) ………..
……….…. fast, in desperation I telephoned hotel security. Eventually the hotel security
men (6) ………..……...….…. up. They entered my room, walked to the side of the bed
and pressed a knob. There was silence. (7) ………..……….…. the time it had been
nothing (8) ………..……...….…. than my own radio with its loudspeaker by the wall
which had been the cause of the trouble.
1, Dr. Harry Harlow, a psychologist, was quite an unsympathetic person and is famous for
his Monkey Love psychological experiments. (SYMPATHY)
2, Almost like a training ground for young people to learn connoisseurship of anything
and I'm a gastronomy student and we certainly talk about tasting a lot.
(CONNOISSEUR)
3, And with that, let's take a look at the suspects with the most circumstantial evidence
against them. (CIRCUMSTANCE)
4, The beginning of the Egyptian year coincides with the reappearance of a particular star
called Sirius, following a long period of invisibility. (APPEAR)
5, He was watching for how often one person would say something seemingly
inconsequential like his weirdest feeling we're being watched right now.
(CONSEQUENT)
6, Please just get out of denial and go looking for disconfirming data that will prove that
in fact, your old stereotypes are wrong. (CONFIRM)
7, In a solar cell, crystalline silicon is sandwiched between conductive layers because
each silicon atom is connected to its neighbors by four strong bonds. (CRYSTAL)
8, The hard approach creates just complicatedness in the organization and for an
automotive company, the engineering is a five-dimensional matrix.
(COMPLICATE)
9, Florentine intellectuals began to spread the idea of reformulating classical Greek and
Roman works while placing greater value on individual creativity.
(FORMULATE)
10, But I was so utterly unqualified for this project and so utterly ridiculous and ignored
the brief so desperately that I think they just embrace it with wholeheartedness.
(WHOLEHEARTED)
11, Its membership includes authoritarian governments with unambiguous and abhorrent
human rights records such as China, Cuba, and Venezuela.
(AMBIGUITY)
KEY:
1, The aim of the European Space Agency’s Rosetta mission is for a probe to be
landed on a comet.
2, The protest is expected to be peaceful.
3, The manager went on record as saying that his team had played very badly.
4, Nothing we were given to eat was (ever) allowed to go to waste at my
grandmother’s house.
• For each space, choose ONE word which you think best completes the sentence. Look
carefully at the words both before and after each space. There is an example at the
beginning (0).

are
The British (0) ___________ a pessimistic people as a (1) ___________ . When we were (2)
___________ awarded the honor of hosting the 2012 Olympic Games, an opinion survey at the time found that
55 percent of us were more (3) ___________ about the potential impact on the transportation network during
the Games than with welcoming the advent of the greatest (4) ___________ on Earth.
Nonetheless, (5) ____________ this fervent pessimism, there is an uneasy sensation that we should be more
optimistic. We are always attempting to remove (6) ___________ other's pessimism. Try it out for yourself: sit
gloomily in a public place and see how long it (7) ___________ before a smiling passer-by says, 'Cheer up, it
might (8) ___________ happen!' or offers one of those trite aphorisms about 'looking on the bright side' or
'clouds (9) ___________ silver linings.
The self-help business makes billions of dollars by selling hope and positive thinking. But can having a
positive attitude truly make a difference in our lives? How (10) ___________ optimism improve people's
trustworthiness or make sporting events more successful? Professor Elaine Fox, a neuroscientist (11)
____________ just published Rainy Brain, Sunny Brain, a book exploring (12) ____________ mixed
sentiments of optimism and pessimism, argues it (13) ___________ .
She asserts that our negativity is a reasonable mind's response and our optimism is an illusion, and that for
(14) ___________ of us, they both serve to balance us out. 'Positivity is a mirage. Nonetheless, it is a helpful
misconception. We (15) ___________ get out of bed in the morning if we didn't have some hope. It is,
nevertheless, a helpful illusion. We wouldn't get out of bed in the morning if we didn't have some (16)
___________ of optimism. Pessimism, on the other hand, has its place,' she argues.

KEY:
1, whole 2, originally 3, concerned 4, show
5, alongside 6, each 7, takes 8, never
9, having 10, might / could 11, who / that 12, our
13, can't / cannot 14, most 15, wouldn't 16, sort
A BRIEF HISTORY OF

The mall
Standardized shopping malls have become the new Main Streets of the USA according to
one (1) ……….….……. on the subject. Along with antibiotics and personal computers,
the shopping mall has been (2) ……….….……. as one of the top 50 wonders that have
revolutionized the lives of people today. But shopping malls didn’t just happen out of the
blue. The mall was originally (3) ……….….……. of as a community center where
people would (4) ……….….……. for shopping, cultural activity, and social interaction.
It is safe to (5) ……….….……. that the mall has achieved and surpassed those early
expectations. The first enclosed mall was developed in a suburb of Minneapolis in 1956
and was designed to get the shopper out of the harsh weather. The phenomenal growth
and development of shopping centers were a natural (6) ……….….……. of the migration
of the population out from the cities and (7) ……….….……. the growth of the use of the
automobile. The success and impact of the shopping mall may have something to do with
their potential to (8) ……….….……. community life.
1, A. scholar B. master C. body D. authority
2, A. put forward B. set up C. picked up D. brought out
3, A. imagined B. devised C. conceived D. formulated
4, A. cluster B. converge C. group D. rally
5, A. say B. tell C. speak D. remark
6, A. sequel B. upshot C. follow-up D. consequence
7, A. corresponded B. paralleled C. correlated D. equated
8, A. raise B. magnify C. enhance D. heighten

3. CONDITIONAL SENTENCES:
TYPE 1,2:
1, He was finally diagnosed as insane and remained in an institution until his death.
→ If he was diagnosed ………..…….……………………..…….…………………….
……………..…
2, I’m a bit of a workaholic and so actually finding someone to have a relationship with
was always tricky.
→ If I ………..…….……………………………………………..…….
…………………….……………..…
3, I don't like the new research assistant because he is always peering.
→ If ………..…….……………………………………………..…….…………………….
……………..…
4, Astronomers say that there is dark matter in the universe, but they haven’t been able to
detect this as yet.
→ If ………..…….……………………………………………..…….…………………….
……………..…
5, Due to recent global restructuring, they will no longer manufacture the car in Europe.
→ If ………..…….……………………………………………..…….…………………….
……………..…
6, Henry didn't use to be so difficult to work with, but he's been under a lot of pressure.
→ If ………..…….……………………………………………..…….…………………….
……………..…
7, If tigers disappear, there will be far-reaching and negative consequences for other parts
of the ecosystem.
→ Unless ………...……………………………………………..…….…………………….
……………..…

1, People after the earthquake in New Zealand visited us to incorporate some of this public-
spiritedness around local growing into the heart of Christchurch.
2, Jackson owned the preservation of his throne to the miraculous interposition of a stranger
who suddenly appeared in his realm.
3, They can be spotted tucking their heads beneath them for warmth, flashing them at
prospective mates, sheltering eggs with them.
4, The baroque theater was all about melodrama - lighting, perspective, and sound effects
were all melodramatic.
5, And this makes the moving around the tonic arpeggio in the final bars sound just as a
prolongation of this final tonic note in the melody.
6, This practical expedient if it is carried forward as proposed could be used under either the
transition method under the full retrospective.
7, For our atmosphere to absorb any kind of radiation, it needs to have some electrically
charged particles for passing electromagnetic waves to push around.
8, This allows the owner to excuse themselves to, go and deal with their dog, extricating
themselves from the problematic situation.
9, When matter and antimatter meet, they annihilate each other in a huge release of energy.
Because these antimatter-matter reactions are so powerful.
10, When matter and antimatter meet, they annihilate each other in a huge release of energy.
Because these antimatter-matter reactions are so powerful.

1, A decline in honeybee populations has been recorded in recent years. The decline of
honeybee populations was significantly lower over the 2013–2014 winter.
→ The decline of honeybee populations that has been recorded in recent years was
significantly lower over the 2013–2014 winter.
2, After an earthquake rocked Washington, D.C., in August 2011, the Washington Monument
was closed for repairs. The Washington Monument honors the first U.S. president.
→ After an earthquake rocked Washington, D.C., in August 2011, the Washington
Monument, which honors the first U.S. president, was closed for repairs.
• For each space, choose ONE word which you think best completes the sentence.
Look carefully at the words both before and after each space.
In those days the council houses stretched (0) ALL over the western side of the city:
row after (1) ___________ of huddled, dingy dwellings in orange half-brick or pale
white stucco. In summer the chemicals (2) ___________ the May and Baker factory two
miles (3) ___________ came and hung (4) ___________ the doors and gardens with an
indescribable smell of sulfur, and the most common sight in that part of Norwich (5)
___________ in the morning was a paperboy wrinkling his nose in disgust as he
negotiated somebody’s front path.
Most of this early life I (6) ___________ forgotten. But there is a memory (7)
____________ sitting, or perhaps balancing, at any rate precariously, on some vantage
point (8) ___________ an upstairs window and looking at the houses as they faded
away into the distance. (9) ___________ on, there are other phantoms - faces that I
can’t put names to, my mother, ironing towels in the (10) ____________ room of a
house that I don’t think was ours, snow falling (11) ___________ the turrets of the great
mansion at Earlham.

1, row 2, from 3, away 4, around 5, early 7, of 8, near 9, Later 10, back 11,
over 6, have
Chimpanzees and you

Some may be elated to hear this and others a tad uneasy, but the DNA
of chimpanzees and humans differs by only around one percent,
making them more similar to us than even gorillas. While they don't
conduct themselves entirely like humans, they exhibit behaviors and
emotions originally thought only to be expressed by humans. They
have a strong sense of community and are rather jubilant when a
human they know enters the room, as can be deduced from their
jumping up and down. Their facial musculature resembles ours very
closely, as is witnessed when they smile, express worry, or vent their
anger. While we and chimpanzees may be on the same wavelength in
son respects, there are huge differences. Even the most talented
chimpanzee is no match for human intelligence and there isn't even a
remote possibility that they will speak ─ they lack the vocal tracts
necessary for language.

THE NEED TO SAVE THE TIGER


Tigers are an important part of the planet’s rich
diversity of life. As top predators in their food
chain, they feed on a variety of prey species and
help maintain the structure and functioning of the
ecosystems they inhabit. Tigers, therefore, are
considered a keystone species. If tigers disappear,
there will be far-reaching and negative
consequences for other parts of the ecosystem.
Protecting tigers, therefore, helps many other
species as well. For example, protecting tiger
habitats in India and Bangladesh has helped to
protect human habitats by reducing land erosion, stabilizing ecosystems, and encouraging wise
land use. It has also led to an increase in global awareness of climate change and the problem of
rising sea levels.
• Complete the sentences with the words and phrases in the box. You
don’t need to change the form of the words and phrases.
(cockpit motorists / overtaking runway/ commuter diesel/ jet lag departure
lounge)
1, The _______________ at Qamdo Bamda Airport in Tibet is 5.5 kilometers long; it
needs to be this length to allow planes to accelerate enough to take off at high altitude.
2, A ______________ is the area of a plane where the pilot and co-pilot sit, and is also
known as the flight deck.
3, It is estimated that there will be about 2 billion ______________ in the world by 2035.

4, Research shows the average American ______________ spends 142 hours every year
stuck in traffic.
5, Vehicles running on ______________ are believed to be more harmful to the
environment and members of the public than petrol vehicles.
6, Dangerous ___________ is the most common cause of car accidents worldwide.

7, The first ever beach ________________ has been planned in Barbados, where
passengers can check in, pick up their boarding passes and relax until their flight.
The book includes contributions from 12 other writers - to name them all would take too much space, and to name a few would be
invidious. Suffice it to say that this is a pleasing aspect of the book in that it adds diversity of voice. Most of the authors are European-
based, and, combined with the non-managerial focus, this seems to make the book less dogmatic or prescriptive. The book is
immensely readable, both because of the writing style(s) and the variety of interrelated topics. It can also act as a useful resource or
reference to find a quick way into topics, such as value, or identity. It lends itself to a course design that would require students to
engage with the journal literature and relate this to contemporary social issues. It is a useful resource for the kind of class where ideas
can be discussed and argued about. Its treatment of issues is helpful, as it brings several different ideas to bear on the same question.
At times, the book strays from its mission and spends a little too much time giving the managerial view. However, the overall de-
privileging of the marketing manager is very welcome indeed - though whether the consumer's viewpoint is the only appropriate
alternative perspective to work from is a matter for debate. (1)

1, In the Netherlands, the International Criminal Court sentenced him to 30 years


imprisonment after he was convicted of crimes including murder, rape, and sexual slavery.
(PRISON)
• Read the following passage and mark the letter (A, B, C or D) on your answer
sheet to indicate the correct answer to each of the questions.
The functioning of the art world is dependent on art institutions, ranging from major museums to
private galleries, non-profit spaces, art schools and publishers, and the practices of individual artists,
curators, writers, collectors, and philanthropists. A major division in the art world is between the for-
profit and non-profit sectors, although in recent years the boundaries between for-profit private and
non-profit public institutions have become increasingly blurred. Most well-known contemporary art is
exhibited by professional artists at commercial contemporary art galleries, private collectors, art
auctions, corporations, publicly funded arts organizations, contemporary art museums, or by artists
themselves in artist-run spaces. Contemporary artists are supported by grants, awards, and prizes as
well as by direct sales of their work. Career artists train at art school or emerge from other fields.

There are close relationships between publicly funded contemporary art organizations and the
commercial sector. For instance, in 2005 the book Understanding International Art Markets and
Management reported that in Britain a handful of dealers represented the artists featured in leading
publicly funded contemporary art museums. Commercial organizations include galleries and art fairs.

Corporations have also integrated themselves into the contemporary art world, exhibiting
contemporary art within their premises, organizing and sponsoring contemporary art awards, and
building up extensive corporate collections. Corporate advertisers frequently use the prestige associated
with contemporary art and coolhunting to draw the attention of consumers to luxury goods.

The institutions of art have been criticized for regulating what is designated as contemporary art.
Outsider art, for instance, is contemporary art, in that it is produced in the present day. However, one
critic has argued it is not considered so because the artists are self-taught and are thus assumed to be
working outside of an art historical context. Craft activities, such as textile design, are also excluded
from the realm of contemporary art, despite large audiences for exhibitions. Art critic Peter Timms has
said that attention is drawn to the way that craft objects must subscribe to particular values to be
admitted to the realm of contemporary art. "A ceramic object that is intended as a subversive comment
on the nature of beauty is more likely to fit the definition of contemporary art than one that is simply
beautiful."

At any one time, a particular place or group of artists can have a strong influence on subsequent
contemporary art. For instance, The Ferus Gallery was a commercial gallery in Los Angeles and re-
invigorated the Californian contemporary art scene in the late fifties and the sixties. Contemporary art
can sometimes seem at probabilities with a public that does not feel that art and its institutions share its
values.

In Britain, in the 1990s, contemporary art became a part of popular culture, with artists becoming stars,
but this did not lead to a hoped-for "cultural utopia". Some critics like Julian Spalding and Donald
Kuspit have suggested that skepticism, even rejection, is a legitimate and reasonable response to much
contemporary art.
B2: FIRST
Use the word given in capitals at the end of some of the lines to form a word that
fits in the space in the same line. There is an example at the beginning (0).
The book includes (0) contributions (CONTRIBUTE) from 12 other writers - to name them
all would take too much space, and to name a few would be ____________ (INVIDIOUSLY).
Suffice it to say that this is a pleasing aspect of the book in that it adds ____________
(DIVERSE) of voice. Most of the authors are European-based, and, combined with the non-
managerial focus, this seems to make the book less dogmatic or ___________ (PRESCRIBE).
The book is __________ (IMMENSE) readable, both because of the writing style(s) and the
variety of interrelated topics. It can also act as a useful resource or ___________ (REFER) to
find a quick way into topics, such as value, or identity. It lends itself to a course design that
would require students to engage with the journal literature and relate this to _______________
(CONTEMPORIZE) social issues. It is a useful resource for the kind of class where ideas can
be discussed and argued about. Its ___________ (ENTREAT) of issues is helpful, as it brings
several different ideas to bear on the same question. At times, the book strays from its mission
and spends a little too much time giving the ___________ (MANAGE) view. However, the
overall de-privileging of the marketing manager is very welcome indeed - though whether the
consumer's viewpoint is the only appropriate ___________ (ALTERNATE) perspective to
work from is a matter for debate.

ANSWER SECTION:
1, ______________ 3, ______________ 5, ______________
2, ______________ 4, ______________ 6, ______________
7, ______________ 8, ______________ 9, ______________

C1: ADVANCED
Use the word given in capitals at the end of some of the lines to form a word that
fits in the space in the same line. There is an example at the beginning (0).
The hypothalamus is a part of the brain that controls ___________ (BODY) functions such as
hunger, sleep, and body temperature. It tells the US when there is a ___________
(BALANCE) in our bodies, for instance, it indicates that we're becoming ___________
(HYDRATE) by making the US feel ___________ (THIRST) when we don't drink enough
water. It is located in the center of our brains and is about the size of an almond. The
hypothalamus works with the body's nervous system and ___________ (INSIDE) organs to
interpret signals that come from all areas of the body. It has a way of telling us when we're
___________ (NOURISH) that we need to change our diets. The hypothalamus also controls
behavioral responses to emotions. It explains why, for example, an actor who has a stage
___________ (FRIGHTEN) can't go out on stage, or why when we see sad films that pull on
our ___________ (HEART), we tend to cry. Poor nutrition can cause the hypothalamus to
suffer from ___________ (ORDER) and if left unchecked, these problems can lead to changes
in weight, fatigue, ___________ (HEAD), and hair loss.

ANSWER SECTION:
1, ______________ 4, ______________
2, ______________ 5, ______________
3, ______________ 6, ______________
7, ______________ 9, ______________
8, ______________ 10,______________

C2: PROFICIENCY
Use the word given in capitals at the end of some of the lines to form a word that
fits in the space in the same line. There is an example at the beginning (0).
1, Cheyenne is an unpredictable and volatile player and anything could happen as far as her
future in the game of golf is concerned. (PREDICT)

2, The fact that some politicians tried to defraud the state by claiming more expenses than they
were due is evidence that corruption is widespread in our society. (POLITICIZE)

3, Statistics from the forces of law and order frequently tell a different story, but these tend to
be dismissed as untrustworthy. (TRUST)

4, We learn about politics because knowledge is our best defense against unscrupulous people
who will use our ignorance. (SCRUPLE)

5, There was a time when it was commonplace for grandparents to live with and be looked
after by the rest of the family. (COMMON)

6, For the enthusiastic hill walker, there is a challenging 25-kilometer trail around the hill
called The Cell, which takes you on a meander through woodland. (ENTHUSE)

7, Mineral means solid homogeneous inorganic substances occurring in nature having a definite
chemical composition. (HOMOGENIZE)

8, There are only two events that are up there in terms of the most catastrophic and that's a
terrorist event or an influenza pandemic. (CATASTROPHE)

9, And between intimidation and emerging discriminatory voting laws, fewer black men voted,
which allowed white Democrats to take control of state governments in the south.
(DISCRIMINATE)
A2: PRE-INTERMEDIATE
• Use the word given in capitals at the end of some of the lines to form a word
that fits in the space in the same line. There is an example at the beginning (0).

DINOSAURS
Nobody has ever __________ (SEE) a dinosaur. The last dinosaur died about 60
million years ago, a __________ (LENGTH) time before there were any people on
the earth. Nobody knows for __________ (SURENESS) why they all died. The
nearest living relatives of dinosaurs are birds.
Dinosaurs didn't all look the ____________ (SAMELY). There were more than 5000
__________ (KINDNESS). Some were very small, but others were giants. The
_____________ (LARGE) ones were bigger than any other animals ever lived on land.
The Brontosaurus, for example, was twenty __________ (METRIC) long, and it ate
plants. The Tyrannosaurus Rex was not as big, but it was stronger. It had sharp
__________ (TOOTHY) for eating meat, and it could also run __________
(FASTLY) because it had long back legs.

1, seen 2, long 3, sure 4, same 5, kinds 6, largest 7, meters 8, teeth 9, fast

B1: INTERMEDIATE
• Use the word given in capitals at the end of some of the lines to form a word
that fits in the space in the same line. There is an example at the beginning (0).
SNAILS
Some people keep snails as pets in __________ (AQUA) together with their fish, but in
some countries, people like eating them for dinner. They are very tasty. When you
keep them as pets, you have to make sure to control the number because snails
__________ (PRODUCT) very quickly. Snails can be found in gardens, in ponds, in
rivers, and even in the sea, but where they don't live is where it is hot and __________
(DRILY) because they need wet places to be happy.
Most snails have a long tongue that has thousands of tiny teeth. The teeth cut the food
they eat into tiny pieces. Snails live on plants and vegetables and that's why
__________ (GARDEN) hate them because they eat everything in the garden. When a
snail is disturbed, it simply pulls itself back into its shell. The snail also retreats into its
shell and seals the __________ (ENTER) in dry weather to protect its body from drying
up.
You don't usually see snails on __________ (SUN) days because they are most
__________ (ACT) at night and on cloudy days. They don't like the sunshine very
much. During very cold weather or winter, they hide in small __________ (HOLING) in
the ground. The body of the snail is long, moist, and __________ (SLIME). It has a shell
to protect its soft body. Snails have different shaped shells, and they can live up to 5 to
10 years.
1, aquariums 2, reproduce 3, dry 4, gardeners 5, entrance 6, sunny 7, active 8,
holes 9, slimy

B2: UPPER-INTERMEDIATE
• Use the word given in capitals at the end of some of the lines to form a word
that fits in the space in the same line. There is an example at the beginning (0).

DON’T CATCH COLD


According to recent (0) RESEARCH, at least some of the advice SEARCH
about health that was passed down to our grandparents by
previous generations may actually contain some (17) ___________. TRUE
A good example is the (18) ___________ between feeling cold and RELATION
catching a cold. My grandmother was convinced that being
outside when the temperature is low, or simply dressing in SUIT
(19) _____________ clothes for the cold weather, was a sure way of
catching a cold. Now, as we know, colds are caused by viruses.
Therefore, in the (20) ___________ of a virus, you cannot catch a cold ABSENT
- no matter how low the temperature or how (21) ___________ your APPROPRIAT
clothing. E
But (22) ___________ now think that we may have viruses in our
bodies already, which are just waiting for the chance to turn into SCIENCE
a cold. In cold weather, for example, blood vessels in the nose get
smaller to stop heat escaping. (23) ___________, this also allows the FORTUNATE
cold virus to attack the nose or throat more (24) ___________. So,
EASY
perhaps Granny was right!

1, truth 2, relationship 3, unsuitable 4, absence 5, inappropriate 6, scientists 7,


unfortunately 8, easily
• Read the following passage and decide which answer (A, B, C or D) best fits each gap:
Sharks were first discovered during the Ordovician period, 450–420 million years ago, when terrestrial vertebrates
and a variety of plants inhabited the continents. Only scales have been obtained from the early sharks, and not all
paleontologists believe that they are from real sharks, with some suspecting that they are from the thecodont agnathans.
The earliest generally accepted shark scales date back to at least the Silurian period, some 420 million years ago. The
original sharks did not resemble contemporary sharks in whatsoever way.
At the moment, the most frequent shark tooth is the cladding, a thin tooth with three tines resembling a trident,
perhaps to assist in the capture of fish. The majority of modern sharks originate from about 100 million years ago. Most
fossils are teeth, which are typically found in enormous quantities. There have been partial skeletons and even full
petrified remains unearthed. Sharks are thought to produce tens of thousands of teeth over their lifespan, which explains
the abundance of fossils. The teeth are composed of apatite, a kind of calcium phosphate that is easily fossilized. When a
shark dies, its skeleton disintegrates, dispersing the apatite prisms. Rapid burial in bottom sediments is required for
preservation.

1, A. which B. whose C. when D. where


2, A. distrusting B. suspecting C. wondering D. mistrusting
3, A. typically B. commonly C. constantly D. especially
4, A. have been B. were C. will be D. were being
5, A. some kind of B. any sort of C. a kind of D. catch sight of
• Complete the sentences with the verb (A, B, C or D) which best fits each gap:
1, Would anyone else like to ............................ their opinion?
A. make B. take C. tell D. give
2, The school is trying to ............................ participation in extra-curricular activities like the chess
team.
A. gain B. increase C. put up D. access
3, Did you manage to ........................ sight of the new sports car at the motor exhibition over the
weekend?
A. catch B. find C. see D. discover
4, There has been little debate as to .............................. it actually makes ecological sense to ban single-
use plastic products.
A. in case B. even if C. whether D. though
5, John was very upset to ............................ last in the 400 metres race despite all the hard training.
A. come B. end C. go D. become
6, After ............................. her bus for the third time that week, Elena knew she was going to be in
trouble.
A. missing B. losing C. failing D. forgetting
7, Hayley was excited because she had just one more week before she could ............................. on
holiday.
A. get B. become C. go D. catch
8, Kerim .............................. a table for four at his favourite restaurant online.
A. commanded B. asked C. demanded D. booked
9, It’s unfortunately easy ….......................... to foresee what will happen if we continue producing
plastic products at the same rate as today.
A. enough B. plenty C. adequate D. not much
10, If you look at professional golfers or tennis players, you will observe that
practice ............................... the basis of their preparation for tournaments.
A. lays B. composes C. enacts D. forms
11, Increased internet access enables individuals to share photos, opinions and even live videos with one
sharp ........................... on the screen.
A. stroke B. tap C. squeeze D. nudge
12, If your opponent keeps winning points by serving high to your backhand, there’s no place to hide;
you either have to ............................ the weakness, or you’ll keep losing to him.
A. alter B. square C. reform D. rectify
13, The children even bring the bill at the end of the meal, all with a little help from
the .............................. staff of course.
A. rising B. increased C. grown-up D. expanding
14, Usually poor service comes ................................. to the behaviour of an individual employee, but
it’s also the restaurant’s responsibility.
A. over B. along C. up D. down
C1: ADVANCED
• Use the word given in capitals at the end of some of the lines to form a word
that fits in the space in the same line. There is an example at the beginning (0).

Hollywood takes on Bollywood


As well as being the wealthiest and most (0) POPULOUS city in POPULATE
India, with over 20 million inhabitants, Mumbai (formerly
Bombay) is home to India's (1) _____________ prolific Hindi- CREDIBLE
language film industry. This is no cottage industry. Every year,
Bollywood - the word is a portmanteau of Bombay and
Hollywood - produces (2) _______________ twice the number of ROUGH
feature films that are made in Hollywood. Those Hindi films are
watched by an audience of 3 billion people worldwide, whereas
the (3) _______________ audience for Hollywood films is only about TOTALITY
2.6 billion. However, the (4) ______________ truth behind those SURPRISE
(5) _____________ is that 80% of the revenue from Bollywood films STATISTICIA
comes from inside India, while 50% of the money made from N
Hollywood films comes from outside the US, but almost none of it
from India.

Anyone who has watched both a (6) _____________ Bollywood film TRADITION
and a Hollywood blockbuster will perhaps understand why there
is so little (7) _____________ between the audiences for these films.
They are simply worlds apart. From the (8) _____________ of an CROSS
PERSPECT
audience used to watching Hollywood blockbusters, Bollywood
films are (9) _______________ and impossible to categorise. They
PUZZLE
follow conventions that are simply not reflected in American
films.

Bollywood films are always very long (three hours is normal) and
therefore they have an (10) ________________. They often combine INTERMIT
(11) ______________ of what a western audience would consider ELEMENTAR
different genres within the same film. For example, whatever the Y
storyline and wherever it takes place, a Bollywood film will
almost (12) ______________ contain scenes with singing and dancing.
Those scenes will involve lots of extras in costume and often be VARIATE
filmed in the Swiss Alps, even though the main story probably
takes place in an Indian city.
1, populous 2, incredibly 3, roughly 4a, total 4b, surprising 5, statistics 6,
traditional 7, crossover 8, perspective 9, puzzling 10, intermission 11, elements
12, invariably

WORDFORM

It is the job of the government to prevent a (1) _____________ of the TRAGIC


commons. That includes the commons of shared values and norms on
which a (2) _____________ depends. The dominant digital platform DEMOCRAT
companies, including Facebook and Google, make their profits using
business models that erode this commons. They have created a haven
for dangerous (3) _____________ and hate speech that has INFORM
undermined trust in democratic (4) _____________. And it is INSTITUTE
troubling when so much information is controlled by so few
companies.
What is the best way to protect and restore this public commons?
Most of the proposals to change platform companies rely on either
antitrust law or (5) _____________ action. I propose a different REGULATE
solution. Instead of banning the current business model — in which
platform companies harvest user information to sell targeted digital
ads — new (6) ____________ could establish a tax that would (7) LEGISLATE
_____________ platform companies to shift toward a healthier, more COURAGEOUS
traditional model.
The tax that I propose would be applied to revenue from sales of
targeted digital ads, which are the key to the (8) ______________ of OPERATIVE
Facebook, Google, and the like. At the (9) ______________ level, FEDERALIZE
(10) ________________ could add it as a surcharge to the CONGRESSION
corporate income tax. At the state level, a (11) ______________ AL
could affirm it as a type of sales tax on the revenue a company LEGISLATE
collects for displaying ads to residents of the state.

KEY:
1, tragedy 2, democracy 3, misinformation 4, institutions 5, regulatory 6, legislation
7, encourage 8, operation 9, federal 10, congress 11, legislature
Respiratory stimulants (analeptic drugs) have a limited place in the treatment of ventilatory
failure in patients with chronic obstructive pulmonary disease. They are effective only when
given by intravenous injection or infusion and have a short duration of action. Their use has
largely been replaced by ventilatory support. However, occasionally when ventilatory support is
contra-indicated and in patients with hypercapnic respiratory failure who are becoming drowsy
or comatose, respiratory stimulants in the short term may arouse patients sufficiently to co-
operate and clear their secretions. Respiratory stimulants can also be harmful in respiratory
failure since they stimulate non-respiratory as well as respiratory muscles. They should only be
given under expert supervision in hospitals and must be combined with active physiotherapy. At
present, there is no oral respiratory stimulant available for the long term.

Many people are hampered by what they think of as a fit and proper subject for the camera. But,
it can be healthy for your creativity to apply the same kind of attention and effort you might to a
so-called ‘important’ subject or spectacle, to instead, the mundane stuff that most people ignore.
The idea of finding the extraordinary simply by giving the ordinary your full attention began
with surrealist painters in the 1920s.
This has even more resonance now, with the onset of globalization, which may homogenize
things in one way, but also means that the remaining differences between cultures are often
found in the simplest things. A henna stencil used as body decoration for special occasions,
maybe unremarkable in Asia, but in the West, it makes an image oddly intriguing. Even so, as
has always been true, it’s not enough just to find something and make a snapshot of it. For the
photography of the ordinary to work, it’s more important than ever to give the image your full
imaginative and skillful treatment. The best photographers often do this instinctively.
• For questions 9 - 16, read the text below and think of the word which best fits each
space. Use only one word in each space. There is an example at the beginning (0).

THE ORIGINAL OF COINS


According (0) to the Greek historian Herodotus (484-425 BC), the Lydian people were the first
to use metallic coins. In fact, these earliest coins were made out (9) _________ electrum, a
naturally occurring mixture of gold and Silver. The coins were first produced in the seventh
century BC with a design on (10) _________ side only; the other was marked with simple
punches. Each coin was assigned a value in units. Some coins were inscribed with names in
Lydian script, but it is unclear (11) _________ these are names of kings or just of rich men who
produced the coins.
(12) _________ of the Irregular size and shape of the coins, it must (13) _________ been
difficult to tell one (14) _________ another, especially some of the smaller ones. Thus, many
costs were expressed (15) ___________ terms of the total weight of the coins required, and
transactions were completed by weighing the coins used together, (16) _________ than
counting individual ones.

Answer: ,9, of 10, one 11, whether/if 12, Because 13, have 14, from 15, in
16, rather

It appears to be a prerequisite (1) ____________ for screen villains to love classical music, and
magnificent examples abound. Take, for instance, Malcolm McDowell's insistence (2)
____________ listening to Beethoven while he kills people in A Clockwork Orange. Glenn
Close's thirst (3) ____________ revenge is fuelled by music from Madame Butterfly in Fatal
Attraction, while the gangster AI Capone finds reason (4) ____________ shed a tear when
listening to Leoncavallo's Pagliacci in The Untouchables.
Hollywood's interpretation (5) ____________ evil genius comprises villains who are highly
intelligent, often slightly aristocratic beings, and their sensitive tastes (6) ____________ art and
music are placed in sharp contrast (7) ____________ the crude brutality of their acts of
violence. Hans Zimmer's score for Hannibal enhances the dramatic effect of Or Lecter's talent
(8) ____________ displaying a veneer of refined elegance just before he murders his victims.
Perhaps, more than any other kind of music, the range and complexity of classical music reflect
the tangled psychological workings of the criminal mind.
• For questions 1 - 8, read the text below and think of the word which best fits
each space. Use only one word in each space. There is an example at the
beginning (0).
WHY CLIMB MOUNTAINS?
There’s (0) NOTHING more likely to irritate a mountaineer or explorer than to ask them
why they do it, or why they are so willing to put (9) ____________ with danger and
discomfort. In 1924 when George Mallory was asked why he wanted to climb Mount
Everest, he replied: ‘Because It’s there.’ It may be that having been asked the same question
several hundred times, Mallory just didn’t care anymore and this was the first phrase to (10)
_______________ into his head. Then again, for (11) _____________ we know, it was
simply his way of saying, ‘Why not?’

This might seem self-evident (12) ____________ someone like Mallory. You climb Everest
because you can. One way to look at people like mountaineers or explorers, or successful
ones, at any rate, is to see them (13) ____________ people who have realized what they are
good at. When you read their books, more often than (14) _____________ they will come
across as people who are (15) ____________ ease with their environment, (16)
_____________ alien it might seem to an outsider.

Answer: 1, up 2, come/pop 3, all 4, to 5, as 6, not 7, at 8, however


The expression of diametrically opposed opinions has a
lways been a great tool in the arsenal of journalists, especially film critics.
Are you frowning as you read this? Habitual frowners may not even be aware that their foreheads are
creased and will need to touch their brows to find out. A permanent frown is forbidding and
unattractive, yet it is very easy to go into the habit of frowning. You can stop yourself by placing your
hand on your forehead to check whether your brow is smooth whenever you happen to be reading or
watching television. In this way, you can begin to unlearn a negative piece of body language – and if
you suffer from headaches, you should find yourself suffering from them much less.
Smiling at yourself may make you fed a little self-conscious – but it works! Next time you are under the
weather, physically or emotionally, you can test for yourself the therapeutic powers of smiling. Each
time the expression fades from your face, try again and again until you begin to notice an improvement
in yourself. In a large number of cases, this simple technique will produce noticeable benefits within a
short space of time – and it's free.
Horses and humans have been partners for a very long time. It is true that many people love
horses because they are so soft and lovable. They have also been used as slaves for a long time,
both to (1) ____________ wagons or other things, or to carry people on their backs wherever
they wanted to go. That was (2) ____________ for the civilizations in the east and the west.
Horses were just the most (3) ___________ way to get around. There are three (4) ___________
types of horse breeds. Hot drops of blood are (5) ____________ horses that are just there for
speed and racing. Cold blood is generally bred for (6) ___________ and heavy work.
Warmbloods are a combination of the other two types and are often used for riding
competitions. Horses are animals that graze and mostly eat hay and grass. They also like peas
and beans and fruit, like apples. Now people don't use horses for transport anymore, but they are
an important (7) _____________ of sport. In that context, they are used for running and
jumping, and so are very well (8) ___________ after.

• For each space, choose ONE word which you think best completes the sentence.
Look carefully at the words both before and after each space.

WORK AND EDUCATION


In the 19th (1) __________, children (2) __________ poor families were expected to
contribute to the family from an early age. However, various measures (3) ___________
then have gradually marked out childhood as a distinct phase of life. For (4)
___________, the school-leaving age was gradually raised. Consequently, school work
(5) __________ replaced paid work, and the period of children's total dependency on (6)
__________ parents has correspondingly expanded. Furthermore, until relatively
recently, children had expected physical punishment for disobedience at school. If current
trends continue, (7) ________ a very short time, most countries will have outlawed this
kind of punishment, even in the home.

Answer: 1, century 2, from 3, since 4, example 5, has 6, their 7, within


BUSY, BUSY
With obesity having shot up (1) __________ the globe to dangerously high levels in (2)
_________ years, it is little wonder that people have (3) ________ to ask why. True, diets
have changed; we all know (4) _________ we live in a McWorld, hunting and gathering our
food from fast-food outlets and supermarket aisles, but it can't all be (5) ________ to diet,
(6) ________ ?
Technology has changed modern life to (7) _________ an extent that few aspects of life
today bear any resemblance to lives only a couple of generations ago. Just taking jobs as an
(8) ___________, how many of us today spend twelve hours a day on our feet physically
slogging ourselves into the ground? Or how many families could you (9) __________ living
without a car? Kids walking to school, parents going to half a dozen local shops, on foot, to
buy the week's food, family holidays by bus to the nearest seaside town. Take Tina Jameson,
a mother of two (10) _________ has to juggle home and a (11) _________ job. She says 'I
haven’t got time to walk anywhere. But I'd have even less time (12) _________ a washing
machine or dishwasher.’

Answer: 1, across 2, recent 3, started 4, that 5, down 6, can it 7, such 8,


example 9, imagine 10, who 11, part-time 12, without

In rural Valparai, India, technology is helping protect people from roaming elephants. Cell
phones are common here, but so are Asian elephants. Now the former is being used to keep people
safe from the latter. Thousands of people in Valparai work on tea and coffee plantations that are
situated between protected areas. These plantations are passageways for elephants moving from one
protected habitat to another. The people who live in the region are scattered throughout the area.
Since 1994, more than 40 people have been killed here in encounters with elephants. Research has
shown that such deaths are preventable if people have sufficient warning. That's where the Nature
Conservation Foundation's early warning system comes in. The system uses bulk text messages to
alert people who live and work in the area to the presence of elephants. If elephants have been
spotted in the vicinity, the locals receive a message by 5 p.m., advising them to be cautious, since
an accidental encounter with an elephant can be fatal. People's safety is of primary importance.
Since the introduction of the warning system, the death rate from elephant encounters has dropped,
allowing people and elephants to coexist.

I, WRITING
PART 1a: (17 points)
The two passages explore the concept of what brings happiness, but with contrasting views.
The first text states that happiness is something that can be bought. It suggests that material possessions
such as luxurious residences and expensive cars, as well as experiences such as frequent holidays, are
enough to make people happy. Furthermore, it states that if these have been acquired through hard
work, they are deserved rewards. In addition, it argues that if these trappings of wealth are a cause for
envy, then those who are envious should consider working hard themselves for the same benefits. As I
see it, not all wealth is accrued honestly or through hard work, and when it is flaunted as well, it
naturally causes ire. It should also be noted that not everyone has the opportunity to get ahead in life,
no matter how hard they work.
The second text expresses the view that people are increasingly equating wealth with happiness and
that this is flawed. It argues that a trouble-free life filled with leisure is no substitute for the things that
bring real happiness, the most important of which is friendship. The text notes that friends can be relied
on to give a hand whenever it is needed. It also claims that the wealthy can never truly know if their
friends like them for themselves or their money. I am inclined to agree with the views expressed in the
second text. It has enhanced obviously that greed is rampant, and that wealth is considered more
important than health or friendship, for example.
In conclusion, it seems to me that wealth can bring a measure of happiness, but it is no substitute for
friendship.

PART 1b: (18 points)


Nobody can repudiate that music features in our lives and yet, as the texts point out, it is undervalued in
society. The first text considers the plight of professional musicians, while the second focuses on the
low prestige of music in schools. This essay argues that music is important and requires investment to
secure its presence in education and culture.
What then is the role of music today? Professional musicians are sometimes highly regarded yet earn a
pittance. From the classical performer’s point of view, the reality is that it is very hard to find orchestral
positions nowadays. This must be disheartening after so many years of study. Perhaps more should be
done by the state to support young musicians who are embarking on their working life?
In the same way, on the question of teaching music in school, not enough is done. People tend to regard
music as a less essential part of the school curriculum than mathematics or science. At the same time,
there are unforeseen advantages for children who learn music from a young age, as it encourages
creativity and allows them to develop social skills through participation in a group event. This is not to
say that everyone should be able to read music, but they should have access to it.
Music plays a fundamental role in both the educational and cultural spheres, and its beneficial
contribution to shaping our lives needs to be better understood by all members of society. If more
resources were made available to schools and young musicians were given adequate financial support
through government subsidy or other means, society would profit significantly.

PART 2a: (18 points)

Long-distance Travel
The Scottish writer, Robert Louis Stevenson, used to say that he traveled ‘not to go anywhere but to
go.’ In other words, he loved traveling for his own sake. Although I enjoy being at a new destination, I
acknowledge that the journey there can be the most exciting and rewarding part of any holiday.
Some years ago I studied Russian at university and used to travel frequently to Russia. I usually went by
train, by far the cheapest way to get there then. This involved a three-day train journey across the
Channel and then over the North European Plain through Belgium, Germany, Poland, and Belarus to
Moscow. The train from Ostend was a Russian one and once onboard you felt as if you were already in
Russia, especially as you poured yourself a glass of tea from the samovar at the end of each carriage and
made yourself comfortable for the two nights you’d spend in your sleeping compartment.
I adored the sense of gradually making my way into a different world. Towns and villages became less
frequent and field turned into forests as we chugged eastwards. The children who waved at the train as it
passed began to have high Slav cheekbones. Traveling slowly gives you time to savor the gradual
changes, to think about where you’re going or where you’ve just been, to adapt to a new way of being.
This opportunity to reflect is immensely enriching.
These days it’s cheaper to hop on a plane than a train and you can now get to Russia in three hours
instead of three days. Lunch in London and dinner in Moscow have become possible thanks to the jet
engine. In many ways, the journey could be seen as easier now. But in my opinion, traveling by rail is
still much more satisfying, providing a real sense of the distance – both geographical and social –
between different countries.
Beauty written in the stars
Gazing at the smoky, glowing gas clouds of the ‘starburst’ galaxy shown here, we are savoring a
beauty that is the accidental product of events that happened in a distant time and part of the
universe. The result looks like a great painting. To be precise, it is reminiscent of the work of
Turner, that masterly nineteenth-century British artist. For John Ruskin, Turner’s champion and
near-contemporary, the object of art were to reveal the divine hand in nature. That was what he
meant by beauty. What might he have said today, having viewed the pictures taken by the
Hubble Space Telescope?

Having been launched in 1990 with an inaccurately ground lens, Hubble was initially a huge
embarrassment, sending back indistinct images that impressed no one. However, the picture
changed, quite literally, in 1993. On being successfully repaired by shuttle astronauts, the
telescope proceeded to relay the most spectacular images to us. Hubble is now like an eye with a
cataract removed, seeing into deep space with a hard, bright precision that is almost
uncomfortable.

There has never been a more gratuitous addition to our store of beauty. Ultimately, when we call
a work of art beautiful we are comparing it to nature. The underlying structures of nature
imitated by artists from ancient Greece until the middle of the twentieth century were chosen
because nature was perceived as beautiful. If we don’t talk much about ‘beauty in contemporary
art, it is because art is no longer concerned with the representation of nature. Arguably, the
photographs taken by Hubble are the most flamboyantly beautiful artworks of our time.
With inspiration from the romantic Saigon river and the symphonies of the Opera
House, The Opera Residence of The Metropole Thu Thiem super-project is designed
and constructed with luxury and sophistication... As you make an entrance to the
pedestrian bridge connecting District 1 and Thu Thiem, enrapture the fresh and airy
air from the Saigon River, you will experience the full magnificence and attraction
to welcome The Opera Residence from many different perspectives.
IV. Look at the signs. Choose the best answer (A, B, C, or D) for questions 15 and 16. (0.5pt)
15. What does the sign say?
A. Smoking impacts not just yourself but also those roughly you.
B. Handling behaviors violation of regulations at places where
smoking is forbidden.
C. The inhabitants need to raise their consciousness to comply with
the regulations on places where smoking is not allowed.
D. Everyone is not permitted to smoke here because it is hazardous
to our health.
16. What does the sign say?
A. Dogs can only use this beach sometimes.
B. Dogs owners may use the beach without their dogs.
C. Dogs can use the beach if they are with their owners.
D. Dogs are not tolerated here.
• Read the following extract from a newspaper article, ignoring the gaps. What method of
training for sporting activities is suggested.

You'll continue to exercise passive ______________ behavior because straightforward


communication was avoided when you were a child. (AGGRESS)
→ aggressive
The United States, the world's biggest economy, is finally starting to move _______________,
and China, the world’s largest emitter. (AGGRESS)
→ aggressively
The other piece of the puzzle is that OCD is associated with low levels of serotonin a neurotransmitter
that communicates between brain structures such as mood _______________ impulse control.
(AGGRESS)
→ aggression
A biopsy is the most effective means of determining the _______________ of the cancer.
(AGGRESS)
→ aggressiveness
We must send a message around the world that there is no disgrace in being a survivor of sexual
violence, that the shame is on the _______________. (AGGRESS)
→ aggressor
• For questions 13 - 27, read the text below and think of the word which best fits
each gap. Use only one word in each gap. There is an example at the beginning
(0).
KANGAROOS
The kangaroo is (0) ONE of Australia’s most iconic animals. Kangaroos of different types live
in all areas of Australia. (13) ___________ cold-climate areas and desert plains to tropical
rainforests and beaches.
Kangaroos are herbivorous, eating a range of plants and. (14) __________ some cases, fungi.
Different kangaroo species inhabit different habitats. Some, for example, make nests on the
ground while tree-kangaroos live (15) __________ the ground. Larger species of kangaroo
tend (16) __________ shelter under trees or in caves.
Most kangaroos are distinguished from (17) ___________ animals by the way they hop on
their strong back legs. A kangaroo's tail is used to balance while hopping and (18)
____________ a fifth limb when moving slowly. All-female kangaroos have front-opening
pouches that contain four teats (19) ___________ is in here that the 'joey' (20) ___________
baby kangaroo is raised (21) __________ it can survive outside the pouch. Most kangaroos
have no set breeding cycle and can breed all year round. (22) ___________ they are such
prolific breeders, a kangaroo population can increase fourfold in five years.
Kangaroos have long been important to the survival of Australia’s indigenous people. (23)
__________ have hunted them for tens of thousands of years, using both the meat and the
skins. (24) _________ Europeans arrived in Australia in the late eighteenth century, they too
hunted kangaroos (25) _________ survival. Kangaroos continue to be used as a resource, but
only under strict government controls. Nowadays only the four most abundant species of
kangaroo may (26) _________ commercially harvested for export, and then only by licensed
hunters in accordance (27) __________ an approved management plan.

+Answer: 1, from 2, in 3, above/off 4, to 5, other 6, as 7, It 8, or 9,


until 10, Because/ As/ Since 11, who …

• Read the following passage . Decide if the statements from 1 to 4 are True or False,
and choose the correct answers (A, B, C or D) for the questions 5 - 8.
People have been hunting for a million years; farming for ten thousand years; and for the last
hundred years, people in the west have been attempting to reject all of this, at least in terms of their
connection with animals. For an increasingly urbanized Western populace, animals are either things,
such as the sterile plastic-wrapped packages of featherless chicken wings offered on a Styrofoam tray,
or people, including the tea-drinking chimpanzees in a television commercial. The actual nature of
animals and their significance in the world, previously known to people whose lives were intertwined
with theirs, is now lost in a miasma of human fantasies.
Nature-worshipping counter-revolution transformed animals into objects of devotion to be
cherished, whereas the Industrial Revolution reduced them to mere tools to be exploited as humans
saw appropriate. Nowadays, wolves, the villains of classic fairy tales, have become ecological heroes,
majestic emblems of the outdoors and freedom. According to a survey of visitors to a well-known
zoo, our favorite animal is the giant panda, which zoo-goers describe as "cute, cuddly, and charming."
Although it's utterly solitary and unsociable with an ill-tempered and aggressive nature, it's not
important, In a world where nature is regarded as either a factory or a theme park, genuine insight,
which used to come through direct experience of the natural world and just might come from
scientific researches of behavior and ecology, is no match for such human assumptions.
• Read the following passage . Decide if the statements from 1 to 4 are True or False,
and choose the correct answers (A, B, C or D) for the questions 5 - 7.
When it comes to language learning, young children are frequently regarded as the real experts.
They are willing to take up the language they are exposed to until the age of around seven, without the
need for a teacher to explain the distinction between the tenses or the difference between subject and
object pronouns. They merely manage to obtain it somehow, and they do so subconsciously, that is,
without making any actual effort. Consequently, it's not strange that so many courses, apps, and
language teaching materials claim to teach you a foreign language as an adult in the same manner you
acquired your first language as a child. But is that even possible? Is it even desirable?
Adult learners should not be underestimated. A child may take seven years to become relatively
skilled (although with a limited vocabulary), but an adult can achieve excellent communication skills in
just one year. It might seem like a bold assertion, but I am living proof, having progressed from
intermediate to advanced in 16 of them — the majority of them as an adult. The ability to integrate the
unconscious ways we utilized as children with the conscious ones employed by adults has been the key
to learning so many languages for me. By following my five principles, we can achieve the best of both
worlds and become expert vocabulary learners in whatsoever language. It also contains five essential
principles for learning any language such as selection, association, review, storage, and use.
The number of words in a language such as English is in the neighborhood of one million words.
Who, though, is aware of that many? Or, more to the point, who wants to know that many? For every
language learner, being able to filter out what you don't need is essential. Trying to look up every single
word in a text without doing this would be like reading a whole newspaper just to get to the sports
section. Commit sure not to make this mistake. The most useful words in a language should be chosen
instead. Concentrate on what you can utilize. Doctors may also need to know that the word "cranium" is
another word for "skull".
BRINGING UP BADGERS
What now?’ was my help of antibiotics, in one so
immediate thought, as one of small.
my helpers carried a
Disheartened by my failure,
cardboard box towards me.
I continued rearing the others.
Since my husband Derek and I
But fate works in strange
turned our dairy farm in
ways: six days later a local
Somer set into an ‘open farm’
farmer, who had heard about
six years ago, we’ve
Bluebell, and a male, Willow. our cubs, came to see me.
established a reputation for
Initially, I used a syringe to feed Behind some silage bags, he had
looking after orphaned animals.
them but each cub had to be discovered a single male cub,
But the noise coming from the wiped with a warm cloth first to abandoned by its mother. He
box - a cross between a cackle simulate the sensation of the knew she would not return; the
and a bark “ was not one I mother licking them. This area had been disturbed too
recognized, so it was with great encouraged them to empty their much and already the cub was
interest that I peered in to see bowels and bladder. For the first cold and hungry.
three small grey badger cubs, two or three days, they were fed Willow II joined the fold. At six
each no more than eight inches every four hours. weeks, he was about two weeks
long. Their coats (later to reveal
Within a week the cubs had younger than the females and
fleas) were like velvet, and
progressed to drinking from a over the next few days, I
milky-colored eyes looked up at
bottle and were moving around, discovered why his mother had
me from three tiny black-and-
albeit shakily. With each other left him. Never had anything
white striped heads.
for company and a heated pad been so difficult to feed. To
I had never seen badger cubs as a substitute ‘mum’, they place the bottle’s teat in his
before. As most of them are seemed very contented. mouth and cajole him into
born between mid January and drinking I had to keep moving it
mid -March, they usually spend Three weeks after their arrival, around and squeezing it. After a
their early life underground and, however, I noticed that Willow full 10 minutes, he would latch
if orphaned, die of starvation, seemed lethargic, although he on to it as if he had not drunk all
never to be found. These three was still taking food as normal. day. By the end of April, the
had been brought in by building It was a warning sign. I should females were weaned onto
contractors; while laying drain have reacted straightaway; not creamed rice and then literally
pipes, their machinery had realizing its importance, I anything. It was to be a different
bored into the sett before they awoke the next morning to find story for Willow. He was happy
realized it. The mother was him dead. The vet’s post to give up the bottle but could
found dead with her babies still mortem revealed that Willow not master the habit of eating
suck ling her. I could see they had died from a lung infection. without walking through his
were healthy and well but they When bottle feeding any animal food, tipping it over, or just
were cold and whimpering. it is important not to let it drink sitting in it. Eventually, I
After defeating them, I took too fast, as the liquid can over offered him a sausage, which
them into the farmhouse kitchen flow into the lungs. In Willow’s little by little was chewed,
and installed them beside the case, this had caused an played with, and finally eaten.
stove for warmth. There were infection that would have been After a week of sausages, he
two females, Primrose and difficult to rectify, even with the
was ready to move on to must have been almost grasped this skill: his reactions
something else. impossible. By August my to being ‘caught’ were
foundlings had begun to turn sometimes as colorful as his
At nearly five months old, they
nocturnal and would go for bruises. It was during two of
were all eating cereals for
walks only at dusk or late at these late-night walks in the
breakfast; a meat and vegetable
night. We often went through very dry period that I spotted
meal for lunch and fruit and
the cider orchard; in its long other badgers in our field.
nuts, cheese, hard-boiled eggs,
grass, everyone was fair game. Presumably, they were having
and sun flower seeds in the
The cubs would get excited, to extend their territories to find
evening. The usual diet for
ruffle up their fur so that they enough food, although badgers
badgers is 60 percent
looked like snow balls, and are very territorial and will kill
earthworms, plus beetles and
chase each other’s tails. As others that wander into their
bugs, baby rabbits, mice and
‘human badgers’ we were territory. We were even warned
voles, and even shoots or roots
included in this sport. I learned that they would climb into our
of plants. Assuredly, my
to avoid those playful charges badger pen and kill the cubs, so
badgers were much better off
that ended with a sharp nip, but sheet metal was placed over the
than other cubs that year; the
Derek accompanied us only gate to make it as inaccessible
summer was exceptionally hot,
occasionally and so never as possible.
and digging for earthworms

+ QUESTIONS:

When I was in high school, I was debating whether or not to pursue a career in psychology, and a
teacher warned me that it required a lot of science. I didn't realize it at the time, although, how
much it had to do with science. Given that most non-specialists have just a rudimentary
knowledge of psychology, it's understandable that they have no idea how much science it entails.
But, whichever field of psychology you're talking about — and there are a lot of them — I
eventually discovered that there are always figures, statistics, experiments, and evidence to
grapple with and that in the process, you develop a repertory of competencies. A psychology
lecture was packed to capacity when I went there. There are many other individuals my age who
appreciate both the practical and pure interest worth of the subject. It's reasonable to believe
that there are many other people my age who appreciate this.

+ STUDYING PSYCHOLOGY:
 I was on the fence about pursuing a ……………………………………………… profession.
 But I did not know how much it has to do with science ……………………………………………… .
psychology
 Psychology is a mind science, yet many people are unaware of how much science it
at the time
………………………
entails
 In the end, though, I realized that no matter which discipline of psychology you are
discussing — and there are many — you must always address the issue of …………………………,
…………………………, ………………………… , and ………………………… . figures statistics
experiment evidence
 Having to struggle with and developing a …………………………………… of capabilities as a result.
s repertory
 The issue is also of interest to many people my age, both in terms of its practical use and its
………………………………………………… .
pure interest worth
THE COLOUR OF BLOOD
It is a gripping and enigmatic tale of power and intrigue in which the hero, Cardinal Stephen
Bem, tries to prevent societal upheaval and martial law in an Eastern bloc country that has
even to be identified. A concordance prevails between church and state, and Bem has been
effective in securing church schools, religious literature, and freedom of worship.
Nonetheless, the concordance is jeopardized by right-wing Catholics who are dissatisfied with
Bem’s politics and try to incite the people to demonstrate against the government.
Bem is detained by right-wing extremists posing as Security Law enforcement officers.
One of the greatest parts of the novel is when Bem escapes from his so-called "protected
custody" at a roadblock established by the army. Bem understands immediately before being
stopped at the checkpoint that the purported Security Policemen are imposters. He manages
to escape the automobile but is then confronted by Colonel Poulnikov, who aims a pistol at him.
Poulnikov must hold his revolver in such a way that only Bem, not the soldiers, can see it. Bem
is unable to openly engage the soldiers because he has to be free to confront the right-wing
Catholics. Nearly excruciating suspense. Bem manages to escape both groups, and the reader is
curious as to how this will all turn out.
The bishop's discussion at Bem's residence on Lazienca Street is another event in which
the reading becomes obsessive. Right-wing leader Archbishop Krasnov questioned Bem's
authority. He confirms having been the author of the leaflets encouraging people to
demonstrate against the government, but he denies following Bem's directions. Every reader
understands that the events at the Pywald ceremony the following day will be critical for the
country's future, and the groundwork for a massive showdown is being prepared at the
bishop's conference. No reader will be able to stop reading at that moment.

• Complete the text with the correct form of the verb in brackets:
I love (1) eating (eat) junk food and for years I (2) overdid (overdo) it. I mean, I used to
eat junk food every week and was about five kilos overweight. Not quite obese, but I
(3) was getting (get) there. Anyway, I went hiking with some friends, and I just couldn’t
keep up. My legs (4) were aching (ache) after the first day and it was a bit
embarrassing, to be honest! So, when I got back, I decided to start (5) keeping (keep)
track of what I ate by recording it in a food diary. I realized I needed to cut down on
processed food, like all those burgers I was eating! I (6) monitored (monitor) the
amount of salt I (7) was consuming (consume) too – just to make sure I kept it within a
reasonable limit. And, I started exercising: walking to work instead of taking the bus,
that type of thing, and then, after about 12 months, I decided I’d like to run a
marathon. Yeah, I know! It was hard work, all that training, but I (8) was genuinely
pleased (genuinely / please) with my rate of progress and, well, I ran my first one last
month in under four hours!
1. who 2. help 3. was

4. most 5. despite / although 6. Throughout / During

9. Because / Regardless
7. joining / attending 8. while / whilst / whereas
10. as
• For each space, choose ONE word which you think best completes the sentence. Look carefully at the
words both before and after each space:
A VIOLIN TO THE RESCUE
A virtuoso violinist (1) _________ overcomes an extraordinary disability received much-needed (2) _________
during a concert on the grounds that a man's desire.
Betty-Ann Fischer, who was born in Ontario, Canada, (3) ___________ a musical child prodigy who
performed her first violin recital at the age of six, but it isn't the (4) ___________ astounding thing about her. She
was born with misaligned fingers. She continued (5) ___________ being told she'd never be able to play the violin
with her stumps. According to performance reviews, she developed her technique and could coax breathtaking
tones from her violin. (6) ___________ her professional career, she became a member of the Toronto Symphony
Orchestra.
Betty-Ann debuted at the Vanity Theater in Windsor, Ontario, at the age of 21, before (7) __________ the
Toronto Symphony. Her violin’s fingerboard broke during the second half of the program, (8) __________ she
was playing Edouard Lalo's Symphonie Espagnole. That would have been the end of the performance if it hadn't
been for Mario De Soto, a local violinist.
(9) ___________ of a dream, he experienced the night before the concert, De Soto was forewarned of what
would happen. De Soto volunteered his violin when Betty-life Ann's transpired precisely (10) __________
prophesied in his dream, and she concluded her concert on the strength of his premonition...
One of the most important is put. Then, unit sales rise when the product is proving
concepts in sales management steadily through the growth successful, competitors will
and marketing is that of the phase to the maturity phase, bring out their own ‘copy-cat'
product life cycle. This is a when the product is widely products. With a competitor in
historical record of the life of a accepted, and so on to saturation the field, the original firm has to
product, showing the stage in its level. By this time, competitors respond to maintain its market
life the product has reached at a will have entered the market position. It can run special sales
particular time. By identifying with their version, and. from promotions, improve deliveries,
the stage that a product is in or this point, the sales team will make more frequent sales calls,
maybe heading towards, have to work even harder to win and so on. Often the extra
companies can formulate better all additional sales. Eventually, expenditure is not accurately
marketing plans. All products the product's sales decline as charged to the product and the
have ‘lives in as much as they better versions enter the market result is that, long before unit
are created, sell with varying and competition becomes too sales are noticeably falling, the
profitability over a while, and strong. unit profit has already fallen.
then become obsolete and are In retrospect, most firms know The product life cycle, then,
replaced or simply no longer what happened to their products presents a picture of what
produced. A product's sales from launch to withdrawal. happened in the product's
position and profitability can be They can compile this ‘lifetime', so how can this be
expected to fluctuate over time information from the records of used as an ongoing aid to
and so. at each successive stage unit sales. management decision-making?
in the product's cycle, it is Every sales manager has a chart
Unfortunately, unit sales are not
necessary to adopt different on which the progress of sales is
the complete story as it is unit
tactics. plotted and this can be used as a
profit that is the decisive factor,
The two main features of the although this is not always guide to the stage of
product life cycle are unit sales recorded accurately. It is this development each product is
and unit profit. The unit sales figure that sales management currently in. Essential
figures usually jump on has to monitor, though, to management skill is being able
introduction, as a response to ensure an effective marketing to interpret sales results and
heavy advertising and strategy and to produce draw in the stages as they occur.
promotion, as customers buy the effective profits. Deciding where each stage
product experimentally. This is At launch, the product is costed begins and ends can be a
generally followed by a leveling accurately based on production random exercise, though usually
off while it is evaluated - the costs plus selling costs. Initially, the stages are based on where
length of this period depending these remain fairly stable, but. the rate of sales growth or
on the use to which the product decline becomes pronounced.
I, READING
A1: ELEMENTARY
THE STORY OF SALLY
Yesterday morning when Sally woke up, she was happy. It was Sunday and she liked Sundays.
She went downstairs and her mother made some pancakes with honey for breakfast. The weather
wasn't very cold, but it was cloudy. Sally put on her jacket before she went outside. She climbed
on a rock and sat there looking at the countryside.
'Well, Sally, what would you like to do today?' her mother asked. 'I want to go and see my friend
Jane.' Sally said. 'Does she live near the river?' her mother asked. 'That's right. You know her
father. He works at the hospital,' Sally said. Sally phoned her friend, and her mother drove her
to Jane's house.
When Sally arrived at the house, both girls decided to go fishing. The girls sat near the river and
started fishing. Then Jane shouted, 'I think I've got a fish!' She caught the fish and took it out of
the water. The fish jumped and then it was in Jane's curly brown hair! Sally laughed. 'Oh dear,
now you need to wash your hair,' The fish jumped out of her hair and landed back in the river.
The girls never caught any more fish. When they got back to the house. Jane's mother was
making fish and chips for lunch.
A2: PRE-INTERMEDIATE
TSUNAMIS
A lot of people who live near the sea are scared of Tsunamis because these are very big waves
that come from the sea and cause a lot of damage to houses on the coast. They happen when
there is an earthquake or some other disturbance under the sea.
When this happens, a lot of water moves from one place to another, and this wave travels under
the sea very quickly. Sometimes they are caused by volcanoes under the sea erupting because
volcanoes don't just erupt on land. Another reason Tsunamis happen is when part of an
underwater mountain falls.
Some people wonder where most Tsunamis happen. That is a difficult question to answer, but
Tsunamis need a lot of water to happen, so tsunamis are the most common places for the oceans.
There is one ocean with many mountains and volcanoes under it, The Pacific Ocean, so
Tsunamis are very common there.
Tsunamis are very dangerous for a few reasons. This big wave of water travels very fast under
the sea and when it gets to land, there is no space for all this water to go, so it goes up into the
air. This makes a very high wave which covers everything on the beach and the land. The main
problem with tsunamis is that they happen without warning. So, people might be asleep at night
and suddenly a tsunami could appear and cover everything in the water. A lot of people get
killed like that, especially those that live on little islands.
B1: INTERMEDIATE
COMEDY KIDS
Fourteen-year-old Miguel Diaz talks about the youth comedy club

Do you enjoy watching the comedy? Do all your friends at school think you tell great jokes?
Then why not come and see what’s happening at Comedy Kids?

Comedy Kids own comedy clubs in several countries that are run just for young people aged 10-
15. They’re real clubs with real comedians, who are just as familiar with working in adult
comedy clubs as they are working with children. But don’t take our word for it – come and see
for yourself!

The Comedy Kids company was set up by John Winterton, who says that no one had thought of
being a stand-up comedian for young people until he came along. In fact, his act quickly became
so popular that he soon found lots of other comedians who wanted to join his company and
perform for families and young people. And if you think, as many others do, that a young
people’s comedian leaps on stage in a brightly colored costume and big red nose – think again.
John usually appears dressed in a cool black jacket and trousers – more like a Hollywood movie
star than a circus clown!

If you’re having a party at home Comedy Kids can come and perform for you there at your party
– just ask! Of course, it’s not the same as performing on stage, so we’re always looking for
comedians in the clubs who are good at working with small numbers of young people – and
we’ve discovered there aren’t many of them about! But the ones we have found are brilliant.
Your very own comedian will also help you to tell your guests some jokes and silly stories – and
they can join in too if they want! It’ll be fun – and very different from any party you’ve ever had
before!

Here at Comedy Kids, it’s not just about the adults telling the jokes. We also have very short
‘open spots’ where young people get the chance to perform. But we’ve yet to find someone who
can do that without preparation, so we’ve set up the world’s first ‘Comedy Classes’ to teach
young people how to tell jokes on stage. At the moment our classes are only monthly, but we’re
planning to start several others over the coming year. So if you want to be a comedian but don’t
want to wait until you grow up, this is where you can do it. And if you’re creative but feel
you’re lacking in the confidence to speak up, we can certainly help. We’ll listen to what you
want to do, and try to make it happen. So what are you waiting for? Come and join Comedy
Kids!

II, READING (B2 – C2)

B2: UPPER-INTERMEDIATE
HOW TO BE A TEENAGER AND SURVIVE!
Being a teenager in these troubled times isn’t to help you get through your darkest moments.
exactly a walk in the park. The uncertainty only Communication has got to be the first tip. If you
adds to the typical list of problems you're already keep negative feelings in, reach a point where
facing. So, what are the challenges you are up you think you're going to explode. Confiden
against and how can you weather the storm and someone about how you are feeling with
arrive safely on the other side? someone you can trust. Even if they can't give
As a teenager, the biggest challenge you, your you the advice you need, just getting it off your
family, and your friends face is mood swings. chest can make a world of difference.
One minute you feel ecstatic, full of confidence But, what can you do if you're at exploding point
and optimism, and the next you feel depressed, and there's no one to talk to? If you want to avoid
angry at the world, and certain that you are a conflict with others, go somewhere on your own
failure. Personal appearance also becomes a huge for a minute or two to process things. Take time
issue. The hormones racing through your body to breathe properly, when we are in stressful
play havoc with your self-image. Suddenly, you situations our breathing becomes short and
have become too fat, too thin, too ugly, too short, rushed, and as a result, less oxygen reaches the
too tall; your hair is too dark, too light, too curly, brain. This can heighten the negative feelings that
too straight, etc. In short, you are completely we have. One technique is to close one nostril
dissatisfied with your appearance. with your thumb and inhale, then close the other
Relationships with others also become more nostril with your index finger and exhale as you
complicated. You may find you no longer have release your thumb from the first nostril. Do this
that much in common with the friends you've at least ten times and your breathing will return to
been hanging around with for years. It can also be normal and you will feel much calmer in the next
distressing when a former best friend now prefers to no time.
to spend time with other friends. As for family Physical exercise like running, cycling and
relationships, well, it often seems that a war has swimming are also great ways to clear the mind
been declared, and parents and siblings have and they have the bonus of keeping you fit. If you
become the enemy. do this whenever you're down, you'll also start to
feel better about yourself and your body. There's
But it needn't all be doom and gloom. The
no need to push yourself to the limits, though.
teenage years are unique in a person's life. They
Listen to your body and stop when it says, 'No
mark the end of childhood and the important
more!'
passage to adulthood. The key to a happy
'teenhood' is to recognize that no matter how your Finally, always try to focus on the positive
emotions change or how insecure you feel about aspects of your life. Being a teenager means you
yourself, you are perfectly normal! Everyone, big have more freedom. Use it in constructive ways
and small, experiences feelings of depression and so that feelings of frustration are replaced by
anxiety from time to time. But here are a few tips feelings of accomplishment.

PARAGRAPH: B2

MAYAN TECHNOLOGY
The ancient Maya, who lived in parts of present-day Central America, had a very sophisticated,
complex civilization and achieved remarkable things. By observing the skies they created a
highly accurate calendar; they introduced new farming methods, and they managed to build
spectacular temples and great cities without the use of metal or the wheel.
One of their greatest accomplishments is vulcanization, which is the process of combining
rubber with other materials to make it stronger and longer-lasting. Until recently, the discovery
was credited to Charles Goodyear of the United States, who patented the technology in 1843.
Nevertheless, scientists and historians now believe that the Maya were producing rubber
products from around 1600 BC - about 3,400 years before Goodyear. In fact, in the 16th century
when Spanish explorers arrived in the area, they were astonished to see rubber balls, which were
objects they had never encountered before. Even describing the stretchy, bouncy material proved
difficult as it did not exist in Europe and was therefore beyond their experience.
It is believed that the Maya discovered this process by accident when they cooked latex from
rubber trees with juice from a plant called morning glory. They quickly realized how strong the
new material was and the wealth of things it could be used for: to line the soles of their sandals
thus making them water-resistant, like glue, and for the large rubber balls they used in the game
known as pokatok played on stone-walled ball courts as part of a religious ceremony.
C1: ADVANCED
INTRODUCTION TO A BOOK ABOUT THE HISTORY OF COLOR
This book examines how the ever-changing role objects? No researcher, no method, has yet been
of color in society has been reflected in able to resolve these problems, because, among
manuscripts, stained glass, clothing, painting, and the numerous facts pertaining to color, a
popular culture. Colour is a natural phenomenon, researcher tends to select those facts that support
of course, but it is also a complex cultural his study and conveniently forget those that
construct that resists generalization and, indeed, contradict it. This is a poor way to conduct
the analysis itself. No doubt this is why serious research.
works devoted to color are rare, and rarer still are
those that aim to study it in a historical context. And it is made worse by the temptation to apply
Many authors search for the universal or to the objects and images of given historical
archetypal truths they imagine reside in color, but period information found in texts of that period.
for the historian, such truths do not exist. The proper method – at least in the first phase of
analysis – is to proceed as do paleontologists
Colour is first and foremost a social phenomenon. (who must study cave paintings without the aid of
There is no transcultural truth to color perception, texts): by extrapolating from the images and the
despite what many books based on poorly grasped objects themselves a logic and a system based on
neurobiology or – even worse – on pseudo various concrete factors such as the rate of
esoteric pop psychology would have us believe. occurrence of particular objects and motifs, their
Such books, unfortunately, clutter the distribution and disposition. In short, one
bibliography on the subject and even do it harm. undertakes the internal structural analysis with
which any study of an image or colored object
The silence of historians on the subject of color, should begin.
or more particularly their difficulty in conceiving
color as a subject separate from other historical The third set of problems is philosophical: it is
phenomena, is the result of three different sets of wrong to project our conceptions and definitions
problems. The first concerns documentation and of color onto the images, objects, and monuments
preservation. We see the colors transmitted to us of past centuries. Our judgments and values are
by the past as time has altered them and not as not those of previous societies (and no doubt they
they were original. Furthermore, we see them will change again in the future). For the writer-
under light conditions that often are entirely historian looking at the definitions and taxonomy
different from those known by past societies. And of color, the danger of anachronism is very real.
finally, over the decades we have developed the For example, the spectrum with its natural order
habit of looking at objects from the past in black- of colors was unknown before the seventeenth
and-white photographs and, despite the current century, while the notion of primary and
diffusion of color photography, our ways of secondary colors did not become common until
thinking about and reacting to these objects seem the nineteenth century. These are not eternal
to have remained more or less black and white. notions but stages in the ever-changing history of
The second set of problems concerns knowledge.
methodology. As soon as the historian seeks to I have reflected on such issues at greater length in
study color, he must grapple with a host of factors my previous work, so while the present book does
all at once: physics, chemistry, materials, and address certain of them, for the most part, it is
techniques of production, as well as iconography, devoted to other topics. Nor is it concerned only
ideology, and the symbolic meanings that colors with the history of color in images and artworks –
convey. How do make sense of all of these in any case, that area still has many gaps to be
elements? How can one establish an analytical filled. Rather, this book aims to examine all kinds
model facilitating the study of images and colored of objects to consider the different facets of the
history of color and to show how far beyond the the history of color err in considering only the
artistic sphere this history reaches. The history of pictorial, artistic or scientific realms. But the
painting is one thing; that of color is another, lessons to be learned from color and its real
much larger, question. Most studies devoted to interest lie elsewhere.

PARAGRAPH: C1
THE CHILD MARRIAGE
Worldwide, more than 700 million women alive today were married before their 18th
birthday. More than one-third entered into union before the age of 15. Boys are also
married as children, but girls are disproportionately affected. According to UNICEF, child
marriage - usually forced, too much older men, and sometimes constituting slavery - is a
manifestation of gender inequality, reflecting social norms that perpetuate discrimination
against girls. Girls who marry are not only denied their childhood. They are often socially
isolated - cut off from family and friends and other sources of support - with limited
opportunities for education and employment. Lower levels of education are common
among women who married in childhood. Child brides typically end up having many
children to care for while still young.
In Nepal, for example, over one-third of women aged 20 to 24 who married before their
15th birthday had three or more children compared to one percent of women who married
as adults. Child brides are also less likely to receive proper medical care while pregnant.
This, along with the fact that girls are not physically mature enough to give birth, places
both mothers and their babies at risk. Pregnancy-related deaths are the leading cause of
mortality for girls aged 15-19 worldwide.
Many cultures condone the practice of child marriage. In some of these cases, the girls may
give consent, but the extent to which this can be described as free and informed when they
are pressured by society, their family, and economic stresses are problematic. Many are
prevented from developing relationships in the name of the family's 'honor' and some face
unspeakable violence if they do. According to the UN Convention on the Rights of Children
(CRC), in exceptional circumstances, a mature and capable child over the age of 16 may
marry provided that such decisions are made by a judge based on legitimate exceptional
grounds defined by law and on the evidence of maturity without deference to cultures and
traditions.' The CRC aims to raise the minimum age of marriage across countries and
cultures. But any legislative changes or enforcement must be accompanied by measures to
change social norms promoting child marriage, including the notion that girls must be wed
early as a survival mechanism for families ridding themselves of an economic burden

C2: PROFICIENCY
FOLK MUSIC & BLUES MUSIC
The most crucial, as well as the most frequently values and as an index of how its communal life
overlooked, point about 'folk music is that the has changed. It is this latter attribute which many
constituency whom it most truly represents traditionalists find alarming or repugnant. For
doesn't consider it to be 'folk music, but simply them, the key element is the preservation of a
their music. 'Folk music' is, invariably, a term piece's pure and unsullied essence, and the
applied from outside the cultures and imposition of an alien style onto a traditional
communities to which it refers. In terms of theory, piece is deemed an act of presumption verging on
'folk music' - the traditional set of forms, styles, outright heresy: at the very least, it effectively
and songs indigenous to a people, a culture, or a amputates the piece from its native roots.
locale - is radically distinguishable from 'art'
music, of both the classical and avant-garde In the blues world, the picture is far more
varieties, and from 'popular' music, mass- complex. Blues obeys a different set of
produced for and mass-marketed to a mass imperatives and simultaneously holds the
audience. In practice, it’s getting harder and following truths to be self-evident: yes, there is a
harder to tell them apart. strong and very clearly defined tradition, and, yes,
its practitioners are expected to improvise freely
Before the advent of recording, distinctions within it, recreating it anew to meet the
between categories of music were not so much immediate needs of both performer and audience.
based on the music itself as on who it was by and There are set themes, and there are specified
for. Such distinctions were a reflection of the functions: dance songs, work songs, celebrations,
class system, which is not surprising since these laments, love songs, hate songs, and so forth. The
are essentially European definitions, and reflect tradition is unfixed; indeed, it demands to be
prevailing European social structures. European freshly reinvented with each performance,
classical music operates according to a strict recreated anew to reflect the changing needs and
hierarchical structure, with the composer (the circumstances of its time and place. Blues artists
monarch, so to speak) at the top. The composer's both ancient and modern have worked from a
wishes are interpreted and enforced by the 'common stock’ of folk materials: instrumental
conductor (the general) and carried out by the motifs and vocal tics, melodies, lyrical tags, chord
orchestra (the troops). During their lifetimes, the progressions, and even complete songs are
great composers often also functioned as the derived directly from the tradition, and some of
featured soloists, but after their deaths, their them long predate the era of recording, let alone
music became fixed and formalized; those who the conventional mechanics of publishing and
succeeded them rarely inherited their license to copyright laws. What counts above all in the
improvise. blues is individuality: the development of a
The classic model of 'folk' is the similarly formal unique and unmistakable voice, the ability to
tradition of the Anglo-American ballads, with place an ineradicable personal stamp on those
their fixed musical structures and set narrative 'common stock' materials freely available to all.
lines. To perform one of these ballads, a singer is While instrumental dexterity, vocal facility, and
by definition required to preserve intact both its stylistic versatility are heartily respected within
storyline and its musical setting. The Anglo- the blues community, what distinguishes the truly
American use of the term 'folk' music implies that great from the merely professional is the fully
such music exists, simply and solely, to fulfill the realized man (or woman)'s communicated essence
needs of a particular community. They develop it of self; the ability to serve as a conduit for the full
by and for themselves over centuries as part of a gamut of human emotion, to feel those emotions
single collective process, only slightly more with sufficient depth and intensity to reach out
personal to any given individual than the shaping and touch listeners in places that those listeners
of rock by water. Through oral transmission, it might not even have known that they had.
filters down through the generations, serving both Without exception, every blues singer who has
as a touchstone of the community's history and managed to pull ahead of the pack or haul himself
from the hordes of hopefuls chasing the blues contemporary brand leaders, is the scale on which
lovers dollar has this quality. Any competent performers are capable of being themselves in
blues artist should have the ability to entertain - public. And, by extension, the depth and
those who don't should simply find another line of complexity of that self. To serve as a neutral
work before they starve to death - but the measure transmitter simply doesn't cut it here.
of true mastery, from the 1920s pioneers to the

PARAGRAPH: C2
THE DEFINITION OF CULTURE
The internally influenced culture of both forces encourages change and forces it to resist change. These
forces are related to both social structures and natural events and are involved in the perpetuation of
cultural ideas and practices within current structures, which themselves are subject to change.
Social inconsistencies and the development of technology can induce changes in society by changing
social dynamics and promoting new cultural models, and promoting or facilitating development
actions. These social transferrals may accompany ideological shifts and other types of cultural change.
For instance, the U.S. feminist movement involved new practices that produced a shift in gender
relations, altering both gender and economic structures. Environmental conditions may also enter as
factors. For instance, after tropical forests returned at the end of the last ice age, plants suitable for
domestication were available, leading to the invention of agriculture, which in turn brought about many
cultural innovations and shifts in social dynamics.
Cultures are externally affected via contact between societies, which may also produce — or inhibit —
social shifts and changes in cultural practices. War or competition over resources may impact
technological development or social dynamics. Additionally, cultural ideas may transfer from one
society to another, through diffusion or acculturation. In diffusion, the form of something moves from
one culture to another. For instance, Western restaurant chains and culinary brands sparked curiosity
and fascination in the Chinese as China opened its economy to international trade in the late 20th-
century. "Stimulus diffusion" refers to an element of one culture leading to an invention or propagation
in another. "Direct borrowing," on the other hand, tends to refer to technological or tangible diffusion
from one culture to another. The proliferation of innovation theories raises why and when individuals
and cultures update concepts, practices, and products based on research-based models.

COOLING THE EARTH


As a last resort to combat global warming, researchers are investigating
two possible ways of applying ''sunscreen'' to the planet.
Even with the best will in the world, reducing our case? Well, fortunately, if the worst comes to the
carbon emissions is not going to prevent global worst, scientists still have a few tricks up their
warming. It has become clear that even if we take the sleeves. For the most part, they have strongly resisted
most drastic measures to curb emissions, the discussing these options for fear of inviting a sense of
uncertainties in our climate models still leave open the complacency that might thwart efforts to tackle the
possibility of extreme warming and rises in sea level. root of the problem. Until now, that is.
At the same time, resistance by governments and
special interest groups makes it quite conceivable that Quite recently a growing number of researchers have
the actions advocated by climate scientists might not been taking a fresh consider large-scale ‘Geo-
be implemented soon enough. Is the game up in that engineering’ projects that might be used to counteract
global warming. Fundamentally, the idea is to apply larger-scale, more expensive alternatives that might
‘sunscreen’ to the whole planet. It’s controversial, but carry fewer risks. One that might do the trick is a
recent studies suggest there are ways to deflect just space-based sunshade system. It may sound
enough of the sunlight reaching the Earth’s surface to passionately implausible but some scientists are
counteract global warming. Climate models designate convinced that it is feasible.
that blocking just 1.8 percent of the incident energy in The idea is to manufacture discs of silicon about 60
the sun’s rays would cancel out the warming effects centimeters across. Each disc would be studded with
produced by a doubling of carbon dioxide and other holes of precisely calculated sizes, close to the
gases in the atmosphere. That could be crucial wavelengths of visible light, which would scatter
because even the most stringent emissions-control incoming light like a lens. The consequence would be
measures being suggested would leave us with a to produce a slight but imperceptible dimming of
doubling of carbon dioxide by the end of this century, sunlight. These simple devices would be packed into
and that would last for at least a century more. metal containers in stacks of a million and propelled
So, is the concept of a technological fix new? Not at into space using electromagnetic rail guns – a method
all; but while most remedies have focused on that has been experimented with in labs but never
combating greenhouse gases themselves – finding actually used. The acceleration is far too rapid for
ways to eliminate them from the air or scrub them people or delicate equipment, but the method has long
from power-plant emissions – only recently have been proposed for shooting bulk material into space,
more radical ideas been taken seriously. There are two such as water, rocket fuel, or building materials. It
distinct proposals: reflecting away sunlight within the could be more economical and more reliable than
Earth’s atmosphere or blocking it in outer space. Each traditional rockets.
approach has its supporters and detractors. While Once launched, the receptacles would travel to the
tinkering with the atmosphere is likely to be more place between the Earth and sun where their
inexpensive and simpler, space-based approaches may gravitational fields cancel out, allowing objects to
be longer-lasting and less likely to cause unwanted remain stationary relative to the two bodies. This is
side effects – though they are much more technically where the contents would be released. Scientists
challenging. speculate they could be deposited in place for 50 years
or more. Independent computer simulations show that
The simplest method put forward has been the space sunshade could almost cancel out the
appreciated for decades. That is to inject sulfur temperature changes in sunlight that are most
dioxide into the stratosphere, mimicking the cooling vulnerable at the poles. This regional difference in
effects of volcanoes. Sulfur is cheap, and the means of cooling might cause unpredictable changes in weather
releasing it could be as simple as pumping it up patterns. And since the poles would see less of an
through a vertical pipe as much as ten kilometers influence from the dimming, they might still
long. Sulfur dioxide forms sulfate particles that are experience a significant decline of ice cover.
big enough to block part of the incoming sunlight, but
small enough to allow infrared wavelengths – the heat So, which approach has the edge? It comes down to
radiation from the Earth – to escape back into space. costs and feasibility. If we were suddenly sustained
In addition, since it is naturally present at elevated with a climate catastrophe, the sulfur-particle
heights above the earth, some researchers think an approach is cheap enough to be essentially free. The
increase might not present as many unforeseen risks engineering is simple enough that it could be put up in
as some other suggested remedies for global a couple of years. The space sunshade, though
warmings, such as seeding the ocean with iron filings attractive, seems unlikely to be implemented. If cost
either another nutrient to encourage the growth of were no object, one would want to utilize something
carbon-consuming organisms. similar to this latter scheme, because it’s identical
clean and controllable, and would likely minimize any
The approach is not without side effects, however. secondary effects. But it’s very high-priced. If you
Anything we do within the Earth’s atmosphere might would like to go to that much effort, it would be
have unpredictable results that turn out to be more simpler just to change our energy systems. Nobody
critical than the cure, such as dramatic changes in wants to have to do this but if you get to the point
regional rainfall or drought patterns, or chemical where the alternative is six meters of sea-level rise,
reactions that might disrupt ecosystems. These we might want to have this as a possibility. We’re not
drawbacks have driven others to look seriously at going to implement it, but you unquestionably have to
know what’s possible. It’s like emergency you still have to practice it.
reinforcement surgery: you never want to do it, but

MONEY MATTERS
Many crucial factors harmonize into providing a solid per student than the districts with the lowest rates of
education to students in public schools. poverty.
Knowledgeable, committed teachers, high-quality Meanwhile, the Education Trust's 2015 study found
tools and equipment, and safe, clean facilities are that, within states, the dispersal rates varied. Some
only some of the provisions essential for an effective, states did provide the most funds to the highest-
positive learning environment. Behind all of these is poverty districts. Others did not, even though states
the most important provision of all: an adequate have the context to best understand which of their
supply of funding. Unfortunately, public school districts require the most aid.
funding from local and federal sources is perpetually
limited and often distributed unfairly. As a result, it is Inadequate and unfair funding for public schools
up to states to ensure that students in each district get leads to an achievement gap, a longstanding
a chance at a good education. Funding must be difference in academic success between students of
increased, at least for lower-income districts, to come different backgrounds. Unfortunately, there is a
close to this goal. growing achievement gap based on families' income
levels. But the news isn't all bad. For example, in
At the local level, money designated for district 1990, Ohio spent $1.41 in state aid for low-income
public schools usually comes from property taxes. districts for every dollar it spent in high-income
Typically, that means the public schools with the districts. By 2011, this figure had grown to $1.94.
most qualified teachers and best facilities will Over twenty-one years, as state aid increased for low-
probably be in areas with the highest property values. income districts, Ohio's socioeconomic achievement
The public schools in impoverished areas are more gap narrowed. Other states implemented similar
likely to suffer from a lack of funding, which hinders educational finance reforms in the 1990s. Data show
their education students. And current data show that that test scores increased for low-income districts
the dispersal of federal funds is also unjust. The
whose state aid increased. In states that did not
Education Trust published a study in 2015 that found
enact any educational finance reform for their low-
the U.S. school districts with the highest rates of
income districts, test scores declined relative to those
poverty receive about $1,200 less in federal funding
of students in high-income districts.
In spite of this evidence, some argue that money is a disadvantage before formal schooling even begins.
not public schools. Detractors point to Camden, New They are less likely to own books, computers, and
Jersey, one of the lowe income districts in the state. educational toys and to have access to other learning
In the late 1990s, New Jersey sharply increased resources. This is why, in October 2000, a judge in
spending for Camden schools, but the long-term North Carolina ordered the state to offer free
results were poor. Today, more than ninety percent of preschool to at-risk children. During the 2008-2009
the district's high school students are not proficient in school year alone, North Carolina spent 170 million
language arts or math. Only one-third of Camden's dollars providing free preschool for about thirty-five
high school seniors graduate on time. And New thousand children. Research conducted since the
Jersey is currently spending about twenty-three program's start shows that the at-risk children make
thousand dollars on each student per year, more than better-than-expected gains in kindergarten, though
double the public school national average. those gains fade by the end of third grade. As such,
Nevertheless, some in Camden point out that much of the program demonstrates that investment in early
the additional money has gone toward long-needed education is crucial to student success.
resources such as cafeterias and textbooks, which,
while important, might not immediately translate to While programs like North Carolina's are a boon for
academic gains. low-income children, the same children are so often
held back by a lack of investment in public schools.
Factors outside of school often affect whether a The research shows that increasing per-pupil
student is successful academically. Poverty, in spending now leads to higher wages and reductions in
particular, creates learning challenges from an early poverty for students when they become adults. If
age. Funding for education is especially crucial for enough officials looked closely at the data, they
young children in low-income households, who are at would recognize the value of increasing public school
funding.

THE ENDURING MYTH OF MUSIC AND MATHS


As a mathematician with strong musical interests who developed into a family of musicians, I have
been asked about the connection between music and maths many times. And I have bad news: although
there are some obvious similarities between mathematical and musical activity, there is (as yet) no
compelling evidence for the kind of mysterious, almost magical connection that many people seem to
believe in. I’m partly referring here to the ‘Mozart Effect’, the hypothesis that children who have heard
music by Mozart are supposedly more intelligent, including at mathematics than children from a
control group.
It is not hard to see why such a theory would be taken seriously: we would all like to become better
at mathematics without putting in any effort. But the conclusions of the original experiment have been
grossly exaggerated. If you want your brain to work better, then not surprisingly, you have to put in
some hard graft; there is no such thing as an intellectual perpetual-motion machine. Mozart CDs for
babies and toys that combine maths and music might help, but not much, and the effects are temporary.
Of course, this conclusion does not show that there is no interesting connection between
mathematics and music. It was always a little implausible that lazily listening to a concerto would earn
you extra marks on that maths test you are taking tomorrow, but what about learning to read music or
spending hours practicing the piano? That takes genuine effort. Could it be that the rewards for that
time-consuming dedication spill over into other areas of intellectual life, and in particular into
mathematics? Is there any evidence that people who have worked hard to become good at music are
better at mathematics than completely unmusical people? And in the other direction, are
mathematicians better than average at music?
Demonstrating a connection of this kind is not as easy as one might think. To begin with, there are
plenty of innumerable musicians and tone-deaf mathematicians, so the best one could hope to
demonstrate would be a significant positive correlation between aptitudes at the two disciplines. And
then one would face all the usual challenges of establishing a statistical connection. For example,
identifying and controlling for other potentially influential factors is difficult, and as far as I know,
there has been no truly convincing study of that type that has shown that musical ability enhances
mathematical ability or vice versa.
Nevertheless, the belief that the two are interestingly related won’t go away without a fight. I cannot
help observing that among the mathematicians I know, there do seem to be a surprising number who
are very good indeed at the piano. I believe that there is a study waiting to be done on this: are
mathematicians more drawn to this rather than to other instruments? Of the mathematicians I can think
of who are superb instrumentalists, all but one are pianists. While we wait for scientific evidence to
back up the anecdotal evidence, can we at least argue that it is plausible that there should be a
connection?
Indeed, yes, we can. For a start, both mathematics and music deal with abstract structures, so if you
become good at one, then it is plausible that you become good at something more general – the
manipulation of abstract structures – that helps you with each other. If this is correct, then it would
show a connection between mathematical and musical ability, but not the kind of obscure connection
that people hope for. I feel that it would be more like the straightforward link between ability at football
and ability at cricket. To become better at one of those then you need to improve your fitness and
coordination. That makes you better at sport in general.

SWEETNESS LOST
As soon as the truck's back doors creaked open, Sweetness didn't think about the things he didn't
Sweetness zoomed out in a loud flapping of know, so he wasn't disturbed by them.
wings. The driver quickly stepped back, his arms
over his face. "What in the—" he sputtered, but He had never been this far away before, but
Sweetness was already high above him. If Home was drawing him with a great warm
Sweetness had looked back, he could have breath, and all Sweetness had to do was lean
watched the driver shrink to a blot, crumb, speck. toward it because Sweetness could always find
But he didn't look back. his way home. Below him, people drooped, stuck
in themselves, but Sweetness was free, of them
Sweetness knew only a few things, but these and himself. This was the best part about flying:
things were grand. Wherever he went, he knew the boundary between objects in the universe
exactly where he was. How many of us can say disappeared, and Sweetness was everything and
the same? He knew the shape of the sky, the tilt everything was Sweetness. When he flew, he
of the earth, the scents of the rain and sun and was sky-blue heat and cool, brilliant water, and
snow. And of course, he knew Home—its that was all, and that was enough.
shapes, shadows, density, timbre. He knew
Home so well, he could find it from miles away. The sweetness was flying higher than ever
before. But suddenly, the breath of Home
The things Sweetness didn't know, however, vanished. The home was like a mouth, and it had
could fill a world, a galaxy, a universe. He didn't clamped itself shut. The sweetness was shut out.
know that the dark, jostling place into which he He startled in a jolt of confusion. Then, he flew
had unintentionally flown was a truck, nor did he in a desperate, random direction. An entirely
know why he now found himself so far away foreign breath washed over him. He turned
from Home. He didn't know about pettiness or again. That way: another foreign breath. This
guilt or envy, about ruthlessness or exploitation way: there was no breath at all. It was a horrific
or insecurity. Perhaps most profoundly, he didn't betrayal. He kept turning. Nothing. Sweetness
know who he was. His person, the one who took scribbled a frantic path across the sky.
care of him back Home, called him "Sweetness,"
even cooed it sometimes, but Sweetness didn't What he didn't know was that he had flown into
know what that meant. Others scorned him as a a place in which Home can no longer be felt or
"sky rat," but he didn't know what that meant heard, and in which the bonds of familiarity
either. disappear. There's nothing one can do in such a
place except give in, and eventually Sweetness
did. Too despondent to quench his thirst, he quickly supped at the stream. Sweetness's shock
found a low, flat building with a good ledge. He was replaced with a wave of primal anger that
roosted there for the night. astonished him and made the stranger back off.
But as Sweetness returned to his dance, he
He woke so parched that, had he been offered the became aware that the stranger was laughing at
choice between Home and water, he would have him. She hopped onto the lever and let the water
chosen the water. He didn't have to look far, flow, staring at him with merry eyes. A new
though. There was a shiny metal thing below, emotion bubbled up in Sweetness—gratitude?
and the scent of water was there. Sweetness Friendship? The sweetness didn't have a name
alighted eagerly upon it, only to discover that for it, but he wouldn't soon forget it.
only a few drops remained in the basin-just
enough to make him even thirstier. But there was Sweetness drank and drank. And then he held the
a lever, and Sweetness suddenly realized that this lever while the stranger drank. Over the next few
held the answer. He must change his thinking; he days, some people even took pictures of them as
must operate the switch. they operated the water fountain for each other
Cautiously, he stepped onto the lever. Water The arrangement lasted until Sweetness finally
streamed out of the spout and immediately got his bearings, and he returned Home.
disappeared down a drain. Sweetness jumped The sweetness found Home to be, unlike himself,
into the basin, supping the few remaining drops. the same. He would always yearn for Home
What torture, to have only a fraction of what he when he was flying, but now the world was
needed! He ended up performing a desperate stretched open in front of him. When he flew, he
dance: hopping from lever to basin, supping a flew to places where he could feel more than the
drop or two, then hopping back to the lever. magnetic pull of the poles. He flew to other
When another of his kind alighted on the edge of birds, strange birds, to people. Not all of them
the drinking fountain, Sweetness nearly fell off liked him. And some made him feel horrible. But
the lever. The stranger, who was not lost herself, for the very first time in his life, Sweetness
began to ask why.
THE HEAT IS ON
We’ve all heard of global warming, but just how much warmer
will the earth get and how will it affect our lives?
Almost everyone has some idea of what global environment and humans, climate changes have the
warming is all about, but no one is quite sure about its potential to affect the geographical location of
consequences. A warmer climate is likely to mean ecological systems, such as forests and grasslands, the
changes to the weather in all parts of the world. And mix of species they contain and their ability to provide
since the atmosphere is intimately linked to every the various benefits on which societies depend for
aspect of the planet on which we live, any changes to their continued existence. Thus, the whole range of
the climate will have significant knock-on effects for resources on which we rely is sensitive to changes in
plants and animals, as well as water and soils. climate. This includes food production, water
resources, and human settlements. The effects, some
We humans have learned to use such natural resources of which are potentially irreversible, are likely to be
to our advantage, enabling us to produce food, build unfavorable in many areas.
great cities and support six billion members of the
human race. Any changes to these resources have to But this would not be true for all. Indeed, some
be taken seriously. The problem we have is knowing climate change impacts will probably be beneficial.
just how the world will change, and what is causing Scientists in some countries have already identified
these changes. useful environmental trends that are closely linked to
the warming that has occurred to date. In Australia,
There is no doubt, for example, that over the last 100 for example, research has shown that the average
years or so, human activity has significantly increased yield of wheat has increased by about 0.5 tonnes per
the atmospheric concentrations of several gases — the hectare since 1952, and climate trends have played a
so-called greenhouse gases — which are closely significant part in this greater food production.
related to global temperature. It seems likely that
these increased concentrations, which are set to The most important climatic factor observed, in this
continue building up shortly, are already affecting case, was the rise, in recent decades, of the minimum
global climate, but our poor knowledge and temperature. Among other things, the warmer
understanding of the global heat balance make the temperatures have meant fewer frosts, and this has
current and future situations uncertain, what we do caused less damage to harvests. However, many of the
know is that atmospheric concentrations of predicted effects of future climate warming are far
greenhouse gases have fluctuated in close harmony from beneficial. Relatively small climate changes can
with global temperatures over the past 40,000 years. influence the availability of water, either due to long-
term drying of the climate or by increasing the
This would indicate that the two are almost certainly frequency of droughts. Associated problems are likely
related. Evidence gleaned from a range of sources to arise first in arid and semi-arid regions and more
suggests that our planet has warmed at the surface by humid areas where demand or pollution have already
about 6°C over the past century. Most scientists think created shortages.
that this trend is unlikely to be natural in origin and is,
in part, a result of human pollution of the atmosphere. The Mediterranean Basin is one example of this and
A lot of research has gone into predicting the in recent decades decreasing trends in precipitation
conditions that will result from higher global totals have already been identified in western-central
temperatures. Most of this research uses computer parts of the basin as well as marked changes in
programs known as general circulation models or seasonality. A clear tendency for rainfall to be
GCMs. They run on powerful computers, use concentrated into a shorter period of the year has been
fundamental laws of physics and chemistry to analyze noted in the Alentejo region of southern Portugal,
the interaction of temperature, pressure, solar with the proportion of annual rainfall falling in
radiation, and other climatic factors to predict climatic autumn and winter, increasing at the expense of spring
conditions for the past, present, or future. totals.
Unfortunately, they are simplifications of the real A further undesirable effect is likely to be changed to
world and have numerous deficiencies. Their results glacial processes. This will impact glacier ice, ground
are only approximate and they are also slow to run ice, and sea ice, which in turn will affect vegetation,
and expensive to use. Part of the problem is that we wildlife habitats, and human structures and facilities.
do not understand fully all the processes of the Indeed, there is a strong possibility that the Arctic’s
climatic system, although we do realize its ice cover will melt completely, making marine
complexity. transport and oil and gas exploration easier but
increasing the danger from icebergs. But probably the
Despite these research difficulties, most people agree most dramatic and visible effect of global warming in
on perhaps the most important aspect of climatic the twenty-first or ‘greenhouse’ century will be the
change from the viewpoint of contemporary human rise in sea levels. This will be caused by the thermal
societies: the rate of change will be faster than expansion of the oceans - warmer water occupies a
anything we have previously experienced. In this case, greater volume than cold water — and the added input
the approximate predictions produced by the GCMs from melting ice. With scientists calculating that
are being used to gain some insight into the nature and about half of the world’s population lives in coastal
conditions of the world that we will inhabit over the zones, the consequences of rising sea levels are
next few generations. potentially very severe.
Currently, they suggest that the average annual global Increased flooding and inundation are the most
surface temperature will increase by between 1°C and obvious results, with London, New York, and Tokyo
3.5°C by the year 2100; that the average sea level being just a few of the candidates for significant
around the world will rise by 15 - 95 cm; and that disruption. Huge numbers of people stand to lose their
changes in the spatial and temporal patterns of homes and livelihoods and this could produce many
precipitation will occur. Scientists also expect extreme millions of environmental refugees. Arguably the
weather conditions, such as heatwaves, floods, and most severe consequences would be experienced by
droughts, to become more frequent in some places. several small, low-lying island states since entire
countries could cease to exist if worst-case scenarios
These forecasts should leave us in little doubt about are realized. The consequences would be devastating,
the potential impact of climatic change on the natural not only for the people and culture of these islands but
also for the countries that would need to accommodate those who had been displaced.
• Read the following passage . Decide if the statements from 1 to 4 are True or
False, and choose the correct answers (A, B, C or D) for the questions 5 - 8.
My name is Tsukasa Richard Nakai. I am 33-years-old and I am an avid Star Wars fan! My
favorite character is Darth Vader. I’m currently involved in the Best Buddies Friendship
Program, Ambassador Program, and the Jobs Program.
I’ve been part of the Best Buddies chapter at UCLA since February 2017 and I am currently
their Buddy Director. I’ve been an ambassador since March 2017, and through the Jobs
Program, I now work as a part-time employee in the Best Buddies California State Office as an
administrative assistant… I love it!
My life before joining Best Buddies was one of self-loathing. In October 2000, I had been
diagnosed with Asperger syndrome. I had never fully accepted who I was as a person and I was
always very self-critical. Despite my achievements, including becoming a published author and
a college graduate, I would never praise myself or give myself a kind word. The one person who
helped me on my journey of self-acceptance was one of my colleagues, Tori Braden. When she
was Chapter President at UCLA, she asked me if I wanted to become an Ambassador. I replied,
YES! — thinking that I wouldn’t get asked to do too much, but I was mistaken!
One month after I participated in the Ambassador Training at UCLA, the Program
Supervisor of our area called me and asked me if I wanted to do a speaking engagement for one
of our sponsors at the Best Buddies Challenge: Hearst Castle! It wasn’t until I started interning
at Best Buddies state office in California, that I found out Tori had advocated on my behalf.
This was unusual; we barely knew each other, but she took a huge risk on me and it paid off for
both of us.
+ QUESTIONS:
1. Nakai has been participating Best Buddies chapter at UCLA since February 2017, then became
ambassador in March 2017 and adopted Jobs Program. ____True_____
2. Tori Braden was diagnosed with Asperger syndrome in October 2000 while she was the Chapter
President at UCLA. _____False_____
3. Richard Nakai was called by our regional program supervisor and asked if he would like to give a
speaking engagement. _____True_____
4. Nakai once achieved becoming a published author and a college graduate although he never
complimented himself or did himself a single word of kindness. _____True_____
5. What is Richard Nakai's favorite program?
A. Best Buddies Friendship Program B. Star Wars
C. Ambassador Program D. Jobs Program
6. Why doesn't Richard Nakai completely accept who he is?
A. Because he had an inferiority complex about his achievements.
B. Because Nakai didn't achieve a presentation for one of our sponsors at the Best Buddies Challenge.
C. Because he believes that he won't be required to do too much.
D. Because he has been diagnosed with Asperger's syndrome and including his achievements.
7. What does the word "ambassador" in line 5 mean?
A. An individual who is represented and carries out the functions of a given organization.
B. The highest-ranking diplomatic officer is appointed to represent a country.
C. A person who freely offers to take part in an enterprise or undertake a task.
D. A person sent on a special mission, usually as a diplomatic representative.
8. The word “self-acceptance” in line 9 is nearest in meaning to:
A. self-discovery B. self-rejection
C. self-approval D. mortification

During my time in medical school, in all the circumnutation I went through, it felt like something
was missing — until I did my anesthesia rotation. Throughout this rotation, I was revealed to simple,
low-cost, and seemingly mundane interventions that made big and occasionally life-saving differences
in a matter of seconds. For instance, a patient with an obstructed airway required only simple airway
maneuvers like a jaw thrust or the insertion of an oral airway to relieve the obstruction and help them
breathe better.
This exposure was very critical to my decision to choose anesthesia as my career, as I just had to be
part of this life-saving action.
Now, as an anesthesiologist, I am part of a team that works with patients who come in with life-
threatening conditions and our interventions help them pull through their illnesses. This is probably the
most rewarding aspect for me as an anesthesiologist—walking patients through their entire surgical
experience and transforming their anxiety into happiness by managing both their pain and expectations.
One of the most challenging aspects of the work is knowing that if I had access to the patient earlier,
their outcome could have been different. There are many reasons why patients don’t receive the care
they need and deserve. It is extremely frustrating knowing that there was lost time in the chain of care
due to so many different circumstances. These are challenges that we are endlessly trying to solve.
Part of solving these challenges is advocacy. I have been a member of the Association of
Anesthesiologists of Uganda since its inception and currently work in the capacity of treasurer. As an
association, we do a lot of advocacy work for safety standards. We are currently developing guidelines
that will be proposed as national guidelines for safe anesthesia delivery in Uganda. The association also
advocates for funding from the government for the training of more physician anesthesia providers as
our numbers are a far cry from the Lancet Commission for Global Surgery’s recommendation of the
surgical specialist density of 20 per 100,000 population.
• Read the passage and circle the correct words:
A. Industrial farming methods are commonplace, but the philosophy of organic farming could
be about to change the way many of us eat. Consumers have (1) dangerous / grave concerns
about eating fruit and vegetables that have been grown with the use of pesticides and chemical
(2) fertilisers / additives. The use of chemicals might make the crops immune to (3) epidemics /
infestations, but consumers have started to worry about the (4) adverse / modified effects that
such farming techniques might have. Organic farming could offer a solution to this problem.
Farmers are able to cultivate (5) abundant / plenty crops using natural fertilisers. The same is
true for farms involved in the meat and (6) dairy / milk product industries. Farmers can use
organic feed for their (7) livestock / meat instead of giving them harmful processed foods.
Organic farming might also avoid some of the (8) hygiene / wellbeing issues that are connected
to industrial farming, as well as giving people better quality meat to choose from.
B. A study in the late 1960s demonstrated how children can become more (9) principled /
neurotic and self-disciplined if they resist temptation regularly. A single marshmallow was
placed in front of a child to monitor how they would (10) exert / conduct themselves in tempting
situations. They were told they could eat that marshmallow immediately or have two if they
waited 15 minutes. Most children said they would wait, but they couldn't resist and polished off
the sweet quickly. The only way some children could keep a cool (11) face / head and avoid
eating the marshmallow was to look away or cover their eyes. Researchers kept track of the
children as they aged, and the ones who couldn't wait were more (12) compatible / vulnerable to
problems such as obesity and were also more (13) tempted / prone to suffering from drug
addiction and severe behavioral (14) disorders / dysfunctions in adulthood.45
C. Mary’s Meals is a charity providing food to over one million schoolchildren in developing
countries, but (15) not only / no sooner does the nutritious meal fill their bellies, but it is also a
motivation to attend school in the first place.
(16) It is rare / Seldom has a single charity has done so much to help so many people. The
teachers themselves are amazed at the difference in their pupils. (17) ‘Never / Often have I felt
and seen such a change in my pupils. They used to be so tired and hungry, they found it
impossible to learn, yet not once (18) I heard them / did I hear them complain. Not until they
started having a hot meal every lunchtime, (19) did they start / they began to be lively and keen
to contribute and participate in class,’ says one primary school teacher. The projects continue to
grow. (20) ‘As soon as / No sooner has one project been set up, than we are asked to do
something in a neighboring village or town,’ comments one of the volunteers.
The food is sourced locally and cooked by volunteers, often relatives of the children. ‘Not
in a thousand years (21) I would think / did I ever think a meal a day could bring such a change
to the lives of these kids, but not until every child has a meal every day (22) can we really stop /
we will never stop our work,’ another volunteer adds.

19, The recurrence of fever following an initial defervescence indicates a septic or non-septic
embolic event, a drug hypersensitivity reaction, or the emergence of a resistant strain. SENSITIVE
I. SENTENCE:
1, Vietnam's coastlines and islands are currently experiencing difficult changes that seriously
threaten the country's territorial sovereignty. (EXPERIENT)
2, During the second half of the 19th century, progress was composed by the use of spectroscopic
and photographic technologies for astronomical investigations. (PHOTOGRAPH)
3, In 1947, the proclamation of independence and "fighting hunger and ignorance" campaigns
started under the framework of the New Democratic Republic of Vietnam. (DEPEND)
4, Following the migration and sea crossing of the Vietnamese after 1975, Banh Mi arrived in
numerous nations and gradually became popular all over the world. (GRADE)
5, The production of huge quantities of air pollution outside of the home appears to be important in
metal forging. (PRODUCE)
6, Furthermore, in the motion of bodies, there seems to be no teleology, as the whole process could
be reduced to collisions specified by law and their continual rebuilding. (BUILD)
7, The biological or instinctive impact is being steadily decreased as humans gained the knowledge
to alter the natural environment themselves. (BIOLOGY)
8, The vast majority of Vietnamese people recognize the significance of protecting the environment,
but most are nevertheless willing to ditch waste into the streets. (SIGN)
9, As a result, when we are exposed to faces of different colors, racial segregation might impact our
picture processing and the way our minds operate. (SEGREGATOR)
10, Despite certain legal measures, women currently have lower average wages than males in the
majority of nations throughout the world. (MAJOR)
11, Some sociologists consider that the proximity or the difference between culture and civilization
reliances on the substance of the culture and civilization. (SOCIAL)
12, As previously stated, the discussion of an overborne will has resulted in the criminal law
conflating the concepts of moral blameworthiness and moral involuntariness. (VOLUNTEER)
13, Car owners should have their own formally approved emissions regulations if attainable so that
they can ensure unsatisfactory results from any police allegations. (SATISFY)
14, His preference for working on cardboard with quick-fire liquid paints, and for leaving large
expanses of this cardboard untouched, contributes to the sense of incompleteness. (COMPLETE)
15, Eventually, someone is airing the ordinary public's grievances about the outrageously high
prices of goods and services in the media. (OUTRAGE)
16, An associate who obtained a letter of recommendation from him, a graduate from his school,
conveyed an important certification of the sheepskin to the realm of trained animals. (COMMEND)
17, In order not to allow physical limitations to hold down the business, new storage technologies
are emerging to match rapid digitalization and worldwide information exchange. (DIGIT)
18, Nonetheless, as a result of various discrepancies on the concepts of divine predestination and
election, many people regard these schools of thought as fundamentally opposed. (DESTINE)
Bake sales and potluck dinners are various events, even so, they are similar because people make
their contributions. Baking is usually a fundraiser held by a community organization such as a
school or a charity group. In the community, people generally volunteer to buy goods like cookies
or pies for baked products. The activity is announced in the community and the funds gained from
buying baked goods go to the cause. A potluck dinner is a dinner with a group of people bringing a
different homemade dish. One of the main ideas behind both events is that people will be able
to homemade food from each other. The bake sale or potluck should not include store-bought items
or dishes.
It's a long-held assumption that bake sales and potlucks only feature homemade goods. Bake
sales became popular in the mid-twentieth century, when large kitchen appliances including
refrigerators and electric ovens became more common in American kitchens, making it easier for
people to bake on a constant schedule. Bake-sales promoted cross-cultural exchange by allowing
people from different backgrounds to share traditional dishes at a time when cookbooks were more
limited than they are now. Potlucks, on the other hand, can be traced back to the sixteenth century.
The term described the meal prepared for unforeseen guests, and the "luck of the pot" determined
the meal of the guests. Even though the potluck is evolving, it still should focus on the sharing of
homemade designs.
The spirit of the tournament in opposition to the event while people sell a boxed, fantastic cake
for a baked sale or a prepared entrance to the potting. No matter if you open yourself to it or
transmit items you have bought to yourself; you violate the trust of your community by disregarding
tradition. As mentioned above, baking and potted goods have a long history of homemade products.
Consequently, people are waiting for food cooked with love and generosity. The attendants feel
treated and unconsciously opposed to what they should feel after a community event when offered
mass-produced food.

II. WRITING
PART 1: (B2) [184 words]

Childhood for many people was a time of adventure. They spent a lot of time out in the fresh air,
running around, climbing trees, and playing games. But it’s known that these days, children don't spend
as much time outside, and some people are concerned that this is becoming a problem.
It’s true that when children are outside, they do lots of physical activity like sports, and this is good
for their health. It helps their muscles get stronger, and it can also help them make friends. On the other
hand, children often fall and hurt themselves. It’s also true that the ‘fresh air’ probably isn’t very fresh
at all because of traffic pollution.
Individually, I don’t think that school teachers and parents should be too worried if children are
spending more time indoors. Many children and teenagers spend a lot of time looking at screens, but
this has a lot of benefits. It’s also wrong to assume that time spent outdoors is healthy and that time
spent indoors is unhealthy. Over time, lifestyles change, and we need to accept that this is true.

PART 2: (B2) [184 words]


Hi Steve,
What a surprise to hear from you after all this time! It sounds as though you’re busy with your course,
and of course, I'm glad to answer your questions.
Essentially, the parks in my country are in poor condition on the whole. New plants and trees are rarely
planted, and tile grass isn’t cut very often. It’s such a shame, because most parks are used quite a lot,
especially by families at weekends, and people play a lot of football there.
According to the law (as far as I know), the government owns the parks and is responsible for the
maintenance. But die thing is that there isn't absolutely enough money to pay for all the schools, roads
and hospitals, as well as keeping the parks in good condition.
What I’d like to see is community action, and people offering to clear up the parks in their own time,
plant frees, and that sort of thing. Oh well, maybe one day!
Anyway, good luck with your research, and stay in touch.
Say ‘hi’ to your family from me.
All the best, Diana

PART 1: (C1) [383 words]


If I was to be given the opportunity of traveling back in time, I would certainly stop in Britain in the
time of the reign of the first female English queen ─ Elizabeth 1.1 have always been fascinated by this
extraordinary historical figure who had such significant influence on this country and Its history.
This decision of my past time destination occurs to me quite naturally for history is one of my life-
long interests, especially the period around the 15th and 16th centuries. Although I am aware that living
in those times was not as comfortable and easy as it Is nowadays, thanks to all the inventions of modern
life, I would love to explore the spirit of life centuries ago. Living in England in the 16th century might
take me to London, where Shakespeare just opened his Globe.
Watching one of his famous plays I could catch a glimpse of the queen, dressed in the clothes of an
ordinary woman, surrounded by her servants. She was said to attend the theatre In disguise to amuse
herself and escape from the formality of her royal court.
My next destination could be a lively market, where I would watch the hassle of the everyday life of
ordinary people coming there to sell handmade pottery and fabrics or buy some fresh eggs and bread.
My main interest would be the queen, as I mentioned before. Having seen a great documentary based on
her life recently I can’t help wondering how this subtle woman managed to rule her country
successfully for so many years. She must have been a person of magnificent character and skills if she
was to lead the country In those unsettled times of continuous wars against Spain and France. She
defeated the rebellion initiated by her cousin Mary and by many others, who didn’t like to see her on the
throne. Surprisingly, she had never got married. It might have been a part of her strategy how to keep
the fragile peace. But it would be her devotion for the nation and her people’s welfare as she once
proclaimed. That is why I want to travel to the Elizabethan times and meet this fantastic person whose
strength and personal experience could teach me a lot and whom I cannot help to admire.

PART 2: (C1) [265 words]


Dear Ashley,
Thanks for your message – it’s great to hear from you. And sorry not to get back to you until now, but
I’ve been having a quiet weekend: out in the countryside!
Unfortunately, it’s not been so relaxing at home. Far from it. Ever since he moved in, our new flatmate
Charlie has refused to tidy up, left both the kitchen and the bathroom in a terrible state, and – to cap it
all – has been keeping me awake until all hours with some of the worst music I’ve ever heard.
So the other week Jamie (my other flatmate) and I had a word with him. Charlie admitted he’d done
almost no housework, said he was sorry, and explained he hadn’t certainly flat-shared before. He
promised he’d make more of an effort in the future, though as yet our chat seems to have had little
effect.
I got the impression he’s a bit down. He positively didn't want to talk about it, but I wouldn’t be
surprised if he’s missing his family, both to talk to and to tidy up after him! I also wonder whether he’s
feeling slightly left out – Jamie and I get on extremely well, and three’s rarely the ideal number.
So what I’d like to ask, firstly, is whether you think there might be something else that’s bothering him,
and also how we can make him see we want to be friends with him. And finally, how do you suggest we
get him to turn the music down?
Hope to hear from you soon.
Best wishes, Lee

I. WORDFORM: (GRADE 9)
1, Mouawad designers have drawn inspiration for this unique design from the principles of strength,
women's empowerment, and communities that are associated with the Miss Universe organization.
(INSPIRE)
2, Finger-rings and ear-studs, stone necklaces and ornamental chokers, brightly-hued bracelets, and
innovatively shaped pendants are there in rich profusion. (INNOVATE)
3, The announcement comes as a result of a national survey, which revealed one in five new
appointments for secondary school teachers to be unsatisfactory. (SATISFY)
4, In 1995, the proposal and unique approach for the development of the major sites were created by
a combination of Virginia preservationists, historians, and tourism authorities. (PRESERVE)
5, Until previously, tales of persons declining Queen Elizabeth's accolades have been based only on
leaked lists or information provided by the nominee. (NOMINATE)
6, The weights of animal carcasses likewise show progress, with the increase in the size of lambs
and calves the best indicators of productivity improvements. (PRODUCE)
7, The French and the Spanish captured numerous West Indian islands effectively and resembled to
be on the approach of ousting the British entirely. (EFFECT)
8, In the case of subsidized schools, the main costs such as teachers' salaries and building
maintenance are borne completely by the Flemish government. (COMPLETE)
9, European legislators are called upon, through pollutants pumped into the open ocean, to tackle
the luxury liners threatening marine life. (POLLUTE)
10, Nevertheless, an attempt to reclaim Malta ended disastrously by supporting the Arabs and
massacring the Byzantine garrison among local populations. (DISASTER)

BLUE DIAMONDS
WORDFORM(B2 – C1 LEVEL)

It is sometimes said that ‘Your schooldays are the happiest days of your life, and people often
feel that this should be a period of enjoyment. However, exams often affect students’ happiness,
and many students express a (1. PREFER) __________ for alternative methods of assessment,
where the work they do throughout the year counts towards their final mark. They say that
exams test short-term memory and (2. KNOW) ___________, which is forgotten immediately
after the exam. Moreover, assessing coursework as part of the final mark changes students’ (3.
BEHAVE) ___________, making them more responsible about studying. There are some
students, however, who prefer final examinations, saying that in (4. COMPARE)
__________ they only have to work hard for two months a year and so they have more time for
their leisure (5. ACT) __________ They say that some students receive (6. ASSIST)
__________ with their coursework from their parents, so it is not an accurate (7. MEASURE)
__________ of how hard they have worked or of their real (8. ABLE) _________ in the subject
they are studying.

ANSWER SECTION: __________

I want to talk today about the real (1. MEAN) __________ of living in a yurt. Since my last
article ' building our house,' there has been a time since my last article 'building our house' has
taken us a long time to organize everything. We had no time to (2. CARE) __________ move
into our new cycle. Winter was just around the corner when we went in with plenty of rain and
wind, followed by two strong weeks of snow. The roof was drained, therefore we needed to
make it as safe as possible. I was afraid that was only the start of worse things to come. But (3.
FORTUNE) ___________, since we have had no (4. DIFFICULT) ___________, so I can't
complain!
Therefore, let me answer the first question people ask us: ‘Aren't you cold?' Well, the answer is
no. I know it looks like a tent, but the thick sheep's wool (5. INSULATE) __________ keeps the
heat in, and our tiny burner heats up (6. RAPID) __________ as well. I can't deny that the
mornings are chilly - they are. But we deal with that by getting up early and getting things done.
Mark starts the fire, and I fold the (7. CONVERT) __________ bed to make more room, before
going for a stroll with the dog. I collect some wood from the shed on my way back to (8. SURE)
__________ that we have enough for the day. When we get back, the yurt is beautiful and warm,
and we sit down to eat breakfast.

ANSWER SECTION: __________

As a language student, you have probably used (1. ART) _________ intelligence in numerous
ways. You may have taken an adaptive (2. PLACE) __________ test to join your class or
practice your grammar with the help of an online learning platform. However, you have
probably felt the (3. LIMIT) __________ of relying solely on technology for feedback on your
progress, particularly when it comes to the (4. PRODUCE) __________ skills of speaking and
writing. How far can a computer help you with those?
If you are studying English - or any other language - in a class with a teacher, the chances are
you have been asked to give a presentation to the rest of the class. It is also likely that the (5.
PRESENT) ___________ you have given have been followed by (6. FEED) __________, first
from the other students and then from your teacher. Fortunately, you found this to be a pleasant
experience. Even then, you may have thought the response was either (7.
ACCURACY) ___________ or too (8. SUBJECT) __________ .

ANSWER SECTION: __________

Every year in August, Scotland's capital city plays host two festivals: the Edinburgh
International Festival and the Edinburgh Fringe. So what's the difference? To perform at the
Edinburgh Festival you need an (1. INVITE) __________ from the organizers. These are only
given to (2. EXCEPT) __________ musicians, singers, or theatre companies. If you attend the
main festival, you will have the opportunity to see an (3. IMPRESS) __________ range of
concerts and plays. Performers at the Fringe are not invited but they turn up (4.
REGARD) __________ . Whereas the main festival is a serious event with ballets, operas, and
plays, the Fringe is a (5. DELIGHT) __________ mix of comedy and shows put on by
amateurs. There are no formal (6. REQUIRE) __________ for taking part: just come along!
Every year a few (7. HOPE) __________ will be talent-spotted at the Fringe and given the
chance to join a top theatre company or orchestra. Although you rarely have to pay more than a
few pounds to attend a Fringe show, it is actually more (8. PROFIT) ___________ than the
main festival!

ANSWER SECTION: __________

Yoga has long been considered one of the best exercises around, and one that can be practiced
(1. VIRTUAL) __________ anywhere, including at home. Hot yoga, as the name suggests, takes
it a stage further by turning up the heat, (2. REQUIRE) __________ a temperature of around
40°C and (3. HUMID) _________ of 60 percent. Distinctly, such a change is going to make you
sweat more, and this is the whole point as it is supposed to eliminate harmful (4. CHEMISTRY)
_________ and toxins from your body. In a nutshell, hot yoga retains the mental focus of the
more traditional version of the discipline but is designed to push your body much harder. The
heat increases your heart rate and helps thin the blood, stimulate your (5. METABOLIC)
_________ and burn calories at a faster rate. As well as increased strength, stamina and (6.
FLEX) _________, (7. PRACTICE) _________ point to several other health benefits, such as
curing back pain and improving the skin, with many even claiming it has boosted their (8.
PERFORM) _________ in other sports.

ANSWER SECTION: __________

When asked to explain their fascination with their hobby, some birdwatchers might mention the
simple (1. PLEASE) _________ of getting out and walking in woods and noting what birds you
come across. But the authors of a highly (2. ENTERTAIN) _________ new book, The
Attraction of Bird Watching, describe a particularly thrilling occasion when they were (3.
DESPAIR) _________ to see an extremely rare visitor to the shores of Maine in the USA. They
compare the (4. EXCITE) __________ of preparing and researching for this expedition to that
of extreme surfers, who eagerly chase the big wave, wherever it might occur. There is the same
sense of anticipation, the (5. CARE) _________ checking of internet sightings and (6. REFER)
_________ guides, and above all the sense of having to be ready for (7. SOME) _________ that
might appear only very briefly if indeed it appears at all. The authors attempt to give an (8.
EXPLAIN) _________ for this feeling and speculate that it is perhaps a throwback to what
primitive hunters would have felt.

ANSWER SECTION: __________

I genuinely thought that I'd get Shirley's part. It would be an (1. UNDERSTATE) __________
to say that I was upset that it was handed to Jodie. I was certain I was more appropriate to the
part in every aspect than she was. My long, black hair and sports builds were (2.
CONSIDER) ___________ closer to what Jodie's (3. CLASS) ___________ blond features (4.
PORTRAY) __________ in this story. And it was a piece of music – that was why I was
supposed to have a part of the music – Shirley had to sing.
Further (5. COMPOUND) __________ problems, I'd been invited to portray one of Shirley's
pals. Aside from Shirley, the other major roles were all played by men, which didn't concern me.
I'd (6. CHEER) __________ put on a fake beard, but all the other vital pieces had been taken by
the lads. Rehearsals lasted eight weeks (7. THROUGH) __________ the hottest portion of the
summer. I'll admit that I wanted Jodie to fail at first, but as time passed, I realized she wasn't that
awful after all. She had a better recall than I had, so I believe she had taken some singing classes
since our previous show. I struggled with my lines, but she always had them memorized (8.
PRECISE) __________ .
It occurred to me that this may be why I hadn't been given a larger role. I discovered that (9.
APPEAR) __________ aren't everything, even in the theater. That summer was more enjoyable
than I'd expected it to be. Our (10. DIRECT) ___________, Matthew, made the practices
pleasant by providing us with ice cream and telling us anecdotes regarding his
(11. PROFESS) ___________ acting career. We used often hang out in the park or at the beach
after (12. HEARSE) ____________ since all of the performers got along so well. I gradually
continue to embrace myself. At the very least, I'd be able to spend my evenings relaxing with the
audience rather than (13. MEMORY) __________ my lines.
When the call came, I was at home. It was the night before the game opened and I was in my
room doing my hair. My mother called up the steps that were online for Jodie's father, Duncan
Parsons. I rushed (14. STAIR) __________ and took my phone, I raced in the heart, why did
Jodie's father contact me on the fence? Jodie phoned my mobile every time. Perhaps it has
happened something very (15. REPEL) __________! Mr. Parsons tells me that when she came
home, Jodie fell off her (16. MOTOR) ___________, she would be well but she hurt her
shoulder.

ANSWER SECTION: __________

I. WORDFORM:
1, For the time being, the government will not engage in _____________________ about green
measures since it is awaiting the results of the most recent research. (DISCUSS)
→ discussions
2, Because he was homeless and hungry, he stole the bread; a proper _____________________ would
be a simple warning. (PUNISH)
→ punishment
3, My _____________________ would be extremely irritated if I neglected a fellow human being who
was in distress. (CONSCIENTIOUS)
→ conscience
4, In terms of the subjects in the field of marketing, I believe the selected study materials are rather too
_____________________ . (THEORY)
→ theoretical
5, In recent years, we've been hearing a lot about the rising menace of _____________________, along
with dire predictions that the rich pattern of local life is being eroded. (GLOBAL)
→ globalization
6, Butlers may also suffer isolation and cultural _____________________ with their bosses, and they
may be compelled to work for people who aren't always pleasant. (DIFFER)
→ differences
7, A recent study figured out that _____________________ liars are just as likely to be self-confident,
attractive, and popular as they are introverted and withdrawn. (PATHOLOGY)
→ pathological
8, A ______________________ photo on a person's social media profile might make all the difference
these days when it comes to obtaining a top job. (HUMILIATE)
→ humiliating
9, Either as part of a movement to combat ____________________ (EQUAL) or as anonymous whistle
blowers who exposed the ____________________ (LAW) activities of companies and governments,
they are individuals who are taking action.
→ inequality / unlawful
10, Many young people nowadays want to live in large cities for a variety of reasons, the most frequent
of which being _____________________ and education. (EMPLOY)
→ employment
• Use the word given in capitals at the end of some of the lines to form a word that fits
in the space in the same line.
Unlike many proverbs, the one about elephant memory is (1. SCIENCE) …….....… accurate. Elephants know
every member in their herd, able to recognize as many as thirty companions by sight or smell. It's tremendous
(2. ASSIST) …….....… when you migrate or meet other potentially (3. AGGRESS) ……......… elephants.
They further remember and (4. DIFFER) ….…...... specific signals of danger and may remind you of crucial
areas long after your previous visit. Nonetheless, it is the most intriguing memory (5. RELATE) ….…....... to
survival.
Elephants not only remember their herd friends, but also other animals that produced a (6. SIGN) ….…..........
impact on them. In one case, two circus elephants who had briefly performed together twenty-three years before
(7. LIGHT) ….….......... when they crossed paths again. This (8. RECOGNIZE) ….….......... extends
beyond other members of their species. Elephants have already identified individuals with whom they had
connected after decades apart. All of these demonstrates that elephant memory expands beyond stimulus
reactions.

KEY:
1, scientifically 2, assistance 3, aggressive 4, differentiate
5, unrelated 6, significant 7, delighted 8, recognition

We comprehend that organic material and oxygen are (1. COMBUST) __________ by fire, but don't you
still wonder what fire is (2. OCCASION) ___________ . Why are blue flames of gas and orange wood fire?
What's so (3. FASCINATE) __________ about why flames move? Chemistry can tell us combustion
recesses, but all physics is the light show. The trouble spots (4. STIMULUS) __________ the molecules to
emit light – often pale blue – from the atomic transition when a flame delivers precisely as a gas flame,
blowtorch, and the basis of the candle.
That's quantum mechanics. Presently, if the fuel isn't almost as pure and thus does not burn like wood or
coal-fire or candle fire, there is some blue light still, but you are not able to see it even though it has a light out
of all the soot and smoke (5. PART) ___________ . They are red-hot glistening! Why, indeed, are heated
items glowing? A (6. PHENOMENA) ___________ entitled black body radiation makes all things,
depending on temperature, shine in color light. And yet because we're too cool to glow with visible light, you
don't notice your buddies shining. Infrarot, we shine. But lava, hot iron or soot in a flame, is hot enough to shine
from that well-known red-orange light.
And why are flames fashioned like writhing tongues crawling skywards? Gravity! The Earth's (7.
GRAVITY) ___________ pull causes heated air to ascend, and this convection molds flames into their (8.
CHARACTER) ___________ shape. When a match is lighted in zero gravity, the flame expands outwards
like a balloon. As there is nothing to instruct it which way to travel, it will move in any direction.

KEY:
1, combustible 2, occasionally 3, fascinating 4, stimulate
5, particles 6, phenomenon 7, gravitational 8, characteristic
The pathways of toad tadpoles hatch in a small oasis in the deserts of the American Southwest. They are not
able to survive without water until they become toadlets, although these pools are temporary and fast (1.
EVAPORATE) …....… The tadpoles are growing up against the clock before their kindergartens vanish.
Some of the brood explodes in size almost overnight. You utilize your bowel-to-lamp-to-lamp teeth and your
massive jaw muscles for (2. CONSUME) ….….... . They develop faster, nourished by this (3. ADD) ….
…...... fuel, allowing the pond to stop flowing before it.
The spadefoot toad is far from the only animal that consumes members of its species as part of its natural life
cycle. All of these animals do. If that surprises you, you're in good company. (4. CANNIBAL) ……….....
was considered to be an unusual response to hunger or other acute stress until recently. Well-known cannibals,
such as the praying mantis and black widow spider, were thought to be peculiar outliers. But we now
understand exactly that they should be, for the most part, (5. PRESENT) ……...... of the rule.
Despite the fact that feeding individuals of the same species may appear (6. PRODUCT)
……........, cannibalism can encourage the survival of the whole species by decreasing (7. COMPETE) …...
…......, cultivating the weak, and strengthening the strong. In reaction to environmental constraints, certain
animals like the spadefoot toad cannibalize. Their perilous condition, but they are not inclined to cannibalism as
a last attempt to prevent hunger. Furthermore, this means that an era in which they are particularly vulnerable to
predation or harmful environmental circumstances is more rapidly overcome. During fodder behavior, other
animals, especially many fish, cannibalize (8. CRIMINATE) …...…...... .

KEY:
1, evaporating 2, consumption 3, additional 4, cannibalism
5, representative 6, counterproductive 7, competitiveness 8, indiscriminately

You understand exactly what concerning your festival "The Day of Vesak". It is indeed a global day of
Buddhists (1. MEMORY) ….…...... Buddha's birth, lighting, and nirvana. On this dawning day, Buddhists
assemble in temples for a prayer ceremony and give candles, flowers, and joss sticks. They would further
celebrate joy, serenity, and (2. INTROSPECT) ………..... on this day. The burning out of candles and sticks
of joss and the (3. WILT) ….……... of flowers during the day are being used to remind all adorers that life is
passing away.
Bathing a Buddha statue is one of the most (4. FAMILY) ….…....... rituals. The rest of the day is spent
performing good actions including blood donation, visiting the elderly, and delivering presents to those in need.
The day is (5. FREQUENCY) ………..... concluded with a (6. CANDLE) ….…....... parade through the
streets. Devotees may be observed during the two-hour, three-step, one-bow ceremony, in which they pray for
peaceful (7. EXIST) ….……..., personal blessings, and (8. REPENT) …….…... .

KEY:
1, commemorating 2, introspection 3, wilting 4, familiar
5, frequently 6, candlelight 7, coexistence 8, repentance

Whether chained to a wheel, converted into a spider, or an eagle eating his liver, Greek (1. MYTH) ….
…......... is replete with the stories of the gods that inflict dreadful tragedies on dead people. Nonetheless, one of
its most well-known penalties is not renowned for its horrific (2. HARSH) …..……....., but its unnerving
(3. FAMILY) ….….......... Sisyphus, now known as Corinth, was the first monarch of Ephyra.
Although a competent ruler who prospered his city, he was likewise a cunning tyrant who seduced his niece
and murdered guests to demonstrate his authority. The gods were enraged by this violation of the sacred (4.
HOSPITAL) ….…...... custom. But if it hadn't been for his reckless optimism, Sisyphus could have escaped
(5. PUNISH) ….…...... . When Zeus (6. NAP) ….…....... the nymph Aegina and carried her away in the
guise of a massive eagle, the problem began. The river god Asopus, Aegina's father, pursued their trail to
Ephyra, where he encountered Sisyphus.
The monarch revealed Asopus which direction Zeus had taken the maiden in exchange for the deity creating
a spring within the city. When Zeus found out it, he had become so enraged that he commanded Thanatos, or
Death, to imprison Sisyphus in the (7. WORLD) ….…........... so he wouldn't cause any more trouble.
Sisyphus, on the other hand, lived up to his crafty reputation. The monarch requested Thanatos to demonstrate
to him how the chains worked as he was going to be (8. PRISON) ….…........., but Thanatos instead shackled
him before fleeing back among the living.

KEY:
1, mythology 2, harshness 3, familiarity 4, hospitality
5, punishment 6, kidnapped 7, underworld 8, imprisoned

Throughout the history of the world, coastlines were exposed to enormous, (1. DEVASTATE) ….
…........ waves crossing the oceans: the Tsunamis. In the 20th century tsunamis typically transpire with little or
no warnings, even with monitoring systems (2. STABLISH) …..……..... . Hundreds of thousands of
individuals have been slaughtered over the millennia. In contemporary times, from ancient Greece, North and
feudal Japan, pre-conquest, through Alaska, India, and Sri Lanka. It is the result of volcanic eruptions, terrible
events, enormous earthquakes, and (3. METEOR) …..……..... influences on the oceans. The energy
generated is similar to the collection and disposal of the whole territory, buildings, and people in New York
City.
A tsunami is a series of waves, not a single one, but rather a sequence of (4. CREDIBLE) …..
…….... lengthy waves that can stretch up to 200 kilometers in distance. Tsunamis may travel at speeds of up to
800 kilometers per hour, which is faster than a passenger plane. Even after traveling thousands of kilometers, a
tsunami can reach a height of 25 or 30 meters when it approaches shallow water. A tsunami could sometimes be
(5. LEAD) …..……..... . Water can recede from the beach as if it were an (6. EXCEPT) …..……..... low
tide if the trough of the first wave is the first to arrive. People who investigate the recently exposed seabed are in
jeopardy when the massive crest arrives. And if the beach is flat enough, the tsunami continues inland,
destroying cities and villages in seconds.

A worldwide tsunami warning system would not be able to prevent every death caused by these waves.
Nonetheless, if a sufficient warning system is (7. COMPLISH) …..……..... after such an (8.
OCCUR) …..……....., individuals on the other side of the world could have several hours to respond and
escape with their lives.

KEY:
1, devastating 2, established 3, meteorological 4, incredibly
5, misleading 6, exceptionally 7, accomplished 8, occurrence

That is truly a fangtooth is gliding beneath you. It has the largest tooth-to-body size ratio in the ocean. Since
food is available in the Twilight Zone, predators have developed terrifying adaptations to make every assault
count. The skin of the fangtooth is ultra-black, reflecting less than 0.5 percent of the light that strikes it, making
this fish extremely difficult to see. Fortunately, the bioluminescent organs that line your stomach are
programmed to match the strength and pattern of the faint light from the ocean's surface. Your silhouette is
obscured by this counter-illumination. As a result, when the fangtooth looks up, you are practically invisible.
In the distance, a brilliant light approaches. You proceed to investigate, but something is looming in the
darkness. The light is a bioluminescent lure used among anglerfish. You dart away before it opens its teeth and
swallows an unlucky fish that strayed too close. Another narrow escape, but you are getting close.
Whereas only 90% of deepwater animals are bioluminescent, only a group generates red color: dragonfish.
One is close to it. As saltwater rapidly absorbs red light, it can't be seen by most deep-sea organisms. The
dragonfishes emanate red light from the eyes of the organ, employing an invisible spotlight that finds unwary
victims scooping into his cavernous maw. You can't even watch your enemy, but your silver, mirror-like scales
are scattered, and you keep your light gentle.

KEY:
1, mythology 2, harshness 3, familiarity 4, hospitality
5, punishment 6, kidnapped 7, underworld 8, imprisoned
Chimpanzees are so similar to us that we are captivated and repulsed at the same time. They are our
deepest connections with nature, and we recall our prehistoric ancestors when we gaze at them. Their
social life is strongly similar to ours and on the one hand, we have political power battles and emotional
devotion. They attempt to communicate with us with comparable motions and expressions.
Furthermore, their development of instruments forced us to revisit our humanity definition. Their
development of sophisticated culture, handed on to their young people, is another feature they share
with us. Medicine seems to be within their grasp as well. When we perceive the spirit of a chimpanzee,
it helps us to realize that we are not alone.

• Complete the passage with a suitable participle form of the verbs in brackets:
In 10,000 years the human race (1. EXPAND) ____________ to approximately 7 billion from around 10 million
people. Some of them live even worse than in the Stone Age. But the overwhelming majority of them (2. FEED)
____________ and housed far better than their predecessors. In 2110 mankind (3. WILL / PROBABLY)
____________ far better off than it is now, and so would our planet's environment. This is not a fashionable, but
(4. CONVINCE) ____________ perspective, which I will term reasonable optimism. The belief is that the
world (5. EMERGE) ____________ from economic and ecological crises because the markets for commodities,
services, and ideas enable the interchange and specialization of people for everyone's improvement. But such
conversation often gets (6. DROWN) ____________ by the relentless drumbeat of gloom. Indeed, you (7.
REGARD) ____________ embarrassingly insane if you dare suggest that the world (8. HELP) ____________
to progress you.

KEY:
1, has expanded 2, are fed 3, would probably be 4, convincing
5, will appear 6, drowned 7, are regarded 8, will help
unhesitating

• Read the following passage and mark the letter (A, B, C, or D) on your answer sheet to
indicate the correct word or phrase that best fits each of the numbered blanks.
This was the last victory for a (1) ….…............ politician nominated that Yoshihide Suga, who after decades had
made the highest political position in Japan. Suga is not of the country's elite political family, as is (2) …..…...........
previous Japanese Premier Ministers, including its predecessor, Shinzo Abe. He is the son of a strawberry farmer. Its
advocacy (3) …..……....... a change in method, but not in substance.
As an authentic right-hand global leader. That comes naturally to Shinzo Abe, who can certainly leave office as Japan's
(8) ………......... prime minister. But I would want to convey my (9) …………..... appreciation to everyone who has
helped me during both tough and man to Abe, the pandemic has been postponed and is now threatening to collapse and the
(4) …..……........ Olympics in Tokyo will pursue the same conservative economic policies while using its recognized
problems to determine talents to the (5) …..….…....... crisis of Covid-19. It was Suga, as a prominent Cabinet Secretary,
a famous person on a domestic political arena, who announced last year's Emperor succession, which gave us the moniker
for the new imperial period. He (6) …..……........ at an audience of Emperor Naruhito in the Imperial Palace as Prime
Minister.
While he is hardworking and dependable, many people assume he lacks the charm required for the job, (7) …..
……........ when interacting with other difficult times. The Suga government will be inaugurated today. I would like to
appeal for your (10) …..……....... support for the ruling LDP party. Suga's function is to empower as continuity prime
minister for the next year before elections are celebrated. Suger must demonstrate that he has the relevant leadership
abilities in light of Japan's increasing difficulties.

1, A. handmade B. self-sustained C. self-made D. self-formed


2, A. the matter of B. the case of C. the instance of D. the cause of
3, A. represents B. symbolizes C. portrays D. designates
4, A. ill-obtained B. inadequate C. insufficient D. ill-disposed
5, A. prolonged B. proceeding C. taking place D. ongoing
6, A. was verified B. was confirmed C. had confirmed D. verified
7, A. particularly B. especially C. exceptionally D. reasonably
8, A. oldest continually B. longest-lasting C. longest-serving D. longest-term
9, A. heartfelt B. sincere C. profound D. wholehearted
10, A. unshakeable B. unfaltering C. unreliable D. unwavering

WORDFORM

1, The concept of strong resolve and ______________ implementation of discipline established after consultations tends
to be associated with the principle of consultative governance. (HESITATE)
2, Their meager physical experiences, along with their meager cerebral ones, resulted in a massive negative total that
______________ their wholesome morality and
deducible healthful sports. (BALANCE)
overbalanced
3, These folks have rejected their private faith as a load of nonsense and ________________ adopted Buddhism, which
vindicatory
they only have a rudimentary understanding of. (ENTHUSIASM)
4, Due to the ______________ of statistical methods to macroscopic systems, it is accurate that associating entropy with a
room's disarray might be deceptive. (APPLY)
5, Manufacturers can reduce the sugar level of enthusiastically
a drink by drawing it pink, relying on our ______________ connection of
irritability
color with sweetness to make up the difference. (CONSCIOUS)
subconscious
6, From the bottom to the top of a hill, it seems to be a virtually ______________ narrow slot in the pavement that runs
inapplicability
alongside a sidewalk. (PERCEIVE) federalization

7, The disturbing reality is that a successful, ______________ publicized, and explicitly attributed piece of design may
immediately become an artifact covered by eager practitioners. (UBIQUITY)
8, Because the incumbent legislative framework is antiquated, overcomplicated, and _____________, the government is
committed to modernizing aquaculture legislation. imperceptible
(FUNCTION)
ubiquitously
9, If any _______________ tissue remained after the initial 6 minute disintegration period, the given sample has
remained for an additional 3 - 6 minutes. (HOMOGENY)
10, Aside from that, the original white plaster has long since succumbed to damp, salty circumstances, and its flawless
white skin has peeled away, peeling most _______________ into strips and cracks. (APPEAL)
dysfunctional
11, When ________________ attuned, shared personal history may also contribute to sentiments of trust, which can assist
to initiate risk-taking, which can lead to spontaneous musical utterances. (EMPATHY)
unhomogenized
12, An ______________ consumer not only safeguards his interests but also serves as a responsible and responsive
customer, as well as a valuable contributor to society. (LIGHT)
13, One of the most prevalent practices of freedom for the post-colonial subject is the ______________ of memory as a
means by which to expose the hidden histories of violence. unappealingly
(INTER)
14, Some interiors are _______________ classified in a acculturate Amerindian population known pejoratively under the
empathetically
name of cholos, which refer to themselves as natural.
(PERCEIVE)
15, Suddenly, there is a man who is _______________ exceptional in terms of health and vigor, and he provides females
enlightened
who mate with him an inherited healthiness in their children that is much beyond normal. (MISTAKE)

disinterment

imperceptibly
WORDFORM (C2)

1, This is simply because what Meinong refers to as the act of thinking is not empirically discoverable or logically
unmistakably
_____________ from whatever we can witness. (DEDUCE)
2, The principal _____________ are exchanged between markets and between sectors which is the most important aspect
of a market system that may be used as a source of support.
(VINDICATE)
3, Symptoms of lethargy and _____________ on presentation typically contribute to a diagnosis of sepsis or
meningoencephalitis in infants with botulism. (IRRITATE)
4, As opposed to maintaining peace and order, non-consensual _____________ must be done in the signature of
protecting judicial authority or due process. (FEDERAL)
5, In contrast with the violence of British settlement, the Macassan encounters were generally amicable, probably because
their ____________________________ was perceived as less threatening. (PERMANENT)
6, Divine _____________, like omnipotence and omnipresence, does not admit itself easily to analogical translation to
creaturely existence, which is bound up in time and change.
(MUTABLE)

• Complete the text with the correct form of the passive. More than one answer could be possible:

The African lion is an iconic symbol of Africa. Historically, lions (1) _____________ (distribute) throughout the
Mediterranean, the Near and Middle East as far as India, and all of Africa. By 100 AD, they (2) _____________
(eliminate) from their last European strongholds in Greece, but survived until the 20th century in Syria, Iran, and Iraq.
The last lion in Iran (3) _____________ (say) to (4) _____________ (shoot) in 1942. In Africa, reduced numbers of
lions are still present in the north of South Africa and Namibia, and south of the equator. Lions are difficult to count
Impermanence
accurately and any survey figures can best (5) _____________ (describe)/ impermanency
as ‘guesstimates' but it is evident that the lion
population In Africa (6) _____________ (currently / threaten) by habitat encroachment because of increased land
cultivation The hope is that, in the future, traditional habitat protection methods (7) _____________ (support) by
immutability
translocation and reintroduction programs. Fortuitously, a certain amount of progress (8) _____________ (already /
make) in this area.
• TRẮC NGHIỆM: (ĐÁP ÁN)
Câu 1: Các nhà thiên văn học ______________ một bức ảnh mới về một trong những vụ va chạm giữa
các thiên hà lớn nhất chưa từng được ghi lại.
A. đã xuất bản B. đã công bố C. đã công khai D. đã ban hành

Câu 2: Đại dịch Covid-19 đã đưa ra những câu hỏi ______________ về phân biệt chủng tộc và địa lý,
công bằng sức khỏe và sự cân bằng giữa các yêu cầu y tế công cộng và quyền lợi cá nhân.
A. không có hồi kết B. không bờ bến C. không thể đo lường D. bất diệt

Câu 3: Hiện nay, việc dành quá nhiều thời gian để chơi trò chơi điện tử ______________ tiêu cực đến
việc học hành của trẻ và sự phát triển xã hội của trẻ em đã trở thành một điều thông thường.
A. có sức mạnh B. có hệ quả C. có ảnh hưởng D. có sức chi phối

Câu 4: Những học sinh muốn thi lại hoặc ______________ điểm số của mình để được nhận vào một cơ sở
giáo dục có uy tín đôi khi dành kỳ nghỉ hè trước mùa thi để hoàn thành các khóa học ôn tập.
A. sửa đổi B. cải cách C. tu chính D. cải thiện

Câu 5: Các công ty công nghệ đang tạo ra hiệu quả kinh tế xuất sắc, nhưng khách hàng muốn
______________ phần cứng lỗi thời bằng thứ gì đó tiên tiến hơn để vận hành máy tính đúng cách.
A. thay thế B. dành chỗ C. đổi chổ D. luân phiên đổi

Câu 6: Ông nội sẽ dành hàng giờ để nói chuyện với chúng tôi về những người trẻ tuổi xung quanh bàn
ăn tối để ______________ về những ngày còn trẻ hạnh phúc của ông ở nông trại.
A. nhớ lại B. hồi tưởng C. nhắc nhở D. ghi nhớ lại

Câu 7: Đội chiến thắng đã bị giới truyền thông địa phương chỉ trích kịch liệt vì cách họ đã
______________ trước đội thua. Nó đã được coi là rất không chuẩn mực.
A. hài lòng B. thể hiện C. chiếm ưu thế D. hả hê

Câu 8: Ba trong số những kẻ trốn thoát, hiện đã được bắt lại nhưng người phát ngôn của cảnh sát cho
biết họ lo ngại rằng, về tù nhân thứ tư, thông tin ______________ được biết có thể giúp chính quyền đưa
anh ta trở lại sau song sắt.
A. hiếm hoi B. nhỏ nhoi C. ít ỏi D. một chút

Câu 9: Các cuộc biểu tình của phe đối lập chống lại chính phủ hiện có nguy cơ ______________ và lực
lượng an ninh có thể sẽ thực hiện các vụ bắt giữ trên khắp đất nước trong ngày hôm nay.
A. vượt khỏi tầm kiểm soát B. vượt khỏi tầm với
C. trở nên bất thường D. trở nên bùng nổ

Câu 10: Mặc dù khách sạn là một nơi ______________ với cửa sổ nứt, gió lùa đóng băng và ván sàn ọp
ẹp, nhân viên rất tuyệt vời và tất cả chúng tôi đã có một khoảng thời gian tuyệt vời.
A. thực sự u ám B. tồi tàn thực sự C. suy nhược thực sự D. thực sự suy tàn

Ví dụ: Nhiều người có phản ứng dị ứng với cá và có thể ______________ đổ mồ hôi, mạch đập nhanh và
cảm thấy buồn nôn khi chất độc ngấm sâu hơn vào cơ thể.
A. bắt đầu B. lúc đầu C. trở nên D. khởi đầu
• WORDFORM: (C1 – C2 LEVEL)

USING VIDEO GAMING IN EDUCATION


It has become conventional wisdom that spending too much time playing video games has a negative influence on
children’s education and their social development. Some educationalists, nevertheless, are increasingly challenging this
idea and employing video games as effective educational tools, bridging the gap between leisure and teaching activities.
As a consequence of the obvious refinement of today's games, teachers may justify the inclusion of video and internet
games for a variety of educational reasons. There might be social, psychological, and ethical consequences integrated into
the games, for example. Harvey Edwards, an IT teacher in London, was one such teacher who determined to incorporate
video games into his teaching. He picked Minecraft, an internet game in which users construct and explore fictional
worlds, to do this. He was hesitant to use such an unorthodox method, not because some pupils were unfamiliar with the
game, but because they couldn't understand what he was seeking to accomplish with it. He was concerned that it might
disrupt his students' concentration, but he was pleasantly pleased by the outcomes.
Minecraft is an example of a 'sandbox game,' in which players may willingly wander and modify a virtual
environment. Instead of needing to go through numbered levels to reach particular areas, there is unlimited access from
beginning to end. The original version may be customized to determine which characters and information remain. Each
student is then assigned tasks such as housebuilding, item location, or problem-solving, which they must perform within
the game. Elements of more general abilities, such as online etiquette and safety, teamwork, and conflict resolution, can be
delicately included in the courses. Edwards believes that providing such teachings in the context of games that students are
likely already familiar with and adore enables him to connect with them more deeply and motivate them.
Nevertheless, not everyone is convinced by the potential intellectual benefit of video games. For the period that many
progressive critics point to considerable evidence that video games enhance teamwork and problem-solving abilities, more
conventional groups continue to pretend that video games are a distraction that should not be included in any classroom
curriculum. Cynics who are less fervent in their beliefs, but who acknowledge that games could have some educational
value, insist that this is only valid in the hands of instructors who are innovative and imaginative. A common complaint
about video games is that they interfere with social interaction in the classroom, especially when it comes to debates.
According to Dr. Helen Conway, an educational researcher, video games can be utilized to encourage social activities.
‘Students become enthusiastic while discussing the game and how to enhance their game playing and problem-solving
skills,' she says. 'I find this picture that many people hold disturbing,' Conway responds. ‘Children are frequently detached
from their classmates when engaging in more conventional hobbies, such as reading books, but we certainly not argue that
books are detrimental because they are a solitary experience.'

+ KEY:
1, conventional 2, educationalists 3, increasingly 4, refinement
5, unorthodox 6, unfamiliar 7, pleasantly 8, willingly
9, unlimited 10, customized 11, delicately 12, considerable
13, acknowledge 14, innovative 15, imaginative 16, imaginative

• VERB-FORM: (B2 – C1 LEVEL)


Nuuk is different. For a start, it's not connected to other places in Green land. If you want to quit town, you need a boat, a
helicopter, or a propeller plane. Nuuk surely is off the beaten track. On the other hand, we 've got mountains, the coastline,
a vast expanse of water and icebergs - a landscape that's as fascinating as it is inspiring, above all for artists. And there are
plenty of those here. I love the albums of Ole Christianson. His surreal lyrics are wonderful. He sings in Greenlandic but
his lyrics alone were reason enough to encourage people to engage with our language.
I like to start my day at the only place in Nuuk where you can get respectable coffee. The espresso is strong and tastes
excellent. But although it's difficult to find good coffee, it's easy to fall hook, line, and sinker for Greenlandic cuisine. In
my favorite restaurant located on the harbor, you get smoked reindeer, fish, and musk ox. Places such as Qoornoq are just
as evocative as the local cuisine. It's a deserted village on an island just off Nuuk. There's nothing there apart from a few
holiday homes. But take heed if you go there: the clouds of mosquitos can be surprisingly bothersome in summer. That's
why people use mosquito nets on boat trips. It looks pretty funny but, as I said, Nuuk is just different.
+ KEY:
1, connected 2, quit 3, 've got / have got 4, were
5, smoked 6, is nothing 7, looks 8, said
then

Good evening and welcome. Tonight, I (1. APPEAL) …..….......... to everyone out there watching this! This live
Helpline Round-the-clock Special is in aid of the famine-stricken province we (2. HEAR) ….……........ so much about
recently – and (3. SEE) …….......... on our TV screens night after night. And, believe me: however little you pledge –
and we will make sure that it reaches its destination – (4. MAKE) ….……........ a tremendous difference.
Last year, in between giving concerts to raise money, I personally (5. GO OUT) ….……........ to supervise the
deliveries of aid to stricken
appear areas, and I know from my own first-hand experience that whatever we do is a mere drop in
/ seem
the ocean – we can never, ever send enough to help these people. But what we do send (6. BE / GREATLY) ….
……........ appreciated, so it all depends on you. Pick up the phone. The number to call will be appearing on your TV
screens shortly and (7. BE / ANNOUNCE) ….……........ on radio programs throughout the evening. You can ring and
quote your credit card number or, if you can’t get through, just send us a cheque or make an online donation, and you can
sleep easily in your bed knowing that (8. DO) ….……........ your bit for Helpline!

+ KEY:
1, am appealing 2, have been hearing 3, seeing 4, will make
5, went out 6, will be greatly 7, will be announced 8, have done
can / may

• For questions 1 – 36, read the text below and think of the word which best fits each gap.
Use only one word in each gap. There is an example at the beginning (0).
+ Passage 1:
But (0) …..……...... again, if you accept (1) ………....... human contribution to global warming is relatively minor, then
(2) .…..…....... policies we implement, including (3) ………........ reducing carbon emissions (4) …..…........ reducing
air travel and changing the (5) …….…..... we power our factories so we consume fewer sources of energy, or attempting
to reduce the (6) …..…..... of greenhouse gases, et cetera, is misplaced and we could be more efficient.

+ Passage 2:
But you (0) …..……………...... to be implying that (7) ………....... of the pollution generated by vehicle traffic and the
consumption of fossil fuels, (8) ………....... other things, does not influence what is (9) ………....... to the world's
climate. (10) ………....... that the majority of evidence suggests the contrary, this seems to be a pretty unproductive
perspective. Look, the truth is that the world is warming, consequently, we need to (11) ………....... taking action straight
now to figure out what's causing it. (12) ………....... if we do not prioritize this action, our planet's future will be
jeopardized.

+ Passage 3:
I believe it (0) …...………….... be a variety of things. When (13) ………....... people think of culture, they think of the
arts, literature, media, and television. (14) ………....... lived in many various nations and cultures, I suppose it is
genuinely the behavior of specific individuals in different countries and (15) ………....... the general behavior in that
country has impacted the way they live. For instance, family traditions. Some countries may have a stronger family
orientation – perhaps it is entrenched in (16) ……..…....... culture to sit down, have family dinners, and be extremely
family-oriented. (17) ……..…....... in Western civilization, it is entirely possible that we don't even have similar cultures
and that the people you know don't have (18) ………......... family values, that people may not eat together...
+ Passage 4:
Scientists (0) …..……… frequently accused of (19) ………....... poor communicators, despite the fact that there are
several (20) ………....... why scientists, in particular, should be and typically are effective communicators. (21) ..
……........ all, science necessitates passion, and scientists frequently exude it in spades. Enthusiasm may be contagious,
but to pique the interest of readers, scientists must improve their other natural abilities: clarity, observation, and
knowledge.
Scientists who think logically can generally write (22) ………....... well, and the more precise concepts are expressed, the
more valuable they may be. Similarly, people who observe (23) ………....... account for subtle differences to establish
important observations. Finally, people who write must have something to say that is significant in and of itself.
A scientist (24) ………....... work never sees the light of day has accomplished nothing of value until someone else hears
about it. Scientists must, therefore, eliminate the misconception that they are unable to communicate, once and for all.

are
+ Passage 5:
before
More people fly today than ever (0) …..…………, yet many – experienced air travelers as well as novices – suffer
anguish and apprehension at the mere thought of flying. As many as one out of seven people are thought to experience
anxiety when flying, with women outnumbering men two to one in these feelings of disquiet.
A certain amount of concern is understandable. The sheer size of modern jet aircraft, which appear awkward and unwieldy
on the ground, makes one wonder how they will manage to get into the air – and stay there. Most of these fears are
illogical and are perhaps based on the knowledge that once in the aircraft, we, as passengers, are powerless to control our
fate, which depends solely on the expertise of the crew. There is little comfort for us in the numerous statistical
compilations which show that modern air transport is many times safer than transport by car or rail.
Most people's fear remains on a manageable scale. For others, however, the anxiety can become an overwhelming fear,
known as aviophobia. Symptoms include feelings of panic, sweating, palpitations, depression, sleeplessness weeping
spells, and sometimes temporary paralysis.

Early civilizations, like the later ones, seem to have a common positive characteristic in that they change the human scale
of things. They bring together the cooperative efforts of larger numbers of men than any earlier societies and usually
achieve this by physically bringing them together in larger agglomerations, too. As the word 'civilization' suggests, it is
usually marked by urbanization. Admittedly, it would be a bold man who was willing to draw a precise line at the moment
when the balance tipped from a dense pattern of agricultural villages clustered around a religious center or a market to
reveal the first true city.
Nevertheless, it is perfectly reasonable to say that more than any other institution the city has provided the critical mass
which produces civilization and that it has fostered innovation better than any other environment so far. Inside the city, the
surpluses of wealth produced by agriculture made possible other things characteristic of civilized life. They provided for
the upkeep of a priestly class which elaborated a complex religious structure, leading to the construction of great buildings
serving more than merely economic functions, and eventually to the writing down of literature.
+ KEY: (Passage 1 – 6)
1, that 2, any 3, such 4, by / with 5, way 6, number

7, all 8, among/amid 9, happening 10, Given 11, start 12, And

17, Whereas
13, most 14, Having 15, how 16, their 18, such
Whilst/ Although

22, rather 23, must / ought to /


19, being 20, reasons 21, After 24, whose
quite/ pretty have to

25, 26, 27, 28, 29, 30,

31, 32, 33, 34, 35, 36,

• WORDFORM:
1, Whether a book is written by a _____________ author or not makes no difference whatsoever to me. If it appeals to
me, I'll read it. (KNOW)
2, In most books about English, the joke would be turned on Charles, used to preface the ____________ (OBSERVE)
that the language he dismissed as ____________ (CULTIVATE) is now a colossus bestriding the world.
3, In terms of the travel, the students were happy and _____________, although I would recommend a direct flight next
time, even if it proves more expensive. (BEHAVE)
4, Reports of unusual animal behavior prior to the _____________ of earthquakes have been recorded in literature dating
as far back as 1784. (OCCUR)
5, Consequently, school work has replaced paid work, and the period of children's total ____________ (DEPEND) on
their parents has ____________ (CORRESPOND) expanded.
6, If current trends continue, within a very short time, most countries will have _____________ this kind of punishment,
even in the home. (LAW)
7, Ironically, children of all classes are now traveling further afield than ever before - including to other countries - yet the
______________ (DEPEND) movement is ever more curtailed.
8, Vietnam's coastlines and islands are currently _____________ difficult changes that seriously threaten the country's
territorial sovereignty. (EXPERIENT)
9, Children can become more ethical and ______________ if they resist temptation regularly, according to research from
the late 1960s. (DISCIPLINE)
10, New Urbanist communities are intended to be more than residential ______________, with shops, a wide range of
personal and consumer services, and workplace sites all conveniently accessible on foot. (DIVIDE)
11, Industrial relations in football in the British appear to be associated with a type of language that makes objective
evaluation impossible and, by its very nature, creates ______________ . (SENT)

+ KEY:
1, well-known 2, observation/uncultivated 3, well-behaved 4, occurrence

5, dependency
6, outlawed 7, independent 8, experiencing
correspondingly

9, self-disciplined 10, subdivisions 11, resentment 12,

13, 14, 15, 16,

17, 18, 19, 20,

+ KEY:
1, Some tourists are hoping to be reimbursed for the hotel's poor state, and I think they have a strong
case.
2, It has become conventional wisdom that children's education and social development are negatively
impacted by spending too much time playing video games.
+ PASSAGE: (B2)

Cao Ba Quat is remembered in history for more than his talent or more
exquisite phrases than anyone. At the time, and afterward, he was honored
and admired for his outstanding personality, turbulent temperament, notably
liberal, clear-minded, and desires to rise above the confined times and live
effectively, meaningfully. Nevertheless, because he lived during a period of
acute crisis in Leism, Cao Ba Quat was soon forced to present the tragedy of
intellectuals who embraced many beautiful ideals but were eventually
disillusioned and stuck on the road he chose. The short song "Walking On
The Sand" is a joyous tune.
In the dark, obscure years of feng shui, the image of the sand and people
walking on it represents the poet as well as many current thinkers. Despite the dismal stalemate,
anger has revealed strong symptoms of a valuable awakening of modern soldiers before the road of
traditional renown and before social reality.
The image of sand and people walking on it are distinct and important artworks that result from the
understanding of nature and social reality, as well as from the realization of Cao Ba Quat's mood. In
addition to conveying the poet's beliefs, these pictures represent the struggle of people at one time,
and frequently the same predicament, again and over again. It's a tremendous movement in the
creative idea of the age, especially in light of the feng shui philosophy that envelops its black
shadow into human consciousness.

• Complete the second sentence so that it has a similar meaning to the first sentence, using
the word given. Do not change the word given. You must use between three and eight words,
including the word given.
1, Some of the tourists are hoping to get compensation for the poor state of the hotel, and I think they have a very strong
case. (REIMBURSED)
→ Some tourists ……………………….……………………………..………….….…. hotel's poor state, and I think they
have a strong case.
2, It has become conventional wisdom that spending too much time playing video games has a negative influence on
children’s education and their social development.
(IMPACTED)
→ It has become conventional wisdom that children's education and ………………………………
………………………………………...……… spending too much time playing video games.
3, There is little to disagree about in the notion that a good voice, whether in opera or rock music, is one which moves its
audience and brings a sense of release and fulfillment to the singer. (REIMBURSED)
→ Some tourists ……………………….……………………………..………….….…. hotel's poor state, and I think they
have a strong case.
• WORDFORM:
It appears that industrial relations in football in Britain are tied to a form of language that makes measured assessment
difficult and causes (1. FEEL) ___________ by its very nature. Just as player-manager relations are conducted in the (2.
DATE) ___________ language of the traditional factory floor; so the terminology used to describe changing Jobs,
'buying' and 'selling' players, distorts the reality. Both sides suffer from this: the management accuses some players of
greed or (3. LOYAL) ___________ , while the club treats them (4. CYNIC) ___________ as expendable objects.
People migrate from one job to another all the time in the real world. They are neither purchased nor sold; instead,
they resign, sign a new contract with another company, or transform. If they have signed a long-term contract, their former
employers may refuse to let them depart or seek (5. COMPENSATE) ___________ . In reality, life in the corporate
world is typically less (6. PAY) ___________ , less secure, and more demanding than life in professional football. The
animosity that players feel about being "sold" is most probably triggered by the terminology used to describe the process
rather than by the process itself. Everybody tends to into stereotypes: the gentleman chairman who considers himself as a
role model for corporate conduct, and the (7. SENSE) ___________ player who believes he is being treated as a (8.
DISPOSE) ___________ commodity, to mention a few.

+ KEY:
1, ill-feeling 2, outdated 3, disloyalty 4, cynically
5, compensation 6, well-paying 7, hypersensitive 8, disposable

You might also like